NCLEX Study Set 2

Lakukan tugas rumah & ujian kamu dengan baik sekarang menggunakan Quizwiz!

6. When conducting research, information is collected. This would be identified as which of the following? A. Subject b. Analysis c. Data d. Abstract

c. Data are observable and verifiable information collected to describe, explain, or predict events

Fluids are the most important intervention with HHNS as well as DKA, so get fluids going first. With HHNS there is no ketosis, and no acidosis. Potassium is low in HHNS (d/t diuresis).

...

For HIV kids avoid OPV and Varicella vaccinations (live), but give Pneumococcal and influenza. MMR is avoided only if the kid is severely immunocompromised. Parents should wear gloves for care, not kiss kids on the mouth, and not share eating utensils.

...

Four side-rails up can be considered a form of restraint. Even in LTC facility when a client is a fall risk, keep lower rails down, and one side of bed against the wall, lowest position, wheels locked.

...

Give neostigmine to clients with Myesthenia Gravis about 45 min. before eating, so it will help with chewing and swallowing.

...

Give synthroid on an empty stomach

...

Gonorrhea is a reportable disease

...

Group-a strep precedes rheumatic fever. Chorea is part of this sickness (grimacing, sudden body movements, etc.) and it embarrasses kids. They have joint pain. Watch for elevated antistreptolysin O to be elevated. Penicillin!

...

HbA1c - test to assess how well blood sugars have been controlled over the past 90-120 days. 4-6 corresponds to a blood sugar of 70-110; 7 is ideal for a diabetic and corresponds to a blood sugar of 130.

...

High potassium is expected with carbon dioxide narcosis (hydrogen floods the cell forcing potassium out). Carbon dioxide narcosis causes increased intracranial pressure.

...

Hirschsprung's diagnosed with rectal biopsy looking for absence of ganglionic cells. Cardinal sign in infants is failure to pass meconium, and later the classic ribbon-like and foul smelling stools.

...

Hyperactive deep tendon reflexes, vision changes, fatigue and spasticity are all symptoms of MS

...

Hypotension and vasoconstricting meds may alter the accuracy of o2 sats.

...

Iatragenic means it was caused by treatment, procedure, or medication.

...

If a TB patient is unable/unwilling to comply with tx they may need supervision (direct observation). TB is a public health risk.

...

If one nurse discovers another nurse has made a mistake it is always appropriate to speak to her before going to management. If the situation persists, then take it higher.

...

If you can remove the white patches from the mouth of a baby it is just formula. If you can't, its candidiasis.

...

If you gave a toddler a choice about taking medicine and he says no, you should leave the room and come back in five minutes, because to a toddler it is another episode. Next time, don't ask.

...

If your laboring mom's water breaks and she is any minus station you better know there is a risk of prolapsed cord.

...

If your normally lucid patient starts seeing bugs you better check his respiratory status first. The first sign of hypoxia is restlessness, followed by agitation, and things go downhill from there all the way to delirium, hallucinations, and coma. So check the o2 stat, and get abg's if possible.

...

In a five-year old breathe once for every 5 compressions doing cpr.

...

In emphysema the stimulus to breathe is low PO2, not increased PCO2 like the rest of us, so don't slam them with oxygen. Encourage pursed-lip breathing which promotes CO2 elimination, encourage up to 3000mL/day fluids, high-fowlers and leaning forward.

...

Intussusception common in kids with CF. Obstruction may cause fecal emesis, currant jellylike stools (blood and mucus). A barium enema may be used to hydrostatically reduce the telescoping. Resolution is obvious, with onset of bowel movements.

...

It is always the correct answer to report suspected cases of child abuse.

...

It is essential to maintain nasal patency with children < 1 yr. because they are obligatory nasal breathers.

...

Lasix can cause a patient to lose his appetite (anorexia) due to reduced potassium.

...

Level of consciousness is the most important assessment parameter with status epilepticus.

...

Listen to FHR tones with a STETHOSCOPE in NCLEX land.

...

You better be making sure that patient on Dig and Lasix is getting enough potassium, because low potassium potentiates Dig and can cause dysrrhythmias.

...

You will ask every new admission if he has an advance directive, and if not you will explain it, and he will have the option to sign or not.

...

Your cancer patient is getting radiation. What should you be most concerned about? Skin irritation? No. Infection kills cancer patients most because of the leukopenia caused by radiation.

...

a kid's hinder should clear the bed when in Bryant's traction (also used for femurs and congenial hip for young kids).

...

9. A pt is hying dyspnea. What would the nurse do first? a. Remove pillows from under the head b. Elevate the head of the bed c. Elevate the foot of the bed d. Take the blood pressure

...b. Elevating the head of the bed allows the abdominal organs to descend, giving the diaphragm greater room for expension and facilitating lung expansion.

Creatinine - Lab Values: Child

.3-.7

Bilirubin - Lab Values: Adult

.3-1

Bilirubin - Lab Values: Child

.3-1

Creatinine - Lab Values: Adult

.6-1.1 M .6-1.2 F.5-1.1

Normal BNP?

0-100 (CHF)

What is a normal ICP?

0-15mmHg

Normal ALT? AST?

0-31; 0-32 (liver enzymes)

Urine: WBC - Lab Values

0-4 per low level power field

Normal Lipase? Amylase?

0-59; 30-110 (pancreas)

Normal bilirubin level?

0.3-1.0

Normal TSH level?

0.3-5.2

Lithium

0.5-1.5

Digoxin

0.5-2.0

Therapeutic range for Digoxin?

0.5-2.0

Normal Lactic Acid?

0.5-2.0 (shock)

Therapeutic range for Lithium?

0.8-1.5

How are deep tendon reflexes graded?

0=absent, 1+ or + = diminished and abnormal, 2+ or ++ = is brisk and normal, 3+ or +++ = hyperactive without clonus and abnormal, and 4+ or ++++ = hyperactive with clonus and abnormal

How is a pulse graded?

0=absent...1=diminished...2=normal...3= increased...4=bouding

Advanced practice nurses generally: 1. function independently 2. function as unit directors 3. work in acute care settings 4. work in the university setting

1

3. What historic event in the 20th century led to an increased emphasis on nursing and broadened the role of nurses? a. Religious reform b. Crimean War c. World War II d. Vietnam War

c. During World War II. Large numbers of women worked outside the home. There was increased emphasis on education and a knowledge explosion in medicine and technology, broadening the roles of nurses.

what is the nursing care for cocaine withdrawal?

maintaining airway, starting IV, cardiac monitoring, checking LOC and VS, monitoring for suicidal ideation, keeping in calm, quiet environment

Most important thing we can do before we start our new job

malpractice insurance

Nephrotic syndrome is characterized by:

massive proteinuria (looks dark and frothy) caused by glomerular damage. Corticosteroids are the mainstay. Generalized edema common.

Administration of intraocular disk

position convex side on fingertip place on conjuctival sac btw iris and lower lid gently pull eyelid over disk carefully pinch disk to remove from patient's eye

Infant w/ Cleft lip

position on back or in infant seat to prevent trauma to suture line. While feeding, hold in upright position.

After Myringotomy

position on side of affected ear after surgery (allows drainage of secretions)

Infant w/ Spina Bifida

position prone (on abdomen) so that sac does not rupture

After Infratentorial Surgery (incision at nape of neck)-

position pt flat and lateral on either side.

Administration of Enema

position pt in left side-lying (Sim's) with knee flexed

Tube Feeding w/ Decreased LOC

position pt on right side (promotes emptying of the stomach) with the HOB elevated (to prevent aspiration)

Cor pulmonale

right sided heart failure caused by left ventricular failure (so pick edema, jvd, if it is a choice.)/ caused by pulmonary disease, occurs with bronchitis or emphysema. - (s/s fluid overload)

What is the highest priority nursing diagnosis for patients in renal failure:

risk for imbalanced fluid volume

dihiscence

rupture separion of one or more layers of a wound.

Pernicious anemia

s/s include pallor, tachycardia, and sore red tongue.

what are the treatments for rheumatoid arthritis?

salicylates or nonsteroidal anti-inflammatories, analgesics, preservation of joint and muscle function, reconstructive surgery, corticosteroids, antidepressants (for depression, sleep, pain management), encourage balance bw rest and activity, promote independence, address self-concept issues

incision

surgical cut or wound produced by a sharp instrument

What is central neurogenic hyperventilation respirations?

sustained period of rapid, regular, deep respirations (hyperpnea)

List 4 common symptoms of pneumonia that the nurse might note on a physical exam

tachypnea, fever with chills, productive cough, bronchial breath sounds

SSRI's (antidepressants) take about 3 weeks to work

take about 3 weeks to work

What is a proper teaching plan for a patient taking nitroglycerin:

take medication at the first sign of anginal pain. Take no more than tree pills 5minutes apart, call 911 if no relief is achieved in 10 minutes

what are the nursing considerations for NSAIDs?

take with food or after meals, monitor liver/ renal fxn, avoid OTCs that contain similar meds

The mixed venous oxygen saturation (SvO2) is decreasing in a patient who has severe pancreatitis. To determine the possible cause of the decreased SvO2, the nurse assesses the patient's ____________

temperature. rationale: Elevated temperature increases metabolic demands and oxygen use by tissues, resulting in a drop in oxygen saturation of mixed venous blood. Information about the patient's weight, urinary output, and amylase will not help in determining the cause of the patient's drop in SvO2.

Think DROP(child drops to floor or squats) or POSH Defect, septal Right Ventricular hypertrophy Overriding aorts Pulmonary stenosis

tetrallogy of fallot

What is the normal pulmonary artery mean pressure?

the PAPm is normally 12-20 mmHg

Circadian Rhythm

the biological clock; regular bodily rhythms that occur on a 24-hour cycle

hematoma

the collection of blood under the skin as the result of blood escaping into the tissue from damaged blood vessels. bruise

When observing the support provided to a pt by a family care giver, it is important to keep in mind that:

the health of the caregiver may be negatively affected by the demands of care giving.

Normal Chloride?

96-108

When phenylalanine increases:

Brain problems occur

Heat or Cold? Reduced muscle tension

Heat

Heat or Cold? Vasodilation

Heat

What is Cranial Nerve III?

Oculomotor; eye movement

eclampsia

eclampsia is seizure in pregnancy

Urine RBC Casts - Lab Values

none

Low Density Lipoprotien - Lab Values: Adult

none listed

what is a normal hgb?

12-15 g/dL (women) 14-16.5 g/dL (men)

Activated Partial Thromboplastin Time - Lab Values: Adult

30-40

Activated Partial Thromboplastin Time - Lab Values: Child

30-40

what is a normal hct?

30-45% (women) 42-50% (men)

Skin flushed Agitation Low grade fever Thirst

Hyper natremia s/sx tip (SALT)

HCO3- Lab Values: Infant

I 20-28 NB 13-22

Potassium - Lab Values: Infant

I 4.1-5.3 NB 3-5.9

For Better Perfusion:

eleVate Veins dAngle Arteries

Head injury

elevate HOB 30 degrees to decrease intracranial pressure

After Supratentorial Surgery (incision behind hairline) -

elevate HOB 30-45 degrees

Buck's traction

elevate foot of bed for counter-traction

What is a normal temperature?

98.6 degrees Fahrenheit +/- 1 degree orally; rectal is normally slightly higher

guided imagery

nonpharmocologic that is great for chronic pain.

rheumatic fever

rheumatic fever can lead to cardiac valves malfunctions.

ICP (intracranial pressure)

ICP (intracranial pressure) should be <2. measure head circonference.

Meningeal Irritation:

S/S: Nuchal rigidity, positive Brudzinski + Kernig Signs and Photophobia

Above Knee Amputation

elevate for first 24 hours on pillow, position prone daily to provide for hip extension.

-As the nurse is preparing a patient for surgery, the patient refuses to remove a wedding ring. Which of the following is the most appropriate action by the nurse?" A. Note the presence of the ring in the nurse's notes of the chart. B. Insist the patient remove the ring. C. Explain that the hospital will not be responsible for the ring. D. Tape the ring securely to the finger."

". Explain that the hospital will not be responsible for the ring. It is customary policy to tape a patient's wedding band to the finger and make a notation on the preoperative checklist that the ring is taped in place."

Don't fall for 'reestablishing a normal bowel pattern' as a priority with small bowel obstruction. Because the patient can't take in oral fluids 'maintaining fluid balance' comes first.

...

"Which task would be most appropriate for the nurse to delegate to the unlicensed assistive personnel (UAP)? "1. Complete the preoperative checklist. 2. Assess the client's preoperative vital signs. 3. Teach the client about coughing and deep breathing. 4. Assist the client to remove clothing and jewelry.

"1. The nurse should complete this form because it requires analysis, which cannot be delegated to the UAP. 2. Nurses cannot delegate assessment. 3. THe nurse cannot delegate teaching to a UAP. 4. The UAP can remove clothing and jewelry."

"While completing the preoperative assessment, the male client tells the nurse that heis allergic to codeine. Which intervention should the nurse implement first? "1.Apply an allergy bracelet on the client's wrist. 2.Label the client's allergies on the front of the chart. 3.Ask the client what happens when he takes the drug. 4.Document the allergy on the medication administration record

"1. This is an important step for the nurse toimplement, but it is not the first intervention. 2.This must be done, but it is not the first inter- vention. CORRECT 3. The nurse should first assess the eventsthat occurred when the client took thismedication because many clients think that a side effect, such as nausea, is an allergicreaction. 4.This information must be put on the medica-tion administration record (MAR), but it is notthe first intervention"

A client is admitted to the hospital for surgical repair of a detached retina in the right eye. In planning care for this patient postoperatively, the nurse should encourage the patient to do which of the following? "1. Perform self-care activities. 2. Maintain patches over both eyes. 3. Limit movement of both eyes. 4. Refrain from excessive talking."

"1. activity restrictions depend on location an size of tear. 2. only affected eye covered 3. CORRECT - bed rest with eye patch or shield 4. no restriction"

The nurse is completing an admission assessment for a client admitted to the medical unit with a diagnosis of Acute Alcohol Intoxication. When asked to describe his drinking pattern and amount, the client states, "I only drink when I am under a lot of stress." The client's response indicates what defense mechanism? 1. Regression 2. Denial 3. Projection 4. Rationalization"

"4. Rationalization. Rationale: 1. Denial. Denial is used to protect a client from reality, especially unpleasant events in life. The client is not denying alcohol use. 2. Rationalization. Substituting acceptable reasons or explanations for real or actual problems that motivate a client's behavior is rationalization. The client is rationalizing his drinking by justifying why he needs to drink. 3. Projection. Attributing one's own unacceptable thoughts, feelings, or impulses to another person is projection. This client is describing his behavior as reasonable under the circumstances. 4. Regression. Returning to comfortable behavior that was used at an earlier point in one's life is regression. For example, a child who has been toilet-trained suddenly starts bedwetting following the parents' separation."

A hospitalized client with a history of alcohol abuse tells a nurse, "I am leaving now. I have to go. I don't want any more treatment. I have things that I have to do right away." The client has not been dischagred. The clientis scheduled for an inportant diagnostic test to be performed in 1 hour. After the nurse discusses the client's concerns with the client, the client dresses and begins to walk out of the hospital room. The appropriate nursing action is to: "A. Call the nursing supervisor B. Call security to block all exit areas. C. Restrain the client until the physician can be reached. D. Tell the client that the client cannot return to this hospital again if the client leaves now."

"A. Call the nursing supervisor A nurse can be charged with false imprisonment if a client is made to believe wrongfully that he or she cannot leave the hospital. Most health care facilities have documents that the client is asked to sign relating to the client's respinsibilities when the client leaves against medical advice. The client should be asked to sign this document before leaving. The nuse should request that the client wait to speak to the physician before leaving, but if the client refuses to do so the nurse cannot hold the client against the clients will.

Don't pick cough over tachycardia for signs of chf in an infant.

...

1. How does palliative surgery differ from any other type of surgery? "The main purpose is cosmetic in nature rather than functional repair or comfort. B. There are fewer risks associated with palliative surgery than with any other type of surgery. C. The outcomes of palliative surgery cannot be ensured to produce the desired effect or restoration of functional ability. D. Palliative surgery is performed to provide temporary relief of distressing symptoms rather than to cure a problem or condition."

"ANS: D The purpose of palliative surgery is to improve the client's quality of life by reducing or eliminating distressing symptoms. It does not cure a health problem and, often, does not prolong life. Although the exact outcomes of palliative surgery cannot be ensured, neither can the outcomes of any other type of surgery."

A home health nurse visits a client at home and determine that the client is dependent on drugs. Whcih of the following assesment questions would assist the nurse to provide appropriate nursing care? 1. ""Why did you get started on these drugs?"" 2. ""How much do you use and what effect does it have on you?"" 3. ""How long did you think you could take these drugs without someone finding out?"" 4. The nurse does not ask anyquestions for fear that the client is in denial and will throw the nurse out of the home."

"Answer two is the correct answer. Whenever the nurse carries out an assesement for a client who is dependent on drugs it is best for the nurse to attempt to elicity information by being nonjudginmental and direct. Option I is incorrect because it is judgemental. Option 3 is incorrect because it is judgemental which is non therapeutic. Option four is incorrect because it indicates pssivity on the nurse's part and uyses rationalization to avoid the therapeutic invervention."

"The physician has ordered preoperative atropine for a client undergoing surgery. Which of the following provides the best rationale for the use of this medication? "A. It induces general calmness and sleepiness B. It reduces oral and pulmonary secretions and prevents laryngeospasms C. It is used to reduce anxiety and ease anesthetic induction. D. It reduces gastric fluid volume and gastric acidity."

"B. Anticholinergics, such as atropine, scopolamine, and glycopyrrolate (Robinul), reduce oral and pulmonary secretions and prevent laryngospasm."

While performing preoperative teaching, the patient asks when she needs to stop drinking water before the surgery. Based on the most recent practice guidelines established by the American Society of Anesthesiologists, the nurse tells the patient that: "A. She must be NPO after breakfast. B. She needs to be NPO after midnight. C. She can drink clear liquids up to 2 hours before surgery. D. She can drink clear liquids up until she is taken to the OR.

"C. She can drink clear liquids up to 2 hours before surgery. Practice guidelines for preoperative fasting state the minimum fasting period for clear liquids is 2 hours. Evidenced-based practice no longer supports the long-standing practice of requiring patients to be NPO after midnight."

During sickle cell crisis there are two interventions to prioritize: fluids and pain relief.

...

"A client with a perforated gastric ulcer is scheduled for surgery. The client cannot sign the operative consent form because of sedation from opiod analgesics that have been administered. The nurse should take which appropriate action in the care of this client? "1.Obtain a court order for the surgery 2.Send the client to surgery without the consent form being signed. 3.Have the hospital chaplain sign the consent form. 4.Obtain a telephone consent from a family member, following agency policy"

"Correct Answer: 4 Rationale: Every effort should be made to obtain consent from a responsible family member to perform the surgery if a client is unable to sign the consent form."

"When preparing a client for surgery, which intervention should the nurse implement first? "1.Check the permit for the spouse's signature. 2.Take and document intake and output. 3.Administer the "on call" sedative. 4.Complete the preoperative checklist."

"Correct answer: 4 Rationale: 1. The client's signature, not the spouse's, shouldbe on the surgical permit. 2.This would be information that would be im-portant if abnormal, but it is not the first inter- vention. 3."On call" sedations should be administeredafter the surgical checklist is completed. 4.Completing the preoperative checklist hasthe highest priority to ensure that all detailsare completed without omissions"

A priority nursing intervention to assist a preoperative patient in coping with fear of pain would be to: "a) Inform the patient that pain medication will be available b) teach the patient to use guided imagery to help manage pain. c) Describe the type of pain expected with the patient's partcular surgery. d) Explain the pain management plan, including the use of a pain intensity scale."

"Correct answer: d Rationale: If a patient has fear of pain and discomfort during and after surgery, the nurse should reassure the patient that drugs are available for anesthesia and analgesia during surgery. The nurse should teach the patient to ask for medications after surgery when pain is present and assure him or her that taking these medications will not contribute to an addiction. Instruct the patient on the use of some form of pain intensity scale (e.g., 0-10, FACES) and to request pain medication before the pain becomes severe"

"To prevent airway obstruction in the postoperative patient who is unconscious or semiconscious, the nurse: "A. encourages deep breathing. B. elevates the head of the bed. C. administers oxygen per mask. D. positions the patient in a side-lying position.

"D. ""positions the patient in a side-lying position."" is the correct answer. An unconscious or semiconscious patient should be placed in a lateral position to protect the airway from obstruction by the tongue. Deep breathing and elevation of the head of the bed are implemented to facilitate gas exchange when the patient is responsive. Oxygen administration is often used, but the patient must first have a patent airway."

A patient is to recieve ranitidine (Zantac) preoperatively. He tells the nurse that he took his esomeprazole (Nexium) today (as ordered). The nurse explains taht these medications will. a.) calm the patient and relieve his anxiety b.) provide sedation and amnesia before surgery c.) prevent aspiration of stomach contents into his lungs d.) work to decrease stomach acids and help avoid nausea after surgery

"D.) work to decrease stomach acids and help avoid nausea after surgery Rationale: Ranitidine (Zantac) is a histamine (H2) receptor antagonist, and esomeprazole (Nexium) is a proton pump inhibitor. These drugs decrease stomach acid and postoperative nausea."

During the acute stage of Hep-A gown and gloves are required. In the convalescent stage it is no longer contagious.

...

The diabetic educator is teaching a patient about type 2 diabetes. The educator recognizes the patient understands the primary treatment for type 2 diabetes when the patient states what?

"I will follow the weight loss plan designed by the dietitian."

Assertive communication begins with:

"I" rather than "you" and clearly states the problem

A 55 y/o patient comes to the clinic for a routine check-up. The patient's blood pressure is 160/100 and the physician diagnoses hypertension. The patient asks why it is important to treat hypertension. What would be the nurses best response?

"In people over the age of 50, hypertension increases the risk of death, stroke, and heart failure."

Which statement best indicates that the nurse understands the meaning of pharmacokinetics?

"It involves the study of physiologic interactions of drugs. "It explains the distribution of the drug between various body compartments. Pharmacokinetics involves the study of how the drug moves through the body, including absorption, distribution, metabolism, and excretion.

Extra insulin may be needed for a patient taking Prednisone (remember, steroids cause increased glucose).

...

Feed upright to avoid otitis media.

...

"While witnessing a preoperative consent, the nurse learns that the client does not understand the risks of the surgery. The nurse's best action is to: "Notify the surgeon Notify the surgical unit Notify the anesthetist Notify the client's family"

"Notify the surgeon Rationale: The primary responsibility for informed consent lies with the attending surgeon, not the interdisciplinary team."

A patient with chronic pancreatitis had a pancreaticojejunostomy created 3mo ago for relief of pain and to restore drainage of pancreatic secretions. The patient has come to the office for a routine post surgical appointment. The patient is frustrated that the pain has not decreased. What is the most appropriate initial response by the nurse?

"Pain relief occurs by 6 months in more that 85% of the patients who undergo this procedure, and it has been 2 months since your surgery."

"A pre-operative nurse prepares a client for surgery, which nursing interventions should be included in the plan of care? Mark all that apply." "a. Maintain NPO status to prevent aspiration. b. Verify the client's signature on the consent prior to surgery. c. Remove dentures and contact lenses prior to surgery. d. Check the client's allergy and blood bands for accuracy. e. Verify the client's mobility in all extremities prior to surgery."

"Rationale: Maintaining nothing by mouth prevents the client from aspirating food particles into the lungs during and after surgery. Because of legal requirements, the surgical consent must be signed prior to surgery to verify the client's acknowledgement of the content on the consent. Dentures, hairpins, glasses, and contacts may interfere with client safety, or compromise the sterile field. Allergy, blood, and identification bands should all be checked prior to surgery to prevent medication errors, blood bank errors, and to facilitate proper identification of the client. Verifying the client's mobility in all extremities prior to surgery is part of the physical assessment, but not a necessary action prior to surgery. Nursing Process: Intervention Client Need: Management of Care Cognitive Level: Application Objective: Provide appropriate nursing care for the client in the preoperative, intraoperative, and postoperative phases of surgery. Strategy: Apply knowledge of the nursing process to the clinical scenario to select the correct interventions."

First sign of PE is sudden chest pain, followed by dyspnea and tachypnea.

...

The nurse must obtain surgical consent forms for the following clients who are sched-uled for surgery. Which client would not be able to consent to surgery? "1.The 65-year-old client who cannot read or write.2.The 30-year-old client who does not understand English.3.The 16-year-old client who has a fractured ankle. 4.The 80-year-old client who is not oriented to the day."

"The 65-year-old client who cannot read canmark an "X" on the form and is legally able tosign a surgical permit as long as the clientunderstands the benefits, alternatives, and allpotential complications of the surgery.2.The client who does not speak English can andshould have information given and questionsanswered in the client's native language. 3.A 16-year-old client is not legally able togive permission for surgery unless theadolescent is given an emancipated statusby a judge. This information was not given in the stem. 4.A client is able to give permission unless deter-mined incompetent. Not knowing the day of the week is not significant"

The physician has explained to the patient that he has developed diabetic neuropathy in his right foot. Later that day the patient calls the nurse and asks what causes diabetic neuropathy. What would be the nurse's best response?

"The cause is not known for sure but it is thought to involve elevated blood glucose levels over a period of years."

"When completing the assessment for the client in the day surgery unit, the client states,"I am really afraid of having this surgery. I'm afraid of what they will find." Which state-ment would be the best therapeutic response by the nurse? "1."Don't worry about your surgery. It is safe." 2."Tell me why you're worried about your surgery." 3."Tell me about your fears of having this surgery." 4."I understand how you feel. Surgery is frightening.""

"The correct answer is 3. 1.This statement is giving false reassurance. 2.This statement is requesting an explanation. 3. This statement focuses on the emotion that the client identified and is therapeutic. 4.This statement belittles the client's fear, andno person understands how another personfeels"

A patient with renal insufficiency has been hospitalized on your unit. The patient knows that renal function depends on the functional status of nephrons. The patient asks you when she will need to start dialysis based upon loss of nephron function. What would your respond?

"When about 80% of the nephrons are no longer functioning."

Fluid volume overload caused by IVC fluids infusing too quickly (or whatever reason) and CHF can cause an S3

...

"A patient who is dependent on barbiturates is scheduled for surgery following an automobile accident. The nurse recognizes that this patient "a. may need less pain medication during the postoperative period. b. should be provided with taper doses of barbiturates following surgery. c. may have an immediate onset of withdrawal symptoms when given anesthetic and analgesic agents. d. has a low risk for physical withdrawal symptoms but is likely to experience craving and drug-seeking behavior during the post-operative period."

"answer: b Rationale: withdrawal from sedative hypnotics can be very serious.After 24 hours, the patient may experience delirium, seizures, and respiratory and cardiac arrest, and withdrawal from high doses requires close monitoring in an inpatient setting. Long-acting agents such as diazepam (Valium), chlordiazepoxide (Librium), clonazepam (Klonopin), or phenobarbital may be substituted for the abused drug and gradually tapered after stabilization. Mild to moderate symptoms can persist for 2 to 3 weeks after a 3- to 5-day period of acute symptoms."

Chp. 1 You practice using nursing's code of ethics for professional registered nurses. This code: 1. Improves self-health care 2. Protects the client from harm 3. Ensures identical care to all clients 4. Defines the principles by which nurses provide care to their clients

#4

Hematocrit - definition

% of blood occupied by RBC.

after task is completed, check what

&Was task done properly?* If not, provide teaching. *Was the task done in the proper timeframe?* Will the delay affect client safety? *Were the client's needs met?* Did the task change and require higher level of education? Maybe you should do the task!

Autonomic dysreflexia

( life threatening inhibited sympathetic response of nervous system to a noxious stimulus- patients with spinal cord injuries at T-7 or above) is usually caused by a full bladder.

dilantin level

(10-20). Can cause gingival hyperplasia

What is the parkland formula for fluid resuscitation post burn?

(4 ml of LR X TBSA% X weight in Kg); intended for 24 hours; half is given over first 8 hours and remainder is given over the next 16 hours

Pulmonary Embolism

(S&S: chest pain, difficulty breathing, tachycardia, pale/cyanotic, sense of impending doom) --> turn pt to left side and lower the head of the bed.

Autonomic Dysreflexia/Hyperreflexia

(S&S: pounding headache, profuse sweating, nasal congestion, goose flesh, bradycardia, hypertension) --> place client in sitting position (elevate HOB) first before any other implementation.

Mevacor

(anticholesterol med) must be given with evening meal if it is QD (per day).

Thorazine, haldol

(antipsychotic) can lead to EPS (extrapyramidal side effects)

Tertiary Intention

(delayed primary intention) healing occurs with delayed suturing of a wound in which two layers of granulation tissue are sutured together This occurs when a contaminated wound is left open and suture closed after the infection is controlled It also occurs when a primary wound becomes infected, is opened is allowed to granulate and is then sutured Tertiary intention usually results in a larger and deeper scar than primary or secondary intention.

Appendicitis

(inflammation of the appendix) pain is in RL quadrant with rebound tenderness.

Diverticulitis

(inflammation of the diverticulum in the colon) pain is around LL quadrant.

Indocin

(nsaid) tx of arthritis (osteo, rhematoid, gouty), bursitis, and tendonitis.

levothyroxine (synthroid)

(nsaid) tx of arthritis (osteo, rhematoid, gouty), bursitis, and tendonitis.

Magnesium sulfate

(used to halt preterm labor) is contraindicated if deep tendon reflexes are ineffective. If patient experiences seizure during magnesium adm. Get the baby out stat (emergency).

What question would the nurse ask a pt prior to an IVP?

***1. Are you allergic to shellfish? 2. do you have burning on urination? 3. have you ever had kidney stones? 4. why are you having this test?

632. The nurse is reviewing the laboratory results in a client with cirrhosis and notes that the ammonia level is elevated. Which diet does the nurse anticipate to be prescribed for this client?

***1. Low-protein diet 2. High-protein diet 3. Moderate-fat diet 4. High-carbohydrate diet

634. The client with hiatal hernia chronically experiences heartburn following meals. The nurse plans to teach the client to avoid which action because it is contraindicated with a hiatal hernia?

***1. Lying recumbent following meals 2. Taking in small, frequent, bland meals 3. Raising the head of bed on 6-inch blocks 4. Taking H2-receptor antagonist medication

629. The nurse is monitoring a client for the early signs and symptoms of dumping syndrome. Which of the following indicate this occurrence?

***1. Sweating and pallor 2. Bradycardia and indigestion 3. Double vision and chest pain 4. Abdominal cramping and pain

A pt hospitalized w/ sever ascites due to cirrhosis develops a fever and confusion. The nurse should do which of the following?

***1. auscultate bowel sounds and palpate for abdominal tenderness 2. Inquire about headache and check for nuchal rigidity 3. observe for neck vein distention and auscultate lung sounds 4. measure abdominal girth and percuss for shifting dullness

Following surgery, a pt has not voided for 12 hours. What assessment should the nurse make?

***1. palpate for bladder distention. 2. ausculatate for bowel sounds. 3. inspect for edema of the urethra 4. percuss for gastric tympany

A 58-year old woman presents at her primary care provider's office w/ symptoms of frequency, urgency, nocturia, dysuria, and cloudy, rust-colored urine for the thrid time in the past 2 years. Th nurse shold plan to include which of the following in her teaching for this patinet? Select all that paply.

***1. return to the office in 10 days for follow-up culture 2. preprocedure insturction for an IVP ***3. the potential benefits of estrogen vaginal cream ***4. recommendations for perineal cleansing 5. recommendations for screening cystoscopy

The physicain has ordered omeprazole 20 mg twice daily, clarithromycin 500 mg twice daily, and amoxicillin 1 g daiy for a pt w/ PUD. It is most important for the nurse to instruct the pt to do which of the following?

***1. stop the drugs immediately and notify the physician if a rash, hives, or itching develop. 2. consume 8 oz of yogurt or buttermilk daily while taking these drugs 3. take drugs on empty stomach, 1 hour before breakfast and at least 2 hours after dinner. 4. take drugs w/ full glass of water

Depression often manifests itself in somatic ways, such as psychomotor retardation, gi complaints, and pain.

...

Digitalis increases ventricular irritability, and could convert a rhythm to v-fib following cardioversion.

...

Do NOT use "WHY" or "I UNDERSTAND" statements with clients.

...

NREM stage 3

15-30 min, early phase of deep sleep, snoring, relaxed muscle tone, little/no physical movement, difficult to arouse

What are the ideal scores on a RASS?

+1, 0, -1 restless, alert and calm, and drowsy respectivley

NREM stage 4

15-30 min, shortens toward morning, deep sleep, sleep-walking, sleep-talking, bed-wetting may occur

full thickness wound repair

- inflammatory (up to 3 days) - proliferative (3-24 days) - remodeling (up to 1 yr.)

partial thickness wound repair

- inflammatory response (24hrs.) - epithelial proliferation/migration - reestablishment of epidermal layers

Diabetic ketoacidosis (DKA)

- when body is breaking down fat instead of sugar for energy. Fats leave ketones (acids) that cause pH to decrease. - rare in diabetes mellitus type II because there is enough insulin to prevent breakdown of fats.

Advantages of Oral, Buccal and Sublingual Routes

--Conveinent and comfortable --Economical --Sometimes produce local or systemic effects --rarely cause anxiety

Disadvantages of Oral, Buccal and Sublingual Routes

--GI irritation

Disadvantages for Skin

--absorption occurs too rapid over abrasions --medications overall absorb slowly through this route

Advantages of Parenteral Routes

--can be used when oral drugs are contraindicated --more rapid absorption --epidural provides excellent pain control

Disadvantages for MM

--highly sensitive --awkward(vaginal and rectal)

Disadvantages of Parenteral Routes

--introducing infection --tissue damage --more expensive --quicker absorption=quicker adverse reactions --more painful

Advantages for MM

--local application provides therapeutic effects --aqueous solutions readily absorbed and capable of causing systemic effects --potential ROA when oral drugs are contraindicated

Advantages for Skin

--local effect --painless --limited side effects

Advantages for Inhalation

--rapid relief

Disadvantages for Inhalation

--serious systemic effects

A nurse is caring for a client after surgery. The client has reported at least once fall in the last 3 months. The client may ambulate with assistance, has a saline lock in place, and has demonstrated appropriate use of the call light to request assesstance. What interventions should the nurse include int he client's plan of care?

-Ensure necessities are within the client's reach. -Identify the client as high risk with the appropriate wristband. -Assess the client for pain and toileting needs.

Neuroleptic malignant syndrome (NMS):

-NMS is like S&M; -you get hot (hyperpyrexia) -stiff (increased muscle tone) -sweaty (diaphoresis) -BP, pulse, and respirations go up & -you start to drool

A nurse who works on an obstetrical inpatient unit has been assigned to the Client Safety Committee. What client safely goals are most applicable to this setting?

-Providing effective and timely hand-off reports between labor and delivery staff and mother-baby staff -Ensuring that health care providers complete pre procedure verification for any invasive procedure -Involving clients in education to prevent cesarean sections, episiotomies, and cord infections -Identifying safely risks specific to the unit, such as infant abduction

During the process of restraining a client, a staff member is injured. The nurse manager would conclude that a peer support program has been helpful for the injured staff member if what outcomes have been achieved?

-The injured staff member has debriefed with other staff involved in the restraint -The injured staff member has had the opportunity to express feelings with a support group -The injured staff member has decided whether to talk to the assaultive client -A plan has been arranged to facilitate the return of the injured staff member to work

Mannitol (osmotic diuretic)

-head injury medication - crystallizes at room temp so ALWAYS use filter needle

Urine: Ketones - definition

-results from the metabolism of fatty acid and fat; acetone is the main substance being tested -healthy person: ketones formed in liver and completely metabolized so only negligible amounts appear in the urine -when carb metabolism is altered, an excessive amt of ketones is formed because fat becomes the predominant body fuel -increased ketones lead to electrolyte imbalance, dehydration, acidosis and eventual coma -presence of ketones in urine is associated w/diabetes or altered carb metabolism

"A nurse is developing a POC for a client scheduled for surgery. The nurse should include which activity in the nursing care plan for the client on the day of surgery? "1. Have the client void immediately before going into surgery 2. Avoid oral hygiene and rinsing with mouthwash. 3. Verify that the client has not eaten for the last 24 hours. 4. Report immediately any slight increase in blood pressure or pulse."

. Have the client void immediately before going into surgery The nurse would assist client to void immediately before surgery so that the bladder will be empty. A slight increase in blood pressure and pulse is common during the preoperative period and usually the result of anxiety. The client usually has a restriction of food and fluids for 6-8 hours before surgery instead of 24.

A laxative is given the night before an IVP in order to better visualize the organs.

...

A newly diagnosed hypertension patient should have BP assessed in both arms

...

A patient with a low hemoglobin and/or hematocrit should be evaluated for signs of bleeding, such as dark stools.

...

A patient with liver cirrhosis and edema may ambulate, then sit with legs elevated to try to mobilize the edema.

...

A positive Western blot in a child <18 months (presence of HIV antibodies) indicates only that the mother is infected. Two or more positive p24 antigen tests will confirm HIV in kids <18 months. The p24 can be used at any age.

...

A typical adverse reaction to oral hypoglycemics is rash, photosensitivity.

...

Absence of menstruation leads to osteoporosis in the anorexic.

...

Adenosine is the treatment of choice for paroxysmal atrial tachycardia.

...

After a hydrocele repair provide ice bags and scrotal support.

...

After g-tube placement the stomach contents are drained by gravity for 24 hours before it can be used for feedings.

...

After pain relief, cough and deep breathe is important in pancreatitis, because of fluid pushing up in the diaphragm.

...

After removal of the pituitary gland you must watch for hypocortisolism and temporary diabetes insipidus.

...

Ambient air (room air) contains 21% oxygen.

...

Amniotic fluid is alkaline, and turns nitrazine paper blue. Urine and normal vaginal discharge are acidic, and turn it pink.

...

An NG tube can be irrigated with cola, and should be taught to family when a client is going home with an NG tube.

...

An antacid should be given to a mechanically ventilated patient w/ an ng tube if the ph of the aspirate is <5.0. Aspirate should be checked at least every 12 hrs.

...

Atropine blocks acetylcholine (remember it reduces secretions).

...

BSA is considered the most accurate method for medication dosing with kids. (I though it was weight, but apparently not)

...

Burning sensation in the mouth, and brassy taste are adverse reactions to Lugol solution (for 34 hyperthyroid). Report it to the doc.

...

Can't cough=ineffective airway clearance

...

Cephalhematoma (caput succinidanium) resolves on its own in a few days. This is the type of edema that crosses the suture lines.

...

Change in color is always a LATE sign!

...

Coarctation of the aorta causes increased blood flow and bounding pulses in the arms

...

Crackles suggest pneumonia, which is likely to be accompanied by hypoxia, which would manifest itself as mental confusion, etc.

...

Dantrium, for spasticity, may take a week or more to be effective.

...

Decorticate positioning in response to pain = Cortex involvement. Decerebrate in response to pain = Cerebellar, brain stem involvement

...

Decreased acetylcholine is related to senile dementia.

...

Look carefully when you have no idea. In a word like rhabdomyosarcoma you can easily ascertain 32 it has something to do with muscle (myo) cancer (sarcoma). The same thing goes for drug names. For example, if it ends in -ide it's probably a diuretic, as in Furosemide, and Amyloride.

...

Low magnesium and high creatinine signal renal failure.

...

MMR and Varicella immunizations come later (15 months).

...

Managing stress in a patient with adrenal insufficiency (Addison's) is paramount, because if the adrenal glands are stressed further it could result in Addisonian crisis. While we're on Addison's, remember blood pressure is the most important ssessment parameter, as it causes severe hypotension.

...

NEVER check the monitor or a machine as a FIRST ACTION. ALWAYS assess the CLIENT FIRST!

...

Never give potassium if the patient is oliguric or anuric.

...

No aspirin with kids b/c it is associated with Reye's Syndrome, and also no nsaids such as ibuprofen. Give Tylenol.

...

No milk (as well as fresh fruit or veggies) on neutropenic precautions.

...

No phenylalanine with a kid positive for PKU (no meat, no dairy, no aspartame).

...

Nonfat milk reduces reflux by increasing lower esophageal sphincter pressure

...

Normal PCWP (pulm capillary wedge pressure) is 8-13. Readings of 18-20 are considered high.

...

One medication that cannot be administered by intraosseous infusion is isoproterenol, a beta agonist.

...

Other than initially to test tolerance, G-tube and J-tube feedings are usually given as continuous feedings.

...

Pain is usually the highest priority with RA

...

Patients with GERD should lay on their left side with the HOB elevated 30 degrees.

...

Place a wheelchair parallel to the bed on the side of weakness

...

Pneumovax 23 gets administered post splenectomy to prevent pneumococcal sepsis.

...

Position prone w hob elevated with gerd. In almost every other case, though, you better lay that kid on his back (Back To Sleep - SIDS).

...

Prolonged hypoxemia is a likely cause of cardiac arrest in a child.

...

Pull pinna down and back for kids < 3 yrs. when instilling eardrops.

...

Pulmonary sarcoidosis leads to right sided heart failure.

...

Remember the action of vasopressin because it sounds like "press in", or vasoconstrict.

...

Remember the phrase "step up" when picturing a person going up stairs with crutches. The good leg goes up first, followed by the crutches and the bad leg. The opposite happens going down. The crutches go first, followed by the good leg.

...

Respiratory problems are the chief concern with CF

...

Safety over Nutrition with a severely depressed client.

...

School-age kids (5 and up) are old enough, and should have an explanation of what will happen a week before surgery such as tonsillectomy.

...

Second voided urine most accurate when testing for ketones and glucose.

...

Sengstaken blakemore tube used for tx of esophageal varices, keep scissors at bedside.

...

Sepsis and anaphylaxis (along with the obvious hemorrhaging) reduce circulating volume by way of increased capillary permeability, which leads to reduced preload (volume in the left ventricle at the end of diastole). This is a toughie...think about it.

...

Serum acetone and serum ketones rise in DKA. As you treat the acidosis and dehydration expect the potassium to drop rapidly, so be ready, with potassium replacement.

...

TB drugs are liver toxic. (Does your patient have hepB?) An adverse reaction is peripheral neuropathy.

...

THE NURSE IS PREPARING TO ADMINISTER A MEDICATION VIA NG TUBE. WHAT GUIDELINE IS APPROPRIATE FOR THE NURSE TO FOLLOW WHEN ADMINISTERING A DRUG VIA THIS ROUTE? FLUSH THE TUBE WITH WATER BETWEEN EACH MED

...

Tensilon test is given if muscles are tense in myasthenia gravis.

...

The biggest concern with cold stress and the newborn is respiratory distress.

...

The first sign of ARDS is increased respirations. Later comes dyspnea, retractions, air hunger, cyanosis.

...

Theophylline causes GI upset, give with food

...

Toddlers need to express autonomy (independence)

...

Too much milk reduces intake of other essential nutrients, especially iron. Watch for anemia with milk-aholics. And don't let that mother put anything but water in that kid's bottle during naps/over-night. Juice or milk will rott that kids teeth right out of his head.

...

Unusual positional tip - Low-fowlers recommended during meals to prevent dumping syndrome. Limit fluids while eating.

...

Water intoxication will be evidenced by drowsiness and altered mental status in a patient with TUR syndrome, or as an adverse reaction to desmopressin (for diabetes insipidus).

...

While treating DKA, bringing the glucose down too far and too fast can result in increased intracranial pressure d/t water being pulled into the CSF. Polyuria is common with the hypercalcemia caused by hyperparathyroidism.

...

When action is taken on one's prejudices: 1. Discrimination occurs 2. Delivery of culturally congruent care is ensured 3. Effective intercultural communication develops 4. Sufficient comparative knowledge of diverse groups is obtained

1

Which action is initially taken by the nurse to verify correct position of a newly placed small-bore feeding tube? 1. place an order for x-ray examination to check position 2. confirm the distal mark on the feeding tube after taping 3. Test the pH of the gastric contents, and observe the color 4. Auscultate over the gastric area as air is injected into the tube

1

How much volume does CSF account for in the intracranial volume?

150 ml

Platelet Count - Lab Values: Newborn

150-300

Incentive Spirometry

1) Sit upright 2) Exhale 3) Insert mouthpiece 4) Inhale for 3 seconds, and then HOLD for 10 seconds

Birth weight triples by:

1 year of age

A client uses a metered-dose inhaler (MDI) to aid in management of his asthma. Which action by the client indicates to the nurse that he needs further instruction regarding its use? Select all that apply. 1. Activation of the MDI is not coordinated with inspiration. 2. The client inspires rapidly when using the MDI. 3. The client holds his breath for 3 seconds after inhaling with the MDI. 4. The client shakes the MDI after use. 5. The client performs puffs in rapid succession.

1, 2, 3, 4, 5. Utilization of an MDI requires coordination between activation and inspiration; deep breaths to ensure that medication is distributed into the lungs, holding the breath for 10 seconds or as long as possible to disperse the medication into the lungs, shaking up the medication in the MDI before use, and a sufficient amount of time between puffs to provide an adequate amount of inhalation medication.

Platelet Count - Lab Values: Adult

150-400

You are caring for a client experiencing dysphagia. Which interventions will help decrease the risk of aspiration during feeding? (choose all that apply) 1. sit the client upright in a chair 2. give liquids at the end of the meal 3. place food in the strong side of the mouth 4. provide thin foods to make it easier to swallow 5. feed the client slowly, allowing time to chew and swallow 6. encourage client to lie down to rest for 30 min. after eating

1, 3, 5

Which of the following is significant data to gather from a client who has been diagnosed with pneumonia? Select all that apply. 1. Quality of breath sounds. 2. Presence of bowel sounds. 3. Occurence of chest pain. 4. Amount of peripheral edema. 5. Color of nail beds.

1, 3, 5. A respiratory assessment, which includes auscultating breath sounds and assessing the color of the nail beds, is a priority for clients with pneumonia. Assessing for the presence of chest pain is also an important respiratory assessment as chest pain can interfere with the client's ability to breathe deeply. Auscultating bowel sounds and assessing for peripheral edema may be appropriate assessments, but these are not priority assessments for the client with pneumonia.

A client is taking clonidine (Catapres) for treatment of hypertension. The nurse should teach the client about which of the following common adverse effects of this drug? Select all that apply. 1. Dry mouth. 2. Hyperkalemia. 3. Impotence. 4. Pancreatitis. 5. Sleep disturbance.

1, 3, 5. Clonidine (Catapres) is a central-acting adrenergic antagonist. It reduces sympathetic outflow from the central nervous system. Dry mouth, impotence, and sleep disturbances are possible adverse effects. Hyperkalemia and pancreatitis are not anticipated with use of this drug.

The nurse administers two 325 mg aspirin every 4 hours to a client with pneumonia. The nurse should evaluate the outcome of administering the drug by assessing which of the following? Select all that apply. 1. Decreased pain when breathing. 2. Prolonged clotting time. 3. Decreased temperature. 4. Decreased respiratory rate. 5. Increased ability to expectorate secretions.

1, 3. Aspirin is administered to clients with pneumonia because it is an analgesic that helps control chest discomfort and an antipyretic that helps reduce fever. Aspirin has an anticoagulant effect, but that is not the reason for prescribing it for a client with pneumonia, and the use of the drug will be short term. Aspirin does not affect the respiratory rate, and does not facilitate expectoration of secretions.

The nurse is teaching the client how to use a metered-dose inhaler (MDI) to administer a corticosteroid. Which of the following client actions indicates that he is using the MDI correctly? Select all that apply. 1. The inhaler is held upright. 2. The head is tilted down while inhaling the medicine. 3. The client waits 5 minutes between puffs. 4. The mouth is rinsed with water following administration. 5. The client lies supine for 15 minutes following administration.

1, 4. The client should shake the inhaler and hold it upright when administering the drug. The head should be tilted back slightly. The client should wait about 1 to 2 minutes between puffs. The mouth should be rinsed following the use of a corticosteroid MDI to decrease the likelihood of developing an oral infection. The client does not need to lie supine; instead, the client will likely to be able to breathe more freely if sitting upright.

Chp. 39..Hygienic care requires close contact with the client. The nurse initially uses which of the following to promote a caring therapeutic relationship? 1. assessment skills 2. therapeutic touch 3. fundamental skills 4. communication skills

1,3,4

Bilirubin - Lab Values: Newborn

1-12

Thyroxine (T-4) - Lab Values: Newborn

1-3 days: 11-22 1-2 weeks: 8-16

how does lochia change during the first 10 post partum days?

1-3 rubra (red), 4-9 serosa (pink-brown), 10+ alba (yellow-white)

When does regular insulin peak?

1-5 hours

Triiodothyronine (T-3) Lab Values: Child

1-5: 105-270 6-10: 95-240 10-15: 80-215 16-20: 80-210

Thyroxine (T-4) - Lab Values: Child

1-5: 7-15 5-10: 6-13 10-15: 5-12

Low Density Lipoprotien - Lab Values: Child

1-9 93-100 10-14 97 15-19 94-96

Which of the following findings would most likely indicate the presence of a respiratory infection in a client with asthma? 1. Cough productive of yellow sputum. 2. Bilateral expiratory wheezing. 3. Chest tightness. 4. Respiratory rate of 30 breaths/ minute.

1. A cough productive of yellow sputum is the most likely indicator of a respiratory infection. The other signs and symptoms- wheezing, chest tightness, and increased respiratory rate- are all findings associated with an asthma attack and do not necessarily mean an infection is present.

A client is prescribed metaproterenol (Alupent) via a metered-dose inhaler, two puffs every 4 hours. The nurse instructs the client to report adverse effects. Which of the following are potential adverse effects of metaproterenol? 1. Irregular heartbeat. 2. Constipation. 3. Pedal edema. 4. Decreased pulse rate.

1. Irregular heartbeats should be reported promptly to the care provider. Metaproterenol (Alupent) may cause irregular heartbeat, tachycardia, or anginal pain because of its adrenergic effect on beta-adrenergic receptors in the heart. It is not recommended for use in clients with known cardiac disorders. Metaproterenol does not cause constipation, pedal edema, or bradycardia.

When developing a discharge plan to manage the care of a client with chronic obstructive pulmonary disease (COPD), the nurse should advise the the client to expect to: 1. Develop respiratory infections easily. 2. Maintain current status. 3. Require less supplemental oxygen. 4. Show permanent improvement.

1. A client with COPD is at high risk for development of respiratory infections. COPD is slowly progressive; therefore, maintaining current status and establishing a goal that the client will require less supplemental oxygen are unrealistic expectations. Treatment may slow progression of the disease, but permanent improvement is highly unlikely.

Platelet Count - Lab Values: Child

150-400

1 lb:

16 oz

Which of the following is a priority goal for the client with chronic obstructive pulmonary disease (COPD)? 1. Maintaining functional ability. 2. Minimizing chest pain. 3. Increasing carbon dioxide levels in the blood. 4. Treating infectious agents.

1. A priority goal for the client with COPD is to manage the signs and symptoms of the disease process so as to maintain the client's functional ability. Chest pain is not a typical symptom of COPD. The carbon dioxide concentration in the blood is increased to an abnormal level in clients with COPD; it would not be a goal to increase the level further. Preventing infection would be a goal of care for the client with COPD.

An industrial health nurse at a large printing plant finds a male employee's blood pressure to be elevated on two occasions 1 month apart and refers him to his private physician. The employee is about 25 lb overweight and has smoked a pack of cigarettes daily for more than 20 years. The client's physician prescribes atenolol (Tenormin) for the hypertension. The nurse should instruct the client to: 1. Avoid sudden discontinuation of the drug. 2. Monitor the blood pressure annually. 3. Follow a 2-g sodium diet. 4. Discontinue the medication if severe headaches develop.

1. Atenolol is a beta-adrenergic antagonist indicated for management of hypertension. Sudden discontinuation of this drug is dangerous because it may exacerbate symptoms. The medication should not be discontinued without a physician's order. Blood pressure needs to be monitored more frequently than annually in a client who is newly diagnosed and treated for hypertension. Clients are not usually placed on a 2-g sodium diet for hypertension.

The client with pneumonia develops mild constipation, and the nurse administers docusate sodium (Colace) as ordered. This drug works by: 1. Softening the stool. 2. Lubricating the stool. 3. Increasing stool bulk. 4. Stimulating peristalsis.

1. Docusate sodium (Colace) is a stool softener that allows fluid and fatty substances to enter the stool and soften it. Docusate sodium does not lubricate the stool, increase stool bulk, or stimulate peristalsis.

A client with deep vein thrombosis suddenly develops dyspnea, tachypnea, and chest discomfort. What should the nurse do first? 1. Elevate the head of the bed 30 to 45 degrees. 2. Encourage the client to cough and deep breathe. 3. Auscultate the lungs to detect abnormal breath sounds. 4. Contact the physician.

1. Elevating the head of the bed facilitates breathing because the lungs are able to expand as the diaphragm descends. Coughing and deep breathing do not alleviate the symptoms of a pulmonary embolus, nor does lung auscultation. The physician must be kept informed of changes in a client's status, but the priority in this case is alleviating the symptoms.

Bed rest is prescribed for a client with pneumonia during the acute phase of the illness. The nurse should determine the effectiveness of bed rest by assessing the client's: 1. Decreased cellular demand for oxygen. 2. Reduced episodes of coughing. 3. Diminished pain when breathing deeply. 4. Ability to expectorate secretions more easily.

1. Exudate in the alveoli interferes with ventilation and the diffusion of gases in clients with pneumonia. During the acute phase of the illness, it is essential to reduce the body's need for oxygen at the cellular level; bed rest is the most effective method for doing so. Bed rest does not decrease coughing or promote clearance of secretions, and it does not reduce pain when taking deep breaths.

The nurse is developing a care plan with an older adult and is instructing the client that hypertension can be a silent killer. The nurse should instruct the client to be aware of signs and symptoms of other system failures and encourage the client to report signs of which of the following diseases that are often a result of undeteced high blood pressure? 1. Cerebrovascular accidents (CVAs). 2. Liver disease. 3. Myocardial infarction. 4. Pulmonary disease.

1. Hypertension is referred to as the silent killer for adults, because until the adult has significant damage to other systems, the hypertension may go undetected. CVAs can be related to long-term hypertension. Liver or pulmonary disease is not generally associated with hypertension. Myocardial infarction is generally related to coronary artery disease.

Which of the following physical assessment findings are normal for a client with advanced chronic obstructive pulmonary disease (COPD)? 1. Increased anteroposterior chest diameter. 2. Underdeveloped neck muscles. 3. Collapsed neck veins. 4. Increased chest excursions with respiration.

1. Increased anteroposterior chest diameter is characteristic of advanced COPD. Air is trapped in the overextended alveoli, and the ribs are fixed in an inspiratory position. The result is the typical barrel-chested appearance. Overly developed, not underdeveloped, neck muscles are associated with COPD because of their increased use in the work of breathing. Distended, not collapsed, neck veins are associated with COPD as a symptom of the heart failure that the client may experience secondary to the increased workload on the heart to pump blood into the pulmonary vasculature. Diminished, not increased, chest excursion is associated with COPD.

Which of the following health promotion activities should the nurse include in the discharge teaching plan for a client with asthma? 1. Incorporate physical exercise as tolerated into the daily routine. 2. Monitor peak flow numbers after meals and at bedtime. 3. Eliminate stressors in the work and home environment. 4. Use sedatives to ensure uninterrupted sleep at night.

1. Physical exercise is beneficial and should be incorporated as tolerated into the client's schedule. Peak flow numbers should be monitored daily, usually in the morning (before taking medication). Peak flow does not need to be monitored after each meal. Stressors in the client's life should be modified but cannot be totally eliminated. Although adequate sleep is important, it is not recommended that sedatives be routinely taken to induce sleep.

When teaching a client about propranolol hydrochloride, the nurse should base the information on the knowledge that propranolol: 1. Blocks beta-adrenergic stimulation and thus causes decreased heart rate, myocardial contractility, and conduction. 2. Increases norepinephrine secretion and thus decreases blood pressure and heart rate. 3. Is a potent arterial and venous vasodilator that reduces peripheral vascular resistance and lowers blood pressure. 4. Is an angiotensin-converting enzyme inhibitor that reduces blood pressure by blocking the conversion of angiotensin I to angiotensin II.

1. Propranolol is a beta-adrenergic blocking agent. Actions of propranolol include reducing heart rate, decreasing myocardial contractility, and slowing conduction. Propranolol does not increase norepinephrine secretion, cause vasodilation, or block conversion of angiotensin I to angiotensin II.

HCO3 - Lab Values: Newborn

16-24

The client has had hypertension for 20 years. The nurse should assess the client for? 1. Renal insufficiency and failure. 2. Valvular heart disease. 3. Endocarditis. 4. Peptic ulcer disease.

1. Renal disease, including renal insufficiency and failure, is a complication of hypertension. Effective treatment of hypertension assists in preventing this complication. Valvular heart disease, endocarditis, and peptic ulcer disease are not complications of hypertension.

A client experiencing a severe asthma attack has the following arterial blood gas: pH 7.33; Pco2 48; Po2 58; HCO3 26. Which of the following orders should the nurse perform first? 1. Albuterol (Proventil) nebulizer. 2. Chest x-ray. 3. Ipratropium (Atrovent) inhaler. 4. Sputum culture.

1. The arterial blood gas reveals a respiratory acidosis with hypoxia. A quick-acting bronchodilator, albuterol, should be administered via nebulizer to improve gas exchange. Ipratropium is a maintenance treatment for bronchospasm that can be used with albuterol. A chest x-ray and sputum sample can be obtained once the client is stable.

A 79-year-old female client is admitted to the hospital with a diagnosis of bacterial pneumonia. While obtaining the client's health history, the nurse learns that the client has osteoarthritis, follows a vegetarian diet, and is very concerned with cleanliness. Which of the following would most likely be a predisposing factor for the diagnosis of pneumonia? 1. Age. 2. Osteoarthritis. 3. Vegetarian diet. 4. Daily bathing.

1. The client's age is a predisposing factor for pneumonia; pneumonia is more common in elderly or debilitated clients. Other predisposing factors include smoking, upper respiratory tract infections, malnutrition, immunosuppression, and the presence of a chronic illness. Osteoarthritis, a nutritionally sound vegetarian diet, and frequent bathing are not predisposing factors for pneumonia.

Metoprolol (Toprol XL) is added to the pharmacologic therapy of a diabetic female diagnosed with stage 2 hypertension initially treated with Furosemide (Lasix) and Ramipril (Altace). An expected therapeutic effect is: 1. Decrease in heart rate. 2. Lessening of fatigue. 3. Improvement in blood sugar levels. 4. Increase in urine output.

1. The effect of a beta blocker is a decrease in heart rate, contractility, and afterload, which leads to a decrease in blood pressure. The client at first may have an increase in fatigue when starting the beta blocker. The mechanism of action does not improve blood sugar or urine output.

The nurse is planning to teach a client with chronic obstructive pulmonary disease how to cough effectively. Which of the following instructions should be included? 1. Take a deep abdominal breath, bend forward, and cough three or four times on exhalation. 2. Lie flat on the back, splint the thorax, take two deep breaths, and cough. 3. Take several rapid, shallow breaths and then cough forcefully. 4. Assume a side-lying position, extend the arm over the head, and alternate deep breathing with coughing.

1. The goal of effective coughing is to conserve energy, facilitate removal of secretions, and minimize airway collapse. The client should assume a sitting position with feet on the floor if possible. The client should bend forward slightly and, using pursed-lip breathing, exhale. After resuming an upright position, the client should use abdominal breathing to slowly and deeply inhale. After repeating this process three or four times, the client should take a deep abdominal breath, bend forward, and cough three or four times upon exhalation (" huff" cough). Lying flat does not enhance lung expansion; sitting upright promotes full expansion of the thorax. Shallow breathing does not facilitate removal of secretions, and forceful coughing promotes collapse of airways. A side-lying position does not allow for adequate chest expansion to promote deep breathing.

The nurse requests a client to sign the surgical consent form for an emergency appen-dectomy. Which statement by the client indicates that further teaching is needed? 1. "I will be glad when this is over so that I can go home."2."I will not be able to eat or drink anything prior to my surgery."3."I need to practice relaxing by listening to my favorite music."4."I will need to get up and walk as soon as possible."

1. "I will be glad when this is over so that I can go home"1. When recuperating from emergency sur-gery, the client will be in the hospital for afew days. This is not a day-surgery proce-dure. The client needs more teaching. 2.Clients are NPO (nothing by mouth) prior tosurgery to prevent aspiration during and afteranesthesia.3.Listening to music and other relaxing tech-niques can be used to alleviate anxiety and pain.4.Clients are encouraged to get out of bed assoon as possible and progress until a return todaily activity is achieved."

648. A client with a peptic ulcer is diagnosed with a Helicobacter pylori infection. The nurse is teaching the client about the medications prescribed, including clarithromycin (Biaxin), esomeprazole (Nexium), and amoxicillin (Amoxil). Which statement by the client indicates the best understanding of the medication regimen?

1. "My ulcer will heal because these medications will kill the bacteria." 2. "These medications are only taken when I have pain from my ulcer." *** 3. "The medications will kill the bacteria and stop the acid production." 4. "These medications will coat the ulcer and decrease the acid production in my stomach."

Normal amount of urine output/hour?

30-50ml/hr

ICP and Shock have OPPOSITE V/S

ICP--Hypertension, Bradycardia, and Bradypnea SHOCK--Hypotension, Tachycardia, and Tachypnea

The nurse identifies which of the follwing nursing diagnoses as highest priority for the pt admitted with PUD and possible perforation?

1. Acute Pain 2. Ineffective Health maintainence 3. Nausea ***4. Impaired Tissue Integrity: GI

A pt w/ a hx of PUD suddenly begins to complain of severe abd pain. The nurse should do which of the following? Select all that apply.

1. Administer the prescribed PPI. 2. Obtain an order for narcotic analgesic ***3. Withhold oral food and fluids. ***4. Place the pt in Fowler's position ***5. Notify the physician

The nurse is caring for a client with pneumonia who is confused about time & place and has intravenous fluids infusing. Despite the nurse's attempt to reorient the client and then provide distraction, the client has begin to pull at the IV tubing. After increasing the frequency of observation, what interventions should the nurse implement to ensure the client's safety in what order?

1. Assess the client's respiratory status including oxygen saturation 2. Ensure the client does not need toileting or pain medications. 3. Review the client's medications for interactions that may cause or increase confusion. 4. Contact the physician & request an order for soft wrist restraints.

When planning care for pt w/ stomatitis, the n urse identifies which of the following as a priority intervention?

1. Assist to cleanse mouth w/ mouthwash following meals. 2. allow pt to select appealing foods from a menu ***3. provide viscous lidocaine to relieve mouth pain before meals 4. refer to pt to smoking cessation program

626. The nurse is monitoring a client with a diagnosis of peptic ulcer. Which assessment finding would most likely indicate perforation of the ulcer?

1. Bradycardia 2. Numbness in the legs 3. Nausea and vomiting ***4. A rigid, board-like abdomen

620. The nurse is planning to teach the client with gastroesophageal reflux disease about substances that will increase the lower esophageal sphincter pressure. Which item should the nurse include on this list?

1. Coffee 2. Chocolate 3. Fatty foods ***4. Nonfat milk

Ken is an RN caring for Mr. DeRosa. Ken has determined Mr. DeRosa is from Russian heritage. Ken has a Russian neighbor who does not keep his property well maintained. Ken tells Mr. DeRosa, "While you are in the rehabilitation center, you will keep your beside area organized." Ken's statement is an example of: 1. Assimilation 2. Ethnocentrism 3. Personalistic Practice 4. Culture bound syndrome

2.

You are caring for a client with end-stage lung dz theclient wants to go home on o2 and be comfortable. The family wants the client to have a new surgical procedure. You explain the risk and benefits of the surgery to the family and discuss the client's wishes with the family. You are acting as the client's: 1. Educator 2. Advocate 3. Caregiver 4. Case manager

2. advocate

As a nurse, you teach a client who has had surgery to increase which nutrient to help with tissue repair? 1. fat 2. protein 3. vitamin 4. carbohydrate

2. protein

During a mumps outbreak at a local school, a patient, who is a school teacher, is exposed. She has previously been immunized form mumps. What type of immunity does she possess?

Acquired immunity

A 23-year old woman presents to the urgency clinic w/ symptoms of a UTI. The nursing hx reveals that the pt was treated 3 months ago for a UTI. What additional question should the nurse ask?

1. Did you complete your antibiotic prescription for your first UTI? ***2. What form of birth control are you using? 3. does your partner have similar symptoms? 4. How much fluid do you drink each day?

631. The nurse is reviewing the record of a client with a diagnosis of cirrhosis and notes that there is documentation of the presence of asterixis. How should the nurse assess for its presence?

1. Dorsiflex the client's foot. 2. Measure the abdominal girth. ***3. Ask the client to extend the arms. 4. Instruct the client to lean forward.

Compulsion is:

Action

A pt hospitalized w/ cirrhosis, ascites, and mild hepatic encephalopathy suddenly vomits 200 mL of bright red blood. which of the following should the nurse do first?

1. Insert a NG tube ***2. Place in Fowler's position 3. contact the physician 4. check stool for occult blood

At a local health fair, a man remarks to the nurse that his urine occasionally appears pink. He ownders if this is anything to be concerned about. How should the nurse respond?

1. Instruct the man to notify his physician if he develops pain or difficulty voiding. ***2. advise the man to make an appointment to see his physician. 3. instruct the man to track the relationship between urine color and his activities. 4. Tell the man to increase his fluid intake to 2 1/2 to 3 quarts per day.

621. The client has undergone esophagogastroduodenoscopy. The nurse places highest priority on which item as part of the client's care plan?

1. Monitoring the temperature 2. Monitoring complaints of heartburn 3. Giving warm gargles for a sore throat ***4. Assessing for the return of the gag reflex

646. The client who chronically uses nonsteroidal anti-inflammatory drugs (NSAIDs) has been taking misoprostol (Cytotec). The nurse determines that the medication is having the intended therapeutic effect if which of the following is noted?

1. Resolved diarrhea ***2. Relief of epigastric pain 3. Decreased platelet count 4. Decreased white blood cell count

633. The nurse is doing an admission assessment on a client with a history of duodenal ulcer. To determine whether the problem is currently active, the nurse should assess the client for which symptom(s) of duodenal ulcer?

1. Weight loss 2. Nausea and vomiting ***3. Pain relieved by food intake 4. Pain radiating down the right arm

645. The client with a gastric ulcer has a prescription for sucralfate (Carafate), 1 g by mouth four times daily. The nurse schedules the medication for which times?

1. With meals and at bedtime 2. Every 6 hours around the clock 3. One hour after meals and at bedtime ***4. One hour before meals and at bedtime

Recognizing the risk for urolithiasis in the immobilized pt, the nurse apprpriately plans to do which of the following?

1. administer a Ca supplement 2. regularly monitor urine pH 3. maintain an indwelling urinary cath ***4. increase fluid intake to 3000 mL per day

A nurse is caring for a man who has returned to the unit from the recovery room following a TURP. His urinary drainage bad is filled with dark red fluid w/ obvious clots. He is having painful bladder spasms. What would the nurse do first?

1. assess his I&O since surgery 2. administer pain medication in the form of a B&O suppository ***3. report your assessments to his urologist 4. nothing, because these manifestations are espected following a TURP

The nurse caring for a pt scheduled for an abdominal paracentesis instructs the pt to do which of the following?

1. avoid eating or drinking fluid for 6 hrs prior to the procedure 2. scrub the abd w/ antiseptic soap before the procedure ***3. empty the bladder before the procedure 4. report excess flatus following the procedure to the physician

The nurse evaluates his teaching of a pt w/ acute stress gastritis as effective when the pt states that sh will do which of the following?

1. avoid using aspirin or NSAIDs for routine pain reliefe

The enlarging prostate in BPH typically is manifested by assessment of problems w/ which of the following?

1. bowel eleimination ***2. urinary eliminatin 3. peripheral vascular function 4. skin integrity

Before beginning the physical assessment of the renal system, then urse should ask the pt to do which of the following?

1. empty the bladder 2. takd several deep breaths ***3. provide a urine specimen 4. drink several glassess of water

A pt admitted w/ possible kidney stones develops sudden complaints of acute crampy pain on the left side that radiates into the groin. He is nauseated, and vomits clear fluid. On voiding, his urine is pink. The most appropriate response by the nurse is which of the following?

1. obtain a bladder scan to assess for residual urine 2. adminster the rpescribed narcotic analgesic ***3. notify the physician 4. strain all urine

The nurse caring for a pt preparing to under gemodialysis includes which of the following in the plan of care? select all that apply

1. obtain wt and orthostatic vital signs 3. monitor serum creatinine, BUN, and hematocrit levels

What diagnostic tests are used to differentiate BPH from prostate cancer? Slect all that apply.

1. pelvic u/s ***2. digital rectal exam 3. blood chemistry ***4. PSA level 5. sperm count

Which of the following nursing interventions are of highest priority when caring for a pt w/ small bowel obstruction?

1. placing the pt in semi-Fowler's position ***2. maintaining NG sunction 3. keeping strict I&O records 4. administering prescribed analgesics

During a health hx interview, a pt tells the nurse that she has to get up to void several times a night. This finding is documented as which of the following?

1. polyuria ***2. nocturia 3. dysuria 4. hematuria

What diagnostic test can be used to determine GFR as well as glomerular damage?

1. routine urinalysis 2. renal scan ***3. creatinine clearance 4. renal biopsy

describe the growth and development of a toddler:

12-36m, slowed growth, profound activity, curiosity, negativism, builds 2-7 block towers, vocab increases from 10-200 words, fears include separation anxiety, animals and strangers, anterior fontanelle closes aprx 18m, age-appropriate toys include push-pull, low rocking horses, dolls, stuffed animals, riding toys, blocks, finger-paints, engages in parallel play

Cholesterol - Lab Values: child

120-200

MRS.WEE M - multidrug resistant organism R - respiratory infection S - skin infections * W - wound infxn E - enteric infxn - clostridium difficile E - eye infxn - conjunctivitis

Contact precautions

the nurse suspects that the client is receiving PN thru a CVC has an air embolus. What action does the nurse need to take first? 1. Raise the head of the bed 90 degrees 2. Turn the client to left lateral decubitis position 3. Notify the health care provider immediately 4. Have the client perform the Valsalva maneuver

2

Normal Platelets?

130,000-450,000 (150,000-400,000)

Normal Sodium?

133-145

(binds to phosphates, increases Ca, robs the bones...leads to increased Ca resortion from bones and WEAK BONES)

Long term use of amphogel

Sodium - Lab Values: Child

135-145

Sodium - Lab Values: Adult

136-145

Hemoglobin - Lab Values Adult

14-16 g/dl M-14-18 F-12-16

Normal Respirations?

14-20; slightly elevated for the elderly

Hemoglobin - Lab Values Newborn

14-24g/dl

What is HTN?

140-159/90-99 is stage 1 HTN and >160/>100 is stage 2 HTN

Prothrombin Time - Lab Values: Child

11-12.5

Prothrombin Time - Lab Values: Infant

11-12.5

Prothrombin Time - Lab Values: Adults

11-12.5 Pregnant - slightly lower

Normal Hemoglobin?

11.5-15.5

Test child for lead poisoning at:

12 months of age

Urine: Specific Gravity - Lab Values: Newborn

1.001-1.020

Urine: Specific Gravity - Lab Values: adult

1.005-1.030

What is the specific gravity of CSF?

1.007

Specific gravity of urine?

1.015-1.025

Normal Magnesium?

1.3-2.1

In feeling for presence of the carotid pulse no more than

10 seconds should be used

Hemoglobin - Lab Values Child

10-14 g/dl

Acetaminophen

10-20

Dilantin

10-20

Theophylline

10-20

Therapeutic range for Dilantin?

10-20

Urea Nitrogen - Lab Results: adult

10-20

NREM stage 2

10-20 min, can be awakened w/effort, deeper relaxation

Normal BUN? Creatinine?

10-20; 0.6-1.2

Normal amonia level?

10-80

Normal Trigylcerides?

100-200

Iron: Lab Values: Newborn

100-250

Platelet Count - Lab Values: Premature Newborn

100-300

Triiodothyronine (T-3) Lab Values: Newborn

100-740

1 g (gram):

1000 mg

Contemporary nursing requires that the nurse possess knowledge & skills for a variety of professional roles and responsibilites. Which of the following are examples? (choose all that apply) 1. Caregiver 2. Autonomy & Accountability 3. Patient advocate 4. Health promotion

1234

Normal neonate length?

18-22 inches

Kidney glucose threshold:

180

Type of Syringe and Needle for ID

1ml 25-27 g 3/8-5/8"

BURNS

1st Degree - Red and Painful 2nd Degree - Blisters 3rd Degree - No Pain because of blocked and burned nerves

The home care nurse is seeing the following clients. Which client is at greatest risk for experiencing inadequate nutrition? 1. A 55-yrold obese man recently diagnosed with diabetes mellitus 2. A recently widowed 76-yr old woman recovering from a mild stroke 3. A 22-yr-old mother with a 3-yr old toddler had tonsillectomy surgery 4. A 46-yr old man recovering at home following coronary artery bypass surgery

2

1 pint:

2 cups

1 quart:

2 pints

Depending on the client's age and physical condition, the room temp should be maintained betwen: 1. 65-70 F 2. 68-74 F 3. 75-77 F 4. 78-80 F

2) 68-74

The nurse is instructing a client with COPD how to do pursed-lip breathing. In which order should the nurse explain the steps to the client? 1. "Breathe in normally through your nose for 2 counts (while counting to yourself, one, two)." 2. "Relax your neck and shoulder muscles." 3. "Pucker your lips as if you were going to whistle." 4. "Breathe out slowly through pursed lips for 4 counts (while counting to yourself, one, two, three, four)."

2, 1, 3, 4. The nurse should instruct the client to first relax the neck and shoulders and then take several normal breaths. After taking a breath in, the client should pucker the lips, and finally breathe out through pursed lips.

In teaching the hypertensive client to avoid orthostatic hypotension, the nurse should emphasize which of the following instructions? Select all that apply. 1. Plan regular times for taking medications. 2. Arise slowly from bed. 3. Avoid standing still for long periods. 4. Avoid excessive alcohol intake. 5. Avoid hot baths.

2, 3. Changing positions slowly and avoiding long periods of standing may limit the occurrence of orthostatic hypotension. Scheduling regular medication times is important for blood pressure management but this aspect is not related to the development of orthostatic hypotension. Excessive alcohol intake and hot baths are associated with vasodilation.

Normal INR?

2-3 (coumadin)

International Normalized Ratio - Lab Values: Adult

2-3.5

International Normalized Ratio - Lab Values: Child

2-3.5

International Normalized Ratio - Lab Values: Newborn

2-3.5

Average pupil size?

2-5mm

What is the normal range for CVP?

2-6 mmHg but 7 is not an emergency

Which of the following is an expected outcome for an elderly client following treatment for bacterial pneumonia? 1. A respiratory rate of 25 to 30 breaths/ minute. 2. The ability to perform activities of daily living without dyspnea. 3. A maximum loss of 5 to 10 lb of body weight. 4. Chest pain that is minimized by splinting the rib cage.

2. An expected outcome for a client recovering from pneumonia would be the ability to perform activities of daily living without experiencing dyspnea. A respiratory rate of 25 to 30 breaths/ minute indicates the client is experiencing tachypnea, which would not be expected on recovery. A weight loss of 5 to 10 lb is undesirable; the expected outcome would be to maintain normal weight. A client who is recovering from pneumonia should experience decreased or no chest pain.

When performing postural drainage, which of the following factors promotes the movement of secretions from the lower to the upper respiratory tract? 1. Friction between the cilia. 2. Force of gravity. 3. Sweeping motion of cilia. 4. Involuntary muscle contractions.

2. The principle behind using postural drainage is that gravity will help move secretions from smaller to larger airways. Postural drainage is best used after percussion has loosened secretions. Coughing or suctioning is then used to remove secretions. Movement of cilia is not sufficient to move secretions. Muscle contractions do not move secretions within the lungs.

A client with bacterial pneumonia is to be started on I.V. antibiotics. Which of the following diagnostic tests must be completed before antibiotic therapy begins? 1. Urinalysis. 2. Sputum culture. 3. Chest radiograph. 4. Red blood cell count.

2. A sputum specimen is obtained for culture to determine the causative organism. After the organism is identified, an appropriate antibiotic can be prescribed. Beginning antibiotic therapy before obtaining the sputum specimen may alter the results of the test. Neither a urinalysis, a chest radiograph, nor a red blood cell count needs to be obtained before initiation of antibiotic therapy for pneumonia.

A client with chronic obstructive pulmonary disease (COPD) is experiencing dyspnea and has a low PaO2 level. The nurse plans to administer oxygen as ordered. Which of the following statements is true concerning oxygen administration to a client with COPD? 1. High oxygen concentrations will cause coughing and dyspnea. 2. High oxygen concentrations may inhibit the hypoxic stimulus to breathe. 3. Increased oxygen use will cause the client to become dependent on the oxygen. 4. Administration of oxygen is contraindicated in clients who are using bronchodilators.

2. Clients who have a long history of COPD may retain carbon dioxide (CO2). Gradually the body adjusts to the higher CO2 concentration, and the high levels of CO2 no longer stimulate the respiratory center. The major respiratory stimulant then becomes hypoxemia. Administration of high concentrations of oxygen eliminates this respiratory stimulus and leads to hypoventilation. Oxygen can be drying if it is not humidified, but it does not cause coughing and dyspnea. Increased oxygen use will not create an oxygen dependency; clients should receive oxygen as needed. Oxygen is not contraindicated with the use of bronchodilators.

The nurse reviews an arterial blood gas report for a client with chronic obstructive pulmonary disease (COPD). pH 7.35; PC02 62; PO2 70; HCO3 34 The nurse should: 1. Apply a 100% non-rebreather mask. 2. Assess the vital signs. 3. Reposition the client. 4. Prepare for intubation.

2. Clients with chronic COPD have CO2 retention and the respiratory drive is stimulated when the PO2 decreases. The heart rate, respiratory rate, and blood pressure should be evaluated to determine if the client is hemodynamically stable. Symptoms, such as dyspnea, should also be assessed. Oxygen supplementation, if indicated, should be titrated upward in small increments. There is no indication that the client is experiencing respiratory distress requiring intubation.

When teaching a client with chronic obstructive pulmonary disease to conserve energy, the nurse should teach the client to lift objects: 1. While inhaling through an open mouth. 2. While exhaling through pursed lips. 3. After exhaling but before inhaling. 4. While taking a deep breath and holding it.

2. Exhaling requires less energy than inhaling. Therefore, lifting while exhaling saves energy and reduces perceived dyspnea. Pursing the lips prolongs exhalation and provides the client with more control over breathing. Lifting after exhaling but before inhaling is similar to lifting with the breath held. This should not be recommended because it is similar to the Valsalva maneuver, which can stimulate cardiac arrhythmias.

A client treated for hypertension with furosemide (Lasix), atenolol (Tenormin), and ramipril (Altace) develops a second degree heart block Mobitz type 1. Which of the following actions should the nurse take? 1. Administer a 250 mL fluid bolus. 2. Withhold the atenolol. 3. Prepare for cardioversion. 4. Set up for an arterial line.

2. The client may be asymptomatic and the underlying cause should be assessed. Drugs that block the AV node should be avoided, such as beta blockers (Atenolol), calcium channel blockers, digoxin, and amiodarone. Symptomatic clients are treated with atropine and transcutaneous pacing. There is no indication for a fluid bolus, cardioversion, or arterial line.

A client's arterial blood gas values are as follows: pH, 7.31; PaO2, 80 mm Hg; PaCO2, 65 mm Hg; HCO3 −, 36 mEq/ L. The nurse should assess the client for? 1. Cyanosis. 2. Flushed skin. 3. Irritability. 4. Anxiety.

2. The high PaCO2 level causes flushing due to vasodilation. The client also becomes drowsy and lethargic because carbon dioxide has a depressant effect on the central nervous system. Cyanosis is a sign of hypoxia. Irritability and anxiety are not common with a PaCO2 level of 65 mm Hg but are associated with hypoxia.

The nurse should teach the client with asthma that which of the following is one of the most common precipitating factors of an acute asthma attack? 1. Occupational exposure to toxins. 2. Viral respiratory infections. 3. Exposure to cigarette smoke. 4. Exercising in cold temperatures.

2. The most common precipitator of asthma attacks is viral respiratory infection. Clients with asthma should avoid people who have the flu or a cold and should get yearly flu vaccinations. Environmental exposure to toxins or heavy particulate matter can trigger asthma attacks; however, far fewer asthmatics are exposed to such toxins than are exposed to viruses. Cigarette smoke can also trigger asthma attacks, but to a lesser extent than viral respiratory infections. Some asthmatic attacks are triggered by exercising in cold weather.

The nurse is discharging a client from an inpatient alcohol treatment unit. Which of the following statements by the client's wife indicates to the nurse that the family is coping adaptively? "1. ""My husband will do well as long as I keep him engaged in activities he likes. "" 2. ""My focus is learning how to live my life."" 3. ""I am so glad that our problems are behind us."" 4. ""I'll make sure that the children don't give my husband any problems."""

2. ""My focus is learning how to live my life."""1. ""My husband..."" - wife is accepting responsibility, codependent behavior 2. ""My focus..."" - (CORRECT): wife is working to change codepended patterns 3. ""I am so glad..."" - unrealistic; discharge is not the final step of treatment 4. ""I'll make sure..."" - wife is accepting responsibility, codependent behavior"

The nurse has just finished explaining the necessity of coughing and deep breathing following surgery to a perioperative client. Which of the following responses by the client would indicate his understanding and acceptance of what he has been taught? "1. ""I thought that spirometry thing was supposed to do the job."" 2. ""When I do the coughing and deep breathing, I reduce my chances of getting pneumonia."" 3. ""It really hurts too much to do that. Deep breathing and coughing are impossible."" 4. ""I guess I'll try to remember to take a couple of deep breaths once and a while.""

2. ""When I do the coughing and deep breathing, I reduce my chances of getting pneumonia.Deep-breathing exercises are encouraged. These exercises help remove mucus, which can form and remain in the lungs due to the effects of general anesthetic and analgesics. Deep breathing increases lung expansion and prevents the accumulation of secretions. It helps prevent pneumonia and atelectasis, which may result from stagnation of fluid in the lungs.

A patient is to have elective surgery. The RN should plan with the patient to schedule the surgery: 1. If the patient wished to have the procedure done. 2. At the patient's convenience 3. Within the next 2 wteeks. 4. Within the next 2 days.

2. AT THE PATIENT'S CONVENIENCE.Rationale: Since this is an elective surgery it will be scheduled at the patient's convenience when the surgeon is available. There is not a time constraint on elective procedures.

Normal BP?

<120/<80; greater than this is pre-hypertensive

Normal LDL?

<160

normal post-prandial blood level taken 2 hours after a meal

<180mg/dl

Glucose - Lab Values: child

<2 60-100 >2 70-110

What is normal capillary refill?

<2 seconds

Normal CK-MB?

<2.5

Cholesterol - Lab Values: Adult

<200

The nurse teaching a pt. w/ GERD includes which of the following instructions? Select all that apply

2. Elevate the head of the bed on 6-8 inch blocks 5. avoid lying down for several hours after eating

A 46-y.o. woman from Bosnia came to the US 6 yrs ago. Although she did not celebrate Christmas when she lived in Bosniaa, she celebrates Christmas with her family now. This woman has experienced assimilation into the culture of the US because she: 1. Choose to be bicultural 2. Adapted to and adopted the American culture 3. Had an extremely negative experience with the American culture 4. Gave up part of her ethnic identity in favor of the American culture

2. adapted

Normal Total Cholesterol?

<200

Normal Prostate Specific Antigen?

<4

What is the pH of stomach acid?

<4

Bryant's traction

= children <3y, <35 lbs with femur fx

caput succedaneum

= diffuse edema of the fetal scalp that crosses the suture lines. Swelling reabsorbs within 1 to 3 days.

Dunlap traction

= skeletal or skin

Russell traction

=femur or lower leg

Buck's traction

=knee immobility

bethamethasone (celestone)

=surfactant. Med for lung expansion.

What is a normal CD4/CD8 ratio?

>1

What are the parameters of blood pressure when diagnosing hypertension?

>140/90

Normal HDL?

>35

following a kidney transplant, the nurse notes that the pt's urine is cloudy. how should the nurse respond?

notify the physician

speaking of TB... PPD is positive if area of induration is:

>5 mm in an immunocompromised patient >10 mm in a normal patient >15 mm in a patient who lives in an area where TB is very rare.

Normal GFR?

>60

1 kg:

2.2 lbs

What is a normal PAS?

20-30 mmHg

The nursing instructor is going over laboratory results for HIV/AIDS patients. The instructor tells the students that upon interpretation of the patient's labs the nurse should recognize that a patient with HIV is considered to have AIDs when the CD4+ T-lymphocyte cell count drops below what?

200 cells/mm^3 of blood

A client with hypertensive emergency is being treated with sodium nitroprusside (Nipride). In a dilution of 50 mg/ 250 mL, how many micrograms of Nipride are in each milliliter? ________________________ mcg.

200 mcg First, calculate the number of milligrams per milliliter: Next, calculate the number of micrograms in each milligram: CN: Pharmacological and parenteral

What percentage of room air is O2?

21%

HCO3 - Lab Values: Adult

21-28

HCO3 - Lab Values: Child

21-28

HCO3 - Lab Values: adult

21-28

Normal HCO3

22 - 29

Normal arterial Bicarb (HCO3)?

22-26

Normal venous CO2?

22-29

The nurse is caring for a patient with suspected renal dysfunction is assessing ankle and sacral edema and the presence of crackles in the bases of the lungs bilaterally. A daily weight indicates that the patient's weight has incrased by 5 pounds in the past 24 hours. The nurse notes that evaluation of the patient's intake and output records for the prior 24 hours indicate greater fluid intake than urine output. Based upon the assessment findings and the daily weight information, the nurse estimates that the patient has retained how much fluid?

2273 mL of fluid in 24 hours

Normal PTT?

23.8-34.6 (heparin)

Pco2 - Lab Values: children under 2

26-41

Low Density Lipoprotien - Lab Values: Newborn

29

A nurse is reviewing a physician's order sheet for a preoperative client that states that the client must be NPO after midnight. The nurse would telephone the physician to clarify whether which of the following medications should be given to the client and not withheld? 1) Ferrous sulfate. 2) Prednisone (Deltasone) . 3) Cyclobenzaprine (Flexeril). 4) Conjugated estrogen (Premarin).

2: Prednisone (Deltasone)Prednisone is a corticosteroid. With prolonged use, corticosteroids cause adrenal atrophy, which reduces the ability of the body to withtand stress. When stress is severe, corticosteroids are essential to life. Before and during surgery, dosages may be increased temporarily. Ferrous sulfate is an oral iron preparation used to treat iron deficiency anemia. Cyclobenzaprine is a skeletal muscle relaxant. Conjugated strogen is used for hormone replacement therapy in postmenopausal women. These three meds can be withheld before surgery without consequences.

Chp. 44..You are teaching a client about healthy nutrition. You recognize that the client understands the teaching when he states: 1. i need to stop eating red meat 2. I will increase the servings of fruit juice to 4X day 3. I will make sure that I eat a balanced diet and exercise reg. 4. I will not eat so many dark green vegetables and eat more yellow vegetables

3

Epiglottitis

3 D's: Drooling Dysphonia Dysphagia

1 Tsp (tablespoon):

3 teaspoons or 15 mL

Urea Nitrogen - Lab Results: Newborn

3-12

Normal phosphorus?

3-4

SSRIs (antidepressants), take how long to work?

3-4 weeks

Evidence-based practice is defined as: 1. nursing care based on tradition 2. scholarly inquiry of nursing and biomedical research literature 3. a problem-solving approach to clinical practice based on best practices 4. quality nursing care provided in an efficient and economically sound manner

3.

The dominant values in American society on individual autonomy and self-determination: 1. Do not have an effect on health care 2. Rarely have an effect on other cultures 3. May be in direct conflict with diverse groups 4. May hinder ability to get into hospice programs

3.

Which of the following mental status changes may occur when a client with pneumonia is first experiencing hypoxia? 1. Coma. 2. Apathy. 3. Irritability. 4. Depression.

3. Clients who are experiencing hypoxia characteristically exhibit irritability, restlessness, or anxiety as initial mental status changes. As the hypoxia becomes more pronounced, the client may become confused and combative. Coma is a late clinical manifestation of hypoxia. Apathy and depression are not symptoms of hypoxia.

A client with pneumonia has a temperature of 102.6 ° F (39.2 ° C), is diaphoretic, and has a productive cough. The nurse should include which of the following measures in the plan of care? 1. Position changes every 4 hours. 2. Nasotracheal suctioning to clear secretions. 3. Frequent linen changes 4. Frequent offering of a bedpan.

3. Frequent linen changes are appropriate for this client because of the diaphoresis. Diaphoresis produces general discomfort. The client should be kept dry to promote comfort. Position changes need to be done every 2 hours. Nasotracheal suctioning is not indicated with the client's productive cough. Frequent offering of a bedpan is not indicated by the data provided in this scenario.

When caring for the client who is receiving an aminoglycoside antibiotic, the nurse should monitor which of the following laboratory values? 1. Serum sodium. 2. Serum potassium. 3. Serum creatinine. 4. Serum calcium.

3. It is essential to monitor serum creatinine in the client receiving an aminoglycoside antibiotic because of the potential of this type of drug to cause acute tubular necrosis. Aminoglycoside antibiotics do not affect serum sodium, potassium, or calcium levels.

The nurse teaches a client with chronic obstructive pulmonary disease (COPD) to assess for signs and symptoms of right-sided heart failure. Which of the following signs and symptoms should be included in the teaching plan? 1. Clubbing of nail beds. 2. Hypertension. 3. Peripheral edema. 4. Increased appetite.

3. Right-sided heart failure is a complication of COPD that occurs because of pulmonary hypertension. Signs and symptoms of right-sided heart failure include peripheral edema, jugular venous distention, hepatomegaly, and weight gain due to increased fluid volume. Clubbing of nail beds is associated with conditions of chronic hypoxemia. Hypertension is associated with left-sided heart failure. Clients with heart failure have decreased appetites.

Essential hypertension would be diagnosed in a 40-year-old male whose blood pressure readings were consistently at or above which of the following? 1. 120/ 90 mm Hg. 2. 130/ 85 mm Hg. 3. 140/ 90 mm Hg. 4. 160/ 80 mm Hg.

3. American Heart Association standards define hypertension as a consistent systolic blood pressure level greater than 140 mm Hg and a consistent diastolic blood pressure level greater than 90 mm Hg.

The client realizes the importance of quitting smoking, and the nurse develops a plan to help the client achieve this goal. Which of the following nursing interventions should be the initial step in this plan? 1. Review the negative effects of smoking on the body. 2. Discuss the effects of passive smoking on environmental pollution. 3. Establish the client's daily smoking pattern. 4. Explain how smoking worsens high blood pressure.

3. A plan to reduce or stop smoking begins with establishing the client's personal daily smoking pattern and activities associated with smoking. It is important that the client understands the associated health and environmental risks, but this knowledge has not been shown to help clients change their smoking behavior.

A client with pneumonia is experiencing pleuritic chest pain. The nurse should assess the client for: 1. A mild but constant aching in the chest. 2. Severe midsternal pain. 3. Moderate pain that worsens on inspiration. 4. Muscle spasm pain that accompanies coughing.

3. Chest pain in pneumonia is generally caused by friction between the pleural layers. It is more severe on inspiration than on expiration, secondary to chest wall movement. Pleuritic chest pain is usually described as sharp, not mild or aching. Pleuritic chest pain is not localized to the sternum, and it is not the result of a muscle spasm.

The most important long-term goal for a client with hypertension would be to: 1. Learn how to avoid stress. 2. Explore a job change or early retirement. 3. Make a commitment to long-term therapy. 4. Lose weight.

3. Compliance is the most critical element of hypertension therapy. In most cases, hypertensive clients require lifelong treatment and their hypertension cannot be managed successfully without drug therapy. Stress management is an important component of hypertension therapy, but the priority goal is related to compliance. It is not necessary for the client to change jobs or retire, but rather to learn to manage stress if the job is stressful. Losing weight may be necessary and will contribute to lower blood pressure, but the client must first accept the need for a lifelong management plan to control the hypertension.

A client with acute asthma is prescribed short-term corticosteroid therapy. Which is the expected outcome for the use of steroids in clients with asthma? 1. Promote bronchodilation. 2. Act as an expectorant. 3. Have an anti-inflammatory effect. 4. Prevent development of respiratory infections.

3. Corticosteroids have an anti-inflammatory effect and act to decrease edema in the bronchial airways and decrease mucus secretion. Corticosteroids do not have a bronchodilator effect, act as expectorants, or prevent respiratory infections.

A nurse notes that a client has kyphosis and generalized muscle atrophy. Which of the following problems is a priority when the nurse develops a nursing plan of care? 1. Infection. 2. Confusion. 3. Ineffective coughing and deep breathing. 4. Difficulty chewing solid foods.

3. In kyphosis, the thoracic spine bends forward with convexity of the curve in a posterior direction, making effective coughing and deep breathing difficult. Although the client may develop other problems because respiratory status deteriorates when pulmonary secretions are not adequately cleared from airways, ineffective coughing and deep breathing should receive priority attention.

The nurse teaches a client, who has recently been diagnosed with hypertension, about dietary restrictions: a low-calorie, low-fat, low-sodium diet. Which of the following menu selections would best meet the client's needs? 1. Mixed green salad with blue cheese dressing, crackers, and cold cuts. 2. Ham sandwich on rye bread and an orange. 3. Baked chicken, an apple, and a slice of white bread. 4. Hot dogs, baked beans, and celery and carrot sticks.

3. Processed and cured meat products, such as cold cuts, ham, and hot dogs, are all high in both fat and sodium and should be avoided on a low-calorie, low-fat, low-salt diet. Dietary restrictions of all types are complex and difficult to implement with clients who are basically asymptomatic.

The nurse administers theophylline (Theo-Dur) to a client. To evaluate the effectiveness of this medication, which of the following drug actions should the nurse anticipate? 1. Suppression of the client's respiratory infection. 2. Decrease in bronchial secretions. 3. Relaxation of bronchial smooth muscle. 4. Thinning of tenacious, purulent sputum.

3. Theophylline (Theo-Dur) is a bronchodilator that is administered to relax airways and decrease dyspnea. Theophylline is not used to treat infections and does not decrease or thin secretions.

A client who has been taking flunisolide (AeroBid), two inhalations a day, for treatment of asthma.has painful, white patches in his mouth. Which response by the nurse would be most appropriate? 1. "This is an anticipated adverse effect of your medication. It should go away in a couple of weeks." 2. "You are using your inhaler too much and it has irritated your mouth." 3. "You have developed a fungal infection from your medication. It will need to be treated with an antifungal agent." 4. "Be sure to brush your teeth and floss daily. Good oral hygiene will treat this problem."

3. Use of oral inhalant corticosteroids such as flunisolide (AeroBid) can lead to the development of oral thrush, a fungal infection. Once developed, thrush must be treated by antifungal therapy; it will not resolve on its own. Fungal infections can develop even without overuse of the corticosteroid inhaler. Although good oral hygiene can help prevent development of a fungal infection, it cannot be used alone to treat the problem.

THE NURSE WOULD RECOGNIZE THAT AN OBESE MALE PATIENT WHO HAS BEEN DIAGNOSED WITH OBSTRUCTIVE SLEEP APNEA FACES AN INCREASED RISK OF WHICH OF THE FOLLOWING? A DEPRESSION B RESPIRATORY ACIDOSIS C HEART DISEASE D SEIZURES

C

THE PHYSICIANS ADMITTING ORDERS INDICATE THAT THE PATIENT IS TO BE PLACED IN A FOWLERS POSITION, UPON POSITIONING THIS PATIENT, HOW MUCH WILL THE NURSE ELEVATE THE HEAD? A 15 B 90 C 45-60 D 30

C

Describe intermittent claudications:

pain related to peripheral vascular disease, the pain occurs with exercise and disappears with rest

Chronic pancreatitis

pancreatic enzymes are given with meals.

5 symptoms of hyperglycemia:

polydipsia, polyuria, polyphagia, wekness and weight loss

Ascites

portal hypotension + albuminemia

Jane is a nursing student who is doing her community health rotation in an inner-city public heatlh department. When she investigates sociodemographic and health data of the people served by the health department, Jane detects disparities in health outcomes between the rich and poor. This is an example of: 1. Illness attributed to natural and biological factors 2. Creation of Jane's interpretation and descriptions of the data 3. Influence of socioeconomic factors in morbidity and mortality 4. Combination of naturalistic, religious, and supernatural modalities

3. Influence of socioeconomic factors

Lacey conrad, an 18 yr old woman, is in the emergency dept. with a fever & cough. The physician asks you to obtain her vital signs, auscultate her lung sounds, listen to her heart sounds, determine her level of comfort, and collect blood and sputum samples for analysis. Which standard of practice are you performing? 1. Diagnosis 2. Evaluation 3. Assessment 4. Implement

3. assessment

Potassium - Lab Values: Child

3.4-4.7

Normal Albumin?

3.4-4.8

Normal Potassium?

3.5-5

Potassium - Lab Values: Adult

3.5-5

Albumin - Lab Values: Adult

3.5-5 Pregnant - slightly high

Normal RBC?

3.5-5.5

1 cup:

30 mL

1 oz:

30 mL

Activated Partial Thromboplastin Time - Lab Values - newborn

30-40

During one rescuer CPR what is the ratio of compressions to breaths in adult patients and pediatric patients?

30:2 in adults and 15:2 in children

The catheter of the client receiving PN becomes occluded. Order the steps for caring for the occluded catheter in the order in which you would perform them. 1. Attempt to aspirate a clot 2. Temporarily stop the infusion 3. Flush line with saline or heparine 4. Use a thrombolytic agent if ordered or per protocol

3124

Hematocrit - Lab Values - Child

32-40

Normal Hematocrit?

34-48%

High Density Lipoprotien - Lab Values: Newborn

35

Normal PCO2

35 - 45

Normal arterial PaCO2?

35-45

Pco2 - Lab Values: adult

35-45

TEF

4 C's: Coughing Choking Cyanosis Continuous drooling

Heart defects. Remember for cyanotic:

3Ts: Tetralogy of Fallot, Truncys arteriosus, and Transporsition of the great vessels. Prevents blood from going to the heart. If problem does not fix itself, or cannot be fixed surgically; CHF will occur followed by death

Type of Syringe and Needle for IM

3ml 20-25g 1-1.5"

Type of Syringe and Needle for SQ

3ml 25-29 g 1/2-5/8"

A client's personal preferences for hygiene are influenced by a number of factors. 1. The nurse is in charge of the care 2. Hygiene care is a routine procedure 3. Hygiene has no influence on client outcomes 4. No two individuals perform hygiene in the same manner

4

Based on knowledge of peptic ulcer disease (PUD), the nurse anticipates the presence of which bacteria when reviewing the laboratory data for a client suspected of having PUD? 1. Micrococcus 2. Staphylococcus 3. Corynebacterium 4. Helicobacter pylori

4

Lacey remains in the ER and has developed wheezing and SOB. The physician orders a medicated nebulizer tx now and in 4 hrs. Which standard of care are you performing? 1. Planning 2. Evaluation 3. Assessment 4. Implementation

4

The examination for RN licensure is exactly the same in every state in the US. This examination: 1. Guarantees safe nursing care for all clients 2. Ensures standard nursing care for all clients 3. Ensures that honest and ethical care is provided 4. Provides a minimal standard of knowledge for practice

4

You are assessing a client receiving enteral feeding via a small-bore nasogastric tube. Which assessment findings need further intervention? 1. Gastric pH of 6.0 during placement check 2. Wt. gain of 1 pound over the course of a week 3. Active bowel sounds in the 4 ab quadrants 4. Gastric residual aspirate of 300 mL for the second consecutive time

4

what is a normal WBC count?

4,500-11,000/mm3

What is a normal range for wedge pressure?

4-12 mmHg

Red Blood Cell Count Lab Values: Child

4-5

what are early s/s of alcohol withdrawal?

4-6h after the last drink: tremors, agitation, anxiety, insomnia, mild tachycardia, HTN

What is a normal cardiac output?

4-8 L/minute

Brooke is a nursing student. In order for Brooke to enhance her cultural awareness, she will need to make an in-depth self-examination of her: 1. Motivation and commitment to caring 2. Social, cultural, and biophysical factors 3. Engagement in cross-cultural interactions 4. Background, recognizing her biases and prejudices

4.

Cultural competence is the process of: 1. Learning about vast cultures 2. Motivation and commitment to caring 3. Influencing treatment and care of clients 4. Acquiring specific knowledge, skills, and attitudes

4.

Culture strongly influences pain expression and need for pain medication. However, cultural pain: 1. Is not expressed verbally or physically 2. Is expressed only to others of liek culture 3. Is more intense, thus necessitating more medication 4. May be suffered by a client whose valued way of life is disregarded by practitioners

4.

Which of the following is an expected outcome of pursed-lip breathing for clients with emphysema? 1. To promote oxygen intake. 2. To strengthen the diaphragm. 3. To strengthen the intercostal muscles. 4. To promote carbon dioxide elimination.

4. Pursed-lip breathing prolongs exhalation and prevents air trapping in the alveoli, thereby promoting carbon dioxide elimination. By prolonging exhalation and helping the client relax, pursed-lip breathing helps the client learn to control the rate and depth of respiration. Pursed-lip breathing does not promote the intake of oxygen, strengthen the diaphragm, or strengthen intercostal muscles.

The cyanosis that accompanies bacterial pneumonia is primarily caused by which of the following? 1. Decreased cardiac output. 2. Pleural effusion. 3. Inadequate peripheral circulation. 4. Decreased oxygenation of the blood.

4. A client with pneumonia has less lung surface available for the diffusion of gases because of the inflammatory pulmonary response that creates lung exudate and results in reduced oxygenation of the blood. The client becomes cyanotic because blood is not adequately oxygenated in the lungs before it enters the peripheral circulation. Decreased cardiac output may be a comorbid condition in some clients with pneumonia; however, it is not the cause of cyanosis. Pleural effusions are a potential complication of pneumonia but are not the primary cause of decreased oxygenation. Inadequate peripheral circulation is also not the cause of the cyanosis that develops with bacterial pneumonia.

Which of the following is an appropriate expected outcome for an adult client with well-controlled asthma? 1. Chest X-ray demonstrates minimal hyperinflation. 2. Temperature remains lower than 100 ° F (37. 8 ° C). 3. Arterial blood gas analysis demonstrates a decrease in PaO2. 4. Breath sounds are clear.

4. Between attacks, breath sounds should be clear on auscultation with good air flow present throughout lung fields. Chest X-rays should be normal. The client should remain afebrile. Arterial blood gases should be normal.

When instructing clients on how to decrease the risk of chronic obstructive pulmonary disease (COPD), the nurse should emphasize which of the following? 1. Participate regularly in aerobic exercises. 2. Maintain a high-protein diet. 3. Avoid exposure to people with known respiratory infections. 4. Abstain from cigarette smoking.

4. Cigarette smoking is the primary cause of COPD. Other risk factors include exposure to environmental pollutants and chronic asthma. Participating in an aerobic exercise program, although beneficial, will not decrease the risk of COPD. Insufficient protein intake and exposure to people with respiratory infections do not increase the risk of COPD.

The nurse assesses the respiratory status of a client who is experiencing an exacerbation of chronic obstructive pulmonary disease (COPD) secondary to an upper respiratory tract infection. Which of the following findings would be expected? 1. Normal breath sounds. 2. Prolonged inspiration. 3. Normal chest movement. 4. Coarse crackles and rhonchi.

4. Exacerbations of COPD are commonly caused by respiratory infections. Coarse crackles and rhonchi would be auscultated as air moves through airways obstructed with secretions. In COPD, breath sounds are diminished because of an enlarged anteroposterior diameter of the chest. Expiration, not inspiration, becomes prolonged. Chest movement is decreased as lungs become overdistended.

A 34-year-old female with a history of asthma is admitted to the emergency department. The nurse notes that the client is dyspneic, with a respiratory rate of 35 breaths/ minute, nasal flaring, and use of accessory muscles. Auscultation of the lung fields reveals greatly diminished breath sounds. Based on these findings, which action should the nurse take to initiate care of the client? 1. Initiate oxygen therapy and reassess the client in 10 minutes. 2. Draw blood for an arterial blood gas analysis and send the client for a chest X-ray. 3. Encourage the client to relax and breathe slowly through the mouth. 4. Administer bronchodilators.

4. In an acute asthma attack, diminished or absent breath sounds can be an ominous sign indicating lack of air movement in the lungs and impending respiratory failure. The client requires immediate intervention with inhaled bronchodilators, I.V. corticosteroids and, possibly, I.V. theophylline (Theo-Dur). Administering oxygen and reassessing the client 10 minutes later would delay needed medical intervention, as would drawing blood for an arterial blood gas analysis and obtaining a chest X-ray. It would be futile to encourage the client to relax and breathe slowly without providing the necessary pharmacologic intervention.

Which of the following indicates that the client with chronic obstructive pulmonary disease (COPD) who has been discharged to home understands his care plan? 1. The client promises to do pursed-lip breathing at home. 2. The client states actions to reduce pain. 3. The client says that he will use oxygen via a nasal cannula at 5 L/ minute. 4. The client agrees to call the physician if dyspnea on exertion increases.

4. Increasing dyspnea on exertion indicates that the client may be experiencing complications of COPD. Therefore, the nurse should notify the physician. Extracting promises from clients is not an outcome criterion. Pain is not a common symptom of COPD. Clients with COPD use low-flow oxygen supplementation (1 to 2 L/ minute) to avoid suppressing the respiratory drive, which, for these clients, is stimulated by hypoxia. .

A client's job involves working in a warm, dry room, frequently bending and crouching to check the underside of a high-speed press, and wearing eye guards. Given this information, the nurse should assess the client for which of the following? 1. Muscle aches. 2. Thirst. 3. Lethargy. 4. Orthostatic hypotension.

4. Possible dizziness from orthostatic hypotension when rising from a crouched or bent position increases the client's risk of being injured by the equipment. The nurse should assess the client's blood pressure in all three positions (lying, sitting, and standing) at all routine visits. The client may experience muscle aches, or thirst from working in a warm, dry room, but these are not as potentially dangerous as orthostatic hypotension. The client should not be experiencing lethargy.

An exercise program is prescribed for the client with hypertension. Which intervention would be most likely to assist the client in maintaining an exercise program? 1. Giving the client a written exercise program. 2. Explaining the exercise program to the client's spouse. 3. Reassuring the client that he or she can do the exercise program. 4. Tailoring a program to the client's needs and abilities.

4. Tailoring or individualizing a program to the client's lifestyle has been shown to be an effective strategy for changing health behaviors. Providing a written program, explaining the program to the client's spouse, and reassuring the client that he or she can do the program may be helpful but are not as likely to promote adherence as individualizing the program.

Which of the following diets would be most appropriate for a client with chronic obstructive pulmonary disease (COPD)? 1. Low-fat, low-cholesterol diet. 2. Bland, soft diet. 3. Low-sodium diet. 4. High-calorie, high-protein diet.

4. The client should eat high-calorie, high-protein meals to maintain nutritional status and prevent weight loss that results from the increased work of breathing. The client should be encouraged to eat small, frequent meals. A low-fat, low-cholesterol diet is indicated for clients with coronary artery disease. The client with COPD does not necessarily need to follow a sodium-restricted diet, unless otherwise medically indicated. There is no need for the client to eat bland, soft foods.

Which of the following measures would most likely be successful in reducing pleuritic chest pain in a client with pneumonia? 1. Encourage the client to breathe shallowly. 2. Have the client practice abdominal breathing. 3. Offer the client incentive spirometry. 4. Teach the client to splint the rib cage when coughing.

4. The pleuritic pain is triggered by chest movement and is particularly severe during coughing. Splinting the chest wall will help reduce the discomfort of coughing. Deep breathing is essential to prevent further atelectasis. Abdominal breathing is not as effective in decreasing pleuritic chest pain as is splinting of the rib cage. Incentive spirometry facilitates effective deep breathing but does not decrease pleuritic chest pain.

A nurse on a pediatric unit is preparing the assignment for the evening shift. The unit employs unlicensed assistive personnel (UAP). Which task is most appropriate for the nurse to delegate to the UAP? a) setting up Bryant's traction b) completing the FACES pain scale for a child with sickle cell crisis c) obtaining post-operative vital signs on a client status post-tonsillectomy d) setting up an intravenous therapy pump

A

An elderly lady is diagnosed with pernicious anemia. The nurse caring for this patient knows that intrinsic factor, which combines with dietary B12, is secreted by what?

Gastric mucosa

what is a T tube?

tube inserted to ensure drainage of common bile duct after cholecystectomy

Hematocrit - Lab Values - Newborn

44-64

"Which of the laboratory result would require immediate intervention by the nurse for the client scheduled for surgery? "1. Calcium 9.2 mg/dL 2. Bleeding time 2min 3. Hemoglobin 15 g/dL 4. Potassium 2.4 mEq/L

4: Potassium 2.4 mEq/L "1. This laboratory value is within normal limits 2. This laboratory value is within normal limits 3. This laboratory value is within normal limits 4. This potassium levels is low and should be reported to the health care provider because potassium is important for muscle function, including the cardiac muscle"

1 tsp (teaspoon):

5 mL

The nurse is tracking data on a group of clients with HF who have been discharged from the hospital and are being followed at a clinic. What data indicates that nursing interventions of monitoring and teaching has been effective?

5 percent of the clients required hospitalization in the last 90 days

White Blood Cell Count - Lab Value: Adult

5-10

Expected physiological weight after birth?

5-10%

Thyroxine (T-4) - Lab Values: Adult

5-12

What is the normal range for PEEP and CPAP?

5-15 cm H2O

Urea Nitrogen - Lab Results: child

5-18

Normal Glucose?

70-110

Triiodothyronine (T-3) Lab Values: adult

70-205

What is the normal range for MAP?

70-90 mmHg

What is a normal PAD?

8-15 mmHg or 2-5 mmHg higher then the wedge pressure

Normal Calcium?

8.6-10.2

Calcium - Lab Values: Child

8.8-10.8

What are the contents of the cranial vault?

80%- brain tissue, 12%- blood, and 8% CSF

Normal PaO2?

80-100

Po2 - Lab Values: Adult

80-100

Iron: Lab Values: Adult

80-160 M 80-180 F 60-160

What is a normal SVR?

800-1200 dynes/sec/cm^-5

Normal CD4? CD8?

800-2500; 600-1500

Calcium - Lab Values: Adult

9-10.5

Calcium - Lab Values: infant older than 10 days

9-10.6

White Blood Cell Count - Lab Value: Newborn

9-30

Hematocrit - Lab Values - Adult

42-47 M-42-52 F-37-47 Pregnant F - >33

Normal WBC?

4.5-11.5

Albumin - Lab Values: Child

4.5-9

what is a normal RBC count?

4.6-6.2/mm3

Red Blood Cell Count Lab Values: Adult

4.7-5.4 M-4.7-6.1 F-4.2-5.4

Red Blood Cell Count Lab Values: Newborn

4.8-7.1

O2 Saturation - Lab Values: Newborn

40-100%

Normal PT?

9.8-12.3 (coumadin)

normal pre-prandial blood glucose level

90-130mg/dl

O2 Saturation - Lab Values: adult

95-100%

O2 Saturation

95-100% Oxygen saturation refers to the level of oxygen carried by red blood cells through the arteries and delivered to internal organs. As red blood cells travel through the lungs, they are saturated with oxygen.

5. A school nurse is teaching a class of junior-high students about the effects of smoking. This educational program will meet which of the aims of nursing? a. Promoting health b. Preventing illness c. Restoring health d. Facilitating coping with disability or death

5. b. Educational programs can reduce the risk of illness by teaching good health habits.

Normal neonate weight?

5.5 - 8 lbs 13 oz

Huntington's Chorea:

50% genetic, autosomal dominant disorder S/S: Chorea--writhing, twisting, movements of the face, limbs and body; gait deteriorates to the point of no ambulation; no cure--just palliative

Iron: Lab Values: Child

50-120

What is a normal PVR?

50-250 dynes/sec/cm^-5

Creatine Phosphokinase - Lab Values: Adult

55-135 M 55-170 F 30-135

Creatine Phosphokinase - Lab Values: child

55-135 M 55-170 F 30-135

Normal CK?

55-170

Diaphragm must stay in place for:

6 hours after intercourse. They are also fitted and must be refitted if you lose or gain a significant amount of weight.

Birth weight doubles by:

6 months

When does NPH insulin peak?

6-14 hours

Albumin - Lab Values: Infant

6-6.7 Preemie: 3-4.2

White Blood Cell Count - Lab Value: Child

6.2-10

Normal Protein?

6.6-8.7

A patient's vital signs are pulse 80, respirations 24, and BP of 124/60 mm Hg and cardiac output is 4.8 L/min. What is the patient's stroke volume? ____________________

60 mL rationale: Stroke volume = cardiac output/heart rate

1 gr (grain):

60 mg

Normal Heart Rate?

60-100

Partial Thromboplastin Time - Lab Values: Child

60-70

Partial Thromboplastin Time - Lab Values: Newborn

60-70

Po2 - Lab Values: Newborn

60-70

Partial Thromboplastin Time - Lab Values: Adult

60-70 Pregnant: slightly lower

Creatine Phosphokinase - Lab Values: Newborn

65-580

Tidal Volume:

7-10 mL/kg

What is the normal range for Vt?

7-9 ml/kg or 500-800 ml for an adult

Stranger anxiety is greatest:

7-9 months

Normal PH

7.35 - 7.45

Normal arterial pH?

7.35-7.45

pH of Blood - Lab Value: adult

7.35-7.45

Calcium - Lab Values: Newborn

7.6-10.4

Glucose - Lab Values: Adult

70-100

normal fasting blood glucose no food for at least 8 hours

70-100

Navane

: tx of schizophrenia..assess for EPS

Premarin

:tx after menopause estrogen replacement

ICP should be:

< 2 (measure the head circumerference)

Normal Troponin?

<0.2

A GRADUATE NURSE IS ADMINISTERING SEVERAL MEDICATIONS TO A NEWLY ADMITTED PATIENT. WHO IS LEGALLY RESPONSIBLE FOR THE DRUGS ADMINISTERED BY THIS NURSE? A THE NURSE ADMINISTERING THE DRUG B PHARMACIST WHO DISPENSED C NURSE MANAGER D PHYSICIAN WHO WROTE THE ORDER

A

AT WHAT POINT SHOULD THE NURSE DO THE 3 CHECKS OF MEDICATION ADMINISTRATION? A AS THE NURSE REACHES FOR THE DRUG PACKAGE B WHEN REVIEWING THE PATIENT;S MAR C AT THE BEGINNING OF SHIFT D AFTER RETRIEVING THE DRUG

A

An operating room nurse has just finished setting up a sterile field for a kidney transplant surgery. She gets word that the donor organ will not be available for another thirty minutes. Which of the following is the best course of action? A) Personally watch the sterile field to ensure that it is not broken. B) Place cones or barriers in front of the main OR doors. C) Place sterile drapes over all surfaces. D) Thirty minutes is too long. The sterile field will need to be broken and reestablished later

A

THE NURSE SHOULD USE EXTREME CAUTION WHEN APPLYING HEAT THERAPY TO WHICH OF THE FOLLOWING PATIENTS: A UNCONSCIOUS B HIGH PAIN SENSITIVITY C VENOUS ULCER D RECEIVING STEROIDS

A

The nurse has delegated administration of 10am medications to an LPN/LVN. At 10:15am, the nurse notes none of the medications have been administer yet. Which is the best action for the nurse to take? a) ask another LPN/LVN assigned to the unit to help administer medications b) begin administering the medications c) report he situation to the head nurse d) ask the LPN/LVN to give the nurse a status report

A

The nurse is caring for a patient after major abdominal surgery. Which of the following demonstrates correct understanding in regard to wound dehiscence? A) The nurse should be alert for an increase in serosanguineous drainage from the wound. B) Wound dehiscence is most likely to occur during the first 24 to 48 hours after surgery. C) The nurse should administer cough suppressant to prevent wound dehiscence. D) The condition is an emergency that requires surgical repair.

A

UPON RESPONDING TO A PATIENTS CALL BELL, THE NURSE DISCOVERS THAT THE PATIENT'S WOUND HAS DEHISCED. INITIAL NURSING MANAGEMENT INCLUDES WHICH OF THE FOLLOWING A COVERING THE WOUND AREA WITH STERILE TOWELS MOISTENED WITH STERILE 0.9% SALINE B CLOSING WOUND WITH STERI STRIPS C HOLDING WOULD TOGETHER AND COVER WITH BLANKET D POURING H202 INTO ABDOMINAL CAVITY AND PACKING WITH GAUZE

A

When teaching a patient about wound healing, the nurse should tell the patient: A) Inadequate nutrition delays wound healing and increases risk of infection. B) Chronic wounds heal more efficiently in a dry, open environment, so leave them open to air when possible. C) Long-term steroid therapy diminishes the inflammatory response and speeds wound healing. D) Fat tissue heals more readily because there is less vascularization.

A

NEVER give K+ via:

IV push

41. A patient with a chronic productive cough and weight loss is receiving a tuberculosis skin test and asks the nurse the reason for the test. Which response should the nurse give? a. The skin test will determine if you have a tuberculosis infection. b. The skin test will indicate whether you have active tuberculosis. c. The skin test is used to decide which antibiotic therapy will work best. d. The skin test is done prior to notification of the public health department.

A Rationale: A positive skin test will indicate whether the patient has been infected with tuberculosis. It does not indicate active infection, which will be established through chest x-ray and sputum culture. Initial drug treatment with 4 antibiotics uses a standardized protocol. Although the public health department should be notified if the patient has TB, the nurse should focus on the patient, rather than on the public health concerns. Cognitive Level: Application Text Reference: p. 571 Nursing Process: Implementation NCLEX: Physiological Integrity

18. After 2 months of TB treatment with a standard four-drug regimen, a patient continues to have positive sputum smears for acid-fast bacilli (AFB). The nurse discusses the treatment regimen with the patient with the knowledge that a. directly observed therapy (DOT) will be necessary if the medications have not been taken correctly. b. the positive sputum smears indicate that the patient is experiencing toxic reactions to the medications. c. twice-weekly administration may be used to improve compliance with the treatment regimen. d. a regimen using only INH and rifampin (Rifadin) will be used for the last 4 months of drug therapy.

A Rationale: After 2 months of therapy, negative sputum smears would be expected if the TB bacillus is susceptible to the medications and if the medications have been taken correctly. The nurse will need to initiate DOT if the patient has not been consistently taking the medications. Toxic reactions to the medications would not result in a positive sputum smear. Twice-weekly medication administration is not one of the options for therapy. INH and rifampin are used for the last 4 months of drug therapy only if the initial four-drug regimen has been effective as evidenced by negative sputum smears. Cognitive Level: Application Text Reference: pp. 571-572 Nursing Process: Implementation NCLEX: Physiological Integrity

31. The emergency department nurse will suspect a tension pneumothorax in a patient who has been in an automobile accident if a. the breath sounds on one side are decreased. b. there are wheezes audible throughout both lungs. c. there is a sucking sound with each patient breath. d. paradoxic movement of the chest is noted.

A Rationale: Breath sounds are decreased on the affected side with tension pneumothorax because air trapped in the pleural space compresses the lung on that side. Wheezes that are heard in both lungs indicate airway narrowing, but not pneumothorax. A sucking sound with inspiration is heard with an open pneumothorax. Paradoxic chest movement is associated with flail chest. Cognitive Level: Application Text Reference: p. 586 Nursing Process: Assessment NCLEX: Physiological Integrity

32. A 19-year-old male patient with CF and his wife are considering having a child. Which statement by the patient indicates that the nurse's teaching has been effective? a. "We will plan on having genetic counseling before we make a decision." b. "My erectile dysfunction will make it more difficult to have a child." c. "It is likely that I will die before any children we have are grown." d. "There should not be any problems as long as I take my medications."

A Rationale: Children of patients with CF are either CF carriers or have the disease. Most men with CF are sterile, but erectile dysfunction is not associated with CF. The life expectancy for CF is getting longer, with a mean age of 35. Despite appropriate therapy, the couple is likely to experience problems becoming pregnant. Cognitive Level: Application Text Reference: p. 659 Nursing Process: Evaluation NCLEX: Health Promotion and Maintenance

4. To promote airway clearance in a patient with pneumonia, the nurse instructs the patient to a. splint the chest when coughing. b. maintain fluid restrictions. c. wear the nasal oxygen cannula. d. try the pursed-lip breathing technique.

A Rationale: Coughing is less painful and more likely to be effective when the patient splints the chest during coughing. Fluids should be encouraged to help liquefy secretions. Nasal oxygen will improve gas exchange in patients with chronic obstructive pulmonary disease (COPD) but will not improve airway clearance in pneumonia. Cognitive Level: Application Text Reference: p. 568 Nursing Process: Implementation NCLEX: Physiological Integrity

2. A patient who was admitted to the hospital with pneumococcal pneumonia has a temperature of 101.6° F with a frequent cough and is complaining of chest pain rated 7 on a 10-point scale with deep inspiration. Which of these ordered medications should the nurse give first? a. Azithromycin (Zithromax) b. Acetaminophen (Tylenol) c. Guaifenesin (Robitussin) d. Codeine phosphate (Codeine)

A Rationale: Early initiation of antibiotic therapy has been demonstrated to reduce mortality. The other medications are also appropriate and should be given as soon as possible, but the priority is to start antibiotic therapy. Cognitive Level: Application Text Reference: pp. 563, 566 Nursing Process: Implementation NCLEX: Physiological Integrity

45. All of the following orders are received for a patient who has just been admitted with probable bacterial pneumonia and sepsis. Which one will the nurse accomplish first? a. Obtain blood cultures from two sites. b. Give ciprofloxin (Cipro) 400 mg IV. c. Send to radiology for chest radiograph. d. Administer aspirin suppository.

A Rationale: Initiating antibiotic therapy rapidly is essential, but it is important that the cultures be obtained before antibiotic administration. The chest radiograph and aspirin administration can be done last. Cognitive Level: Application Text Reference: p. 566 Nursing Process: Implementation NCLEX: Physiological Integrity

11. While teaching a patient with asthma the appropriate use of a peak flow meter, the nurse instructs the patient to a. take and record peak flow readings when having asthma symptoms or an attack. b. increase the doses of long-term control medications for peak flows in the red zone. c. use the flow meter each morning after taking asthma medications. d. empty the lungs and then inhale rapidly through the mouthpiece.

A Rationale: It is recommended that patients check peak flows when asthma symptoms or attacks occur to compare the peak flow with the baseline. Increased doses of rapidly acting 2-agonists are indicated for peak flows in the red zone. Peak flows should be checked every morning before using medications. Peak flows are assessed during rapid exhalation. Cognitive Level: Application Text Reference: pp. 625, 628 Nursing Process: Implementation NCLEX: Physiological Integrity

7. Which finding would be the best indication to the nurse that the patient having an acute asthma attack was responding to the prescribed bronchodilator therapy? a. Wheezes are more easily heard. b. The oxygen saturation is 89%. c. Vesicular breath sounds resolve. d. The respiratory effort decreases.

A Rationale: Louder wheezes indicate that more air is moving through the airways and that the bronchodilator therapy is working. An oxygen saturation level less than 90% indicates continued hypoxemia. Vesicular breath sounds are normal. A decreased respiratory effort may indicate that the patient is becoming too fatigued to breathe effectively and needs mechanical ventilation. Cognitive Level: Application Text Reference: p. 617 Nursing Process: Evaluation NCLEX: Physiological Integrity

16. When teaching the patient who is receiving standard multidrug therapy for TB about possible toxic effects of the antitubercular medications, the nurse will give instructions to notify the health care provider if the patient develops a. yellow-tinged skin. b. changes in hearing. c. orange-colored urine. d. thickening of the nails.

A Rationale: Noninfectious hepatitis is a toxic effect of INH, rifampin, and pyrazinamide, and patients who develop hepatotoxicity will need to use other medications. Changes in hearing and nail thickening are not expected with the four medications used for initial drug therapy. Orange discoloration of body fluids is an expected side effect of rifampin and not an indication to call the health care provider. Cognitive Level: Application Text Reference: pp. 572-573 Nursing Process: Implementation NCLEX: Physiological Integrity

27. A patient with COPD is admitted to the hospital. How can the nurse best position the patient to improve gas exchange? a. Sitting up at the bedside in a chair and leaning slightly forward b. Resting in bed with the head elevated to 45 to 60 degrees c. In the Trendelenburg's position with several pillows behind the head d. Resting in bed in a high-Fowler's position with the knees flexed

A Rationale: Patients with COPD improve the mechanics of breathing by sitting up in the "tripod" position. Resting in bed with the head elevated would be an alternative position if the patient was confined to bed, but sitting in a chair allows better ventilation. The Trendelenburg's position or sitting upright in bed with the knees flexed would decrease the patient's ability to ventilate well. Cognitive Level: Application Text Reference: p. 634 Nursing Process: Implementation NCLEX: Physiological Integrity

15. A patient is receiving isoniazid (INH) after having a positive tuberculin skin test. Which information will the nurse include in the patient teaching plan? a. "Take vitamin B6 daily to prevent peripheral nerve damage." b. "Read a newspaper daily to check for changes in vision." c. "Schedule an audiometric examination to monitor for hearing loss." d. "Avoid wearing soft contact lenses to avoid orange staining."

A Rationale: Peripheral neurotoxicity associated can be prevented by taking vitamin B6 when being treated with INH. Visual changes, hearing problems, and orange staining are adverse effects of other TB medications. Cognitive Level: Application Text Reference: p. 573 Nursing Process: Planning NCLEX: Physiological Integrity

11. The nurse observes a nursing assistant doing all the following activities when caring for a patient with right lower-lobe pneumonia. The nurse will need to intervene when the nursing assistant a. turns the patient over to the right side. b. splints the patient's chest during coughing. c. elevates the patient's head to 45 degrees. d. assists the patient to get up to the bathroom.

A Rationale: Positioning the patient with the left (or "good" lung) down will improve oxygenation. The other actions are appropriate for a patient with pneumonia. Cognitive Level: Application Text Reference: p. 569 Nursing Process: Implementation NCLEX: Physiological Integrity

24. Postural drainage with percussion and vibration is ordered bid for a patient with chronic bronchitis. The nurse will plan to a. carry out the procedure 3 hours after the patient eats. b. maintain the patient in the lateral positions for 20 minutes. c. perform percussion and vibration before placing the patient in the drainage position. d. give the ordered albuterol (Proventil) after the patient has received the therapy.

A Rationale: Postural drainage, percussion, and vibration should be done 1 hour before or 3 hours after meals. Patients remain in each postural drainage position for 5 to 15 minutes. Percussion and vibration are done after the postural drainage. Bronchodilators are administered before chest physiotherapy. Cognitive Level: Application Text Reference: p. 647 Nursing Process: Planning NCLEX: Physiological Integrity

22. A patient with COPD asks the home health nurse about home oxygen use. The nurse should teach the patient that long-term home O2 therapy a. can improve the patient's prognosis and quality of life. b. may cause oxygen dependency in patients with COPD. c. is used only for patients who have severe end-stage respiratory disease. d. should never be used at night because the patient cannot monitor its effect.

A Rationale: Research supports the use of home oxygen to improve quality of life and prognosis. Oxygen dependency is not an issue for patients with COPD. Although most patients using home oxygen have SpO2 levels less than 89% on room air, it would not be appropriate to tell the patient that he or she was at the end stage of the disease. Oxygen use at night can improve sleep quality and is frequently recommended. Cognitive Level: Application Text Reference: p. 644 Nursing Process: Implementation NCLEX: Physiological Integrity

37. A patient with primary pulmonary hypertension is receiving nifedipine (Procardia). The nurse will evaluate that the treatment is effective if a. the patient reports decreased exertional dyspnea. b. the blood pressure is less than 140/90 mm Hg. c. the heart rate is between 60 and 100 beats/minute. d. the patient's chest x-ray indicates clear lung fields.

A Rationale: Since a major symptom of PPH is exertional dyspnea, an improvement in this symptom would indicate that the medication was effective. Nifedipine will affect BP and heart rate, but these parameters would not be used to monitor effectiveness of therapy for a patient with PPH. The chest x-ray will show clear lung fields even if the therapy is not effective. Cognitive Level: Application Text Reference: p. 601 Nursing Process: Evaluation NCLEX: Physiological Integrity

19. The nurse makes a diagnosis of impaired gas exchange for a patient with COPD in acute respiratory distress, based on the assessment finding of a. a pulse oximetry reading of 86%. b. dyspnea and respiratory rate of 36. c. use of the accessory respiratory muscles. d. the presence of crackles in both lungs.

A Rationale: The best data to support the diagnosis of impaired gas exchange are abnormalities in the ABGs or pulse oximetry. The other data would support a diagnosis of risk for impaired gas exchange. Cognitive Level: Application Text Reference: pp. 650-651 Nursing Process: Diagnosis NCLEX: Physiological Integrity

19. A staff nurse has a TB skin test of 16-mm induration. A chest radiograph is negative, and the nurse has no symptoms of TB. The occupational health nurse will plan on teaching the staff nurse about the a. use and side effects of INH. b. standard four-drug therapy for TB. c. need for annual repeat TB skin testing. d. recommendation guidelines for bacille Calmette-Guérin (BCG) vaccine.

A Rationale: The nurse is considered to have a latent TB infection and should be treated with INH daily for 6 to 9 months. The four-drug therapy would be appropriate if the nurse had active TB. TB skin testing is not done for individuals who have already had a positive skin test. BCG vaccine is not used in the United States and would not be helpful for this individual, who already has a TB infection. Cognitive Level: Application Text Reference: p. 572 Nursing Process: Planning NCLEX: Health Promotion and Maintenance

23. The occupational nurse at a manufacturing plant where there is high worker exposure to beryllium dust will monitor workers for a. shortness of breath. b. chest pain. c. elevated temperature. d. barrel-chest.

A Rationale: The nurse will monitor for the earliest signs of occupational lung disease, which are dyspnea and a cough. The other symptoms are also consistent with occupational lung disease but would occur much later, after significant lung involvement has occurred. Cognitive Level: Application Text Reference: pp. 577-578 Nursing Process: Assessment NCLEX: Health Promotion and Maintenance

47. A patient with a deep vein thrombophlebitis complains of sudden chest pain and difficulty breathing. The nurse finds a heart rate of 142, BP of 100/60, and respirations of 42. The nurse's first action should be to a. elevate the head of the bed. b. administer the ordered pain medication. c. notify the patient's health care provider. d. offer emotional support and reassurance.

A Rationale: The patient has symptoms consistent with a pulmonary embolism; elevating the head of the bed will improve ventilation and gas exchange. The other actions can be accomplished after the head is elevated (and oxygen is started). Cognitive Level: Application Text Reference: p. 599 Nursing Process: Implementation NCLEX: Physiological Integrity

40. A patient who was admitted the previous day with pneumonia complains of a sharp pain "whenever I take a deep breath." Which action will the nurse take next? a. Listen to the patient's lungs. b. Check the patient's O2 saturation. c. Have the patient cough forcefully. d. Notify the patient's health care provider.

A Rationale: The patient's statement indicates that pleurisy or a pleural effusion may have developed and the nurse will need to listen for a pleural friction rub and/or decreased breath sounds. The re is no indication that the oxygen saturation has decreased The patient is unlikely to be able to cough forcefully until pain medication has been administered. The nurse will want to obtain more assessment data before calling the health care provider. Cognitive Level: Application Text Reference: p. 597 Nursing Process: Assessment NCLEX: Physiological Integrity

12. A 32-year-old patient is seen in the clinic for dyspnea associated with the diagnosis of emphysema. The patient denies any history of smoking. The nurse will anticipate teaching the patient about a. 1-antitrypsin testing. b. use of the nicotine patch. c. continuous pulse oximetry. d. effects of leukotriene modifiers.

A Rationale: When emphysema occurs in young patients, especially without a smoking history, a congenital deficiency in 1-antitrypsin should be suspected. Because the patient does not smoke, a nicotine patch would not be ordered. There is no indication that the patient requires continuous pulse oximetry. Leukotriene modifiers would be used in patients with asthma, not with emphysema. Cognitive Level: Application Text Reference: p. 632 Nursing Process: Planning NCLEX: Physiological Integrity

38. A patient with a pleural effusion is scheduled for a thoracentesis. Prior to the procedure, the nurse will plan to a. position the patient sitting upright on the edge of the bed and leaning forward. b. instruct the patient about the importance of incentive spirometer use after the procedure. c. start a peripheral intravenous line to administer the necessary sedative drugs. d. remove the water pitcher and remind the patient not to eat or drink anything for 8 hours.

A Rationale: When the patient is sitting up, fluid accumulates in the pleural space at the lung bases and can more easily be located and removed. The lung will expand after the effusion is removed; incentive spirometry is not needed to assure alveolar expansion. The patient does not usually require sedation for the procedure and there are no restrictions on oral intake, since the patient is not sedated or unconscious. Cognitive Level: Application Text Reference: p. 596 Nursing Process: Planning NCLEX: Physiological Integrity

The charge nurse is making client care assignments for the evening shift. One of the LPNs is a new graduate in orientation. What client would be an appropriate care assignment for this nurse?

A 72-year old client with diverticulitits

A nurse is caring for a 73 y/o male patient with a urethral obstruction related to prostatic enlargement. The nurse is aware this may result in what?

A UTI

Several pregnant clients are waiting to be seen in the triage area of the obstetrical unit. What client should the nurse see first?

A client at 32 weeks' gestation who has preeclampsia and + 3 proteinuria who is returning for evaluation of epigastric pain

Before going for Pulmonary Function Tests (PFTs):

A client's BRONCHODILATORS will be WITHHELD and they are NOT allowed to smoke for 4 hours prior

NREM stage 1

A few minutes, light sleep, easily aroused, gradual reduction in vital signs

O2 Saturation - Nursing Implications

A low saturation level could indicate a respiratory illness or other medical condition. When oxygen saturation levels dip below the normal range, treatment usually involves oxygen therapy. A high concentration of oxygen is delivered by a ventilator through a mask or nasal cannulae (tubes).

You are caring for four patients, all diagnosed with type 1 diabetes. Based upon components of managing diabetes, which patient will likely have the greatest success in maintaining tight glucose control of type 1 diabetes?

A patient who adheres to a meal plan and meal schedule.

Myasthenia Crisis

A positive reaction to the Tensilon test (symptoms IMPROVE)

Urine: Nitrates Nursing Implications - testing

A positive test indicates the need for a culture

The nurse is planning patient teaching for a patient with end-stage renal disease who is scheduled for the creation of a fistula. The nurse would included which of the following in teaching the patient about the fistula?

A vein and an artery in your arm will be attached surgically?

How long before an increased ICP becomes life-threatening?

If ICP is >15-20 for more then 5 minutes then it can become life-threatening

Fat soluble vitamins:

A, D, E, and K

How can you determine a patient's history of allergies? (Select all that apply.) A) By looking at the patient's allergy bracelet B) By looking at the MAR C) By asking the patient D) By looking at the front of the chart E) By administering a dose and monitoring the patient's response

ABCD

All of the following are examples of increased risk for pressure ulcers (select all that apply) A Wheelchair Bound B Peripheral Vascular Disease C Diabetes D Malnourishment E Incontinence

ABCDE

Which of the following patients have risk factors for developing a wound infection? (Select all that apply.) A) An 80-year-old man who has a burn B) A 17-year-old patient who has a metal fragment lodged in his thigh C) A 30-year-old female who had an episiotomy after childbirth D) A patient receiving chemotherapy who has a surgical incision E) A patient with peripheral vascular disease and an ulcer on the heel

ABDE

Lumbar puncture

AFTER the procedure, the client should be placed in the supine position for 4 to 12 hrs as prescribed. (

A little trick regarding potassium:

ALKALOSIS: K is LOW Acidosis is just the opposite: K is High

Graves' Disease/Hyperthyroidism

Accelerated physical and mental function; sensitivity to the heat; fine/soft hair

What is Cranial Nerve XI?

Accessory; controls movements of the neck and shoulders

Medication of choice for CHF:

Ace Inhibitor

A= appearance (color all pink, pink and blue, blue [pale]) P= pulse (>100, < 100, absent) G= grimace (cough, grimace, no response) A= activity (flexed, flaccid, limp) R= respirations (strong cry, weak cry, absent)

APGAR SCORE

Activated Partial Thromboplastin Time - Abbreviation

APPT or aPPT

What is Cranial Nerve VIII?

Acoustic or Vestibulocochlear; senses sound, rotation, and gravity (equilibrium)

What is Cranial Nerve VI?

Abducens; eye movement

Hematocrit - What can alter Values?

Abnormal RBC Size

Hypospadias:

Abnormality in which urethral meatus is located on the ventral (back) surface of the penis anywhere from the corona to the perineum. Remember: HYPO or LOW (for LOWER side or UNDERSIDE)

What does the APGAR score assess?

Activity (muscle tone), Pulses, Grimace (reflex irritability), Appearance (skin color), and Respirations

A patient comes to the walk-in clinic complaining of a lump in her gum and severe pain. What would the nurse expect the diagnosis to be?

Acute pariapical abscess

The nurse notes thick, white respiratory secretions for a patient who is receiving mechanical ventilation. Which intervention will be most effective in resolving this problem?

Add additional water to the patient's enteral feedings. rationale: Because the patient's secretions are thick, better hydration is indicated. Suctioning every hour without any specific evidence for the need will increase the incidence of mucosal trauma and would not address the etiology of the ineffective airway clearance. Instillation of saline does not liquefy secretions and may decrease the SpO2. Repositioning the patient is appropriate but will not decrease the thickness of secretions.

Medication of choice for SVT:

Adenosine/Adenocard

Liver biopsy

Adm vit k , npo morning of exam 6hr, give sedative, Teach pt that he will be asked to hold breath for 5-10sec, supine postion, lateral with upper arms elevated. Post- postion on right side, frequent v.s., report severe ab pain stat, no heavy lifting 1wk.

Meniere's Disease

Admin diuretics to decrease endolymph in the cochlea, restrict Na, lay on affected ear when in bed. Triad: 1)Vertigo 2)Tinnitus 3)N/V

A pt is admitted to the medical unit with a 103.7 F temp. Which intervention would be most effective in restoring normal body temp?

Administer anti pyretics on an around-the-clock schedule.

When using a bronchodilator in conjunction with a glucocorticoid:

Administer the bronchodilator FIRST

RN must do what

Admission history. If someone else does it for you, NEVER sign off on the form until you have validated the data.

Aminoglycocide (__Mycin ; except erythromycine)

Adverse Effects are bean shaped - Nephrotoxic to Kidneys and Ototoxic to Ears

Caclium Channel Blockers:

Affect the AFTERLOAD

Lumbar Puncture

After the procedure, the client should be placed in the supine position for 4-12 hours as prescribed

5 variables that increase surgical risk:

Age, obesity, malnutrition, preoperative dehydration, preoperative infection, the use of anticoagulants preoperatively

SARS (severe acute respiratory syndrome):

Airborne and Contact precautions (same as varicella)

My - Measles Chicken - Chicken Pox/Varicella Hez - Herpez Zoster/Shingles TB Private Room - negative pressure with 6-12 air exchanges/hr Mask, N95 for TB

Airborne precautions

Contraindication to Hep B vaccine:

Allergy to Baker's Yeast (may cause anaphylaxis)

Alzheimer's disease

Alzheimer's disease is a chronic, progressive, degenerative cognitive disorder that accounts for more than 60% of all dementias

What medication is effective in both ventricular and atrial complication?

Amiodarone

Preload effects:

Amount of blood that goes to the RIGHT ventricle

When ASA is given once daily it acts as:

An Antiplatelet

Greek heritage:

An amulet or other object is placed around their baby's neck to avoid "evil eye" or envy of others.

A local public health nurse is informed that a cook in a local restaurant has been diagnoses with hepatitis A. What should the nurse advise individuals who ate at his restaurant and have never received the hepatitis A vaccine?

An immune globulin injection

Manchausen by proxy (MSBP):

An individual, typically a mother, intentionally causes or fabricates illness in a child or other person under her care

puncture wound

An open wound that tears through the skin and destroys underlaying tissues. A penetrating puncture wound can be shallow or deep. A perforating puncture wound has both an entrance and an exit wound.

What physiologic abnormalities indicate the development of chronic renal disease? A.Hypophosphatemia B.Anemia C.Metabolic acidosis D.Respiratory Alkalosis E.Hypercalcemia

Anemia, Metabolic acidosis, Hypercalcemia

White Blood Cell Count - What are some things that might cause a rise in values?

Anestetics, stress, exercize and convulsions

"The nurse is assessing a client who has a current history of alcohol dependence for signs of major withdrawal. What findings would the nurse expect to find?" "1. Tachycardia, severe diaphoresis 2. Hypotension, bradycardia 3. Cold, clammy skin, decreased body temperature 4. Anxiety and increased appetite

Answer 1: "1. Hypotension, bradycardia. Symptoms associated with major withdrawal, also known as delirium tremens (DTs), are not low blood pressure and a slow heartbeat, but hypertension and tachycardia. 2. Cold, clammy skin, decreased body temperature. Severe diaphoresis and elevated body temperature are physical symptoms of impending DTs (delirium tremens). 3. Tachycardia, severe diaphoresis. Tachycardia and severe diaphoresis are associated with major withdrawal from alcohol. 4. Anxiety and increased appetite. Clients experiencing a minor withdrawal from alcohol may experience anxiety and gastrointestinal-related symptoms such as nausea, vomiting, and anorexia."

"Preoperatively, the nurse develops a plan to prepare a 7-month-old infant psychologically for a scheduled herniorrhaphy the next day. Which of the following should the nurse expect to implement to accomplish this goal? 1. Explaining the preoperative and postoperative procedures to the mother. 2. Having the mother stay with the infant. 3. Making sure the infant's favorite toy is available. 4. Allowing the infant to play with surgical equipment.

Answer 2, 2. The best way to prepare a 7-month-old infant psychologically for surgery is to have the primary caretaker stay with the child. Infants in the second 6 months of life commonly develop separation anxiety. Therefore, the priority in this case is to support the child by having the parent present. Teaching the mother what to expect may decrease her anxiety; this is important because infants sense anxiety and distress in parents, but the priority in this case is to have the parent present. Actual play and acting out life experiences are appropriate for preschool-age children. Allowing an infant to play with surgical equipment would be inappropriate and dangerous.

A nurse is monitoring a client who abuses alcohol for signs of alcohol withdrawal. Which of the following would alert the nurse to the potential for withdrawal delirium? "1. Hypotension, ataxia, hunger 2. Stupor, agitation, muscualr rigidity 3. Hypotension, coarse hand tremors, agitaition 4. Hypertension, changes in LOC, hallucinations

Answer 4, Syptoms associated with withdrawal delirium typically include anxiety, insomnia, anorexia, hypertension, disorientation, hallucinations, changes in level of consciousness, agitation, fever, and delusions.

"A client taking the drug disulfiram (Antabuse) is admitted to the ER. Which clinical manifestations are most indicative of recent alcohol ingestion? "A. Vomiting, heart rate 120, chest pain B. Nausea, mild headache, bradycardia C. Respirations 16, heart rate 62, diarrhea D. Temp 101°F, tachycardia, respirations 20"

Answer A is correct. Vomiting, a heart rate of 120, and chest pain are symptoms of drinking alcohol while taking Antabuse. Additional symptoms include severe headache, nausea, ardiac collapse, respiratory collapse, convulsions, and death. Answers B, C, and D contain incomplete or inaccurate clinical signs of the combination of alcohol and Antabuse.

"An order is written to start an IV on a 74-year-old client who is getting ready to go to the operating room for a total hip replacement. What gauge of catheter would best meet the needs of this client? "A) 18 B) 20 C) 21 butterfly D) 25"

Answer A. Clients going to the operating room ideally should have an 18- gauge catheter. This is large enough to handle blood products safely and to allow rapid administration of large amounts of fluid if indicated during the perioperative period. An 18-gauge catheter is recommended. A 20-gauge catheter is a second choice. A 21-gauge needle is too small and a butterfly too unstable for a client going to surgery. A 25-gauge needle is too small.

The nurse is aware that the client requires clarification of preoperative instructions when the client says "a. ""It is OK to drink some juice in the morning."" b. ""I should remove all jewelry before surgery. c. ""I should shower the night before with this soap I was give d. ""I will have an IV put in before surgery."""

Answer A. The client repeats correct information, except that juice is not allowed the morning of surgery. B. All jewelry should be remove C. Patient will be given specific soap to wash with night before D. IV will help adminster meds during surgery

The physician has ordered sequential compression devices (SCDs) for a client going to surgery. The best rationale for use of SCDs is to: A) keep the legs warm B) Promote venous return (C) Maintain balanced fluid volume D) Induce Relaxation

Answer B, Clients who are undergoing surgery may benefit from a sequential compression device (SCD) to promote venous return from the legs. SCDs inflate and deflate plastic sleeves wrapped around the legs to promote venous flow.

"A client is diagnosed with a spontaneous pneumothorax necessitating the insertion of a chest tube. What is the best explanation for the nurse to provide this client? "A. "The tube will drain fluid from your chest." B. "The tube will remove excess air from your chest." C. "The tube controls the amount of air that enters your chest." D. The tube will seal the hole in your lung.""

Answer B. The purpose of the chest tube is to create negative pressure and remove the air that has accumulated in the pleural space.

"Which of the following preoperative patients likely faces the greatest risk of bleeding as a result of their medication? "a. A woman who takes metoprolol (Lopressor) for the treatment of hypertension b. a man whose type 1 diabetes is controlled with insulin injections four times daily c. a man who is taking copidogrel (Plavix) after the placement of a coronary artery stent d. a man who recently started taking finasteride (Proscar) for the treatment of benign prostatic hyperplasia"

Answer C: Any drug that inhibits platelet aggregation, such as clopidogrel (Plavix), represents a bleeding risk. Insulin, metoprolol (Lopressor), and finasteride (Proscar) are less likely to contribute to a risk for bleeding.

During preparation for bowel surgery, a male client receives an antibiotic to reduce intestinal bacteria. Antibiotic therapy may interfere with synthesis of which vitamin and may lead to hypoprothrombinemia? "a. vitamin A b. vitamin D c. vitamin E d. vitamin K

Answer D. Intestinal bacteria synthesize such nutritional substances as vitamin K, thiamine, riboflavin, vitamin B12, folic acid, biotin, and nicotinic acid. Therefore, antibiotic therapy may interfere with synthesis of these substances, including vitamin K. Intestinal bacteria don't synthesize vitamins A, D, or E.

"Which of the following is the primary reason for accurately recording the patient's current medications during a preoperative assessment? "A. Some medications may alter the patient's perceptions about surgery. B. Many anesthetics alter renal and hepatic function, causing toxicity of other drugs. C. Some medications may interact with anesthetics, altering the potency and effect of the drugs. D. Routine medications are withheld the day of surgery, requiring dosage and schedule adjustments after surgery."

Answer, C, Drug interactions may occur between prescribed medications and anesthetic agents used during surgery. For this reason, it is important to take a careful medication history and check that they have been communicated to the anesthesia care provider.

Mucomyst

Antedote for Tylenol OD and is given PO

Bactrim

Antibiotic; don't take if you are allergic to sulfa drugs; diarrhea is a common side effect; encourage increased fluid intake

APGAR

Appearance (color all pink, pink and blue, or blue) Pulse ( > 100, < 100, or absent) Grimace (cough, grimace, no response Activity (flexed, flaccid, limp) Respirations (strong cry, weak cry, absent)

Detached Retina

Area of detachment should be in the dependent position

Guillain Barre Syndrome

Ascending muscle weakness

Guillain-Barre syndrme:

Ascending paralysis--keep an eye on the respiratory system

Potal hypotension + albuminemia =

Ascites

While family members are visiting, a patient has a cardiac arrest and is being resuscitated. Which action by the nurse is best?

Ask family members if they wish to remain in the room during the resuscitation. rationale: Research indicates that family members want the option of remaining in the room during procedures such as CPR and that this decreases anxiety and facilitates grieving. The other options may be appropriate if the family decides not to remain with the patient.

Which type of communication skills is neccessary to implement a democratic leadership style?

Assertive communication skills

Assessing extraocular eye movement:

Assess cranial nerves: 3, 4, and 6

Latex allergies:

Assess for allergies to bananas, apricots, cherries, grapes, kiwis, passion fruit, avocados, chestnuts, tomatoes, and peaches.

Prior to giving the Flu Shot:

Assess for allergy to eggs

Prior to given the MMR:

Assess for allergy to eggs or neomycin

Which action should the nurse take when the low pressure alarm sounds for a patient who has an arterial line in the right radial artery?

Assess for cardiac dysrhythmias. rationale: The low pressure alarm indicates a drop in the patient's blood pressure, which may be caused by cardiac dysrhythmias. There is no indication to rezero the equipment. Pallor of the right hand would be caused by occlusion of the radial artery by the arterial catheter, not by low pressure. There is no indication of a need for flushing the line.

Prior to giving Asparginase:

Assess for hypersensitivity

MRI:

Assess for: Claustrophobia, metal objects, pacemaker

IV push is given over:

At least 2 mins

why could we not assign a client with abdominal pain scheduled for a CT scan

At this point client has not been diagnosed whether stable or unstable

To DECREASE secretions:

Atropine

For CHRONIC PAIN:

Attempt GUIDED IMAGERY or HEAT therapy

The nurse notes that a client who has undergone skin, tissue, and muscle grafting following a modified radical neck dissection requires suctioning. What is the most important consideration for the nurse when suctioning this client?

Avoid applying suction on or near the graft site

The nurse has implemented a bladder retraining program in a 65 y/o woman after the removal of an indwelling urinary catheter from this patient. The nurse places the patient on a timed voiding schedule and performs an ultrasonic bladder scan after each void. The nurse notes that the patient has 50 mL of urine remaining in her bladder after voiding. What would be the nurse's best response to this finding?

Avoid further interventions at this time, as this is an acceptable finding.

A patient with AIDS is being prepared for discharge. The nurse caring for the patient with AIDS knows the patient receives Agenerase. What dietary counseling will the nurse provide based upon the patient's medication regimen?

Avoid high-fat meals while taking this medication

When taking Digoxin and K+ supplements:

Avoid salt substitutes--as they contain potassium

A nursing student is writing a teaching plan for a patient with venous insufficiency. The student covers measures to prevent complications from venous insufficiency. What is one measure the student should include in the plan?

Avoiding tight-fitting socks

Decerebrate:

Away from the cord

DURING A SKIN ASSESSMENT, THE NURSE RECOGNIZES THE 1ST INDICATION THAT A PRESSURE ULCER MAY BE DEVELOPING WHEN SHE NOTICES THE SKIN IS WHICH COLOR? A BLUE B WHITE C YELLOW D RED

B

PATIENT TELLS NURSE "I CANT GET ANY SLEEP AROUND HERE" NURSES FIRST RESPONSE: A ADD MORE CARBS TO DINNER B ASSESS FACTORS THAT PATIENT BELIEVES TO BE PROBLEM C TEACH PATIENT RELAXATION TECHNIQUES AND REDUCE NOISE ON THE UNIT D OBTAIN PRN ODER FOR SEDATIVE

B

THE NURSE WOULD RECOGNIZE WHICH OF THE FOLLOWING PATIENTS TO HAVE IMPAIRED WOUND HEALING A NPO FOLLOWING SURGERY B OBESE WOMAN WITH TYPE 1 DIABETES C MAN WITH SEDENTARY LIFESTYLE AND LIFELONG SMOKER D A WOMAN WHO'S BREAST RECONSTRUCTION SURGERY REQUIRED NUMEROUS INCISION

B

Diverticulitis (inflammation of the diverticulum in the colon); pain is located in the:

LLQ

Diverticulitis:

LLQ pain; treat with low residue, no seeds, nuts, or peas; monitor for peritonitis

Urine: Leukocyte esterase - Nursing Implications - general

False-positive results may occur in specimens contaminated by vaginal secretions that contain WBC's (heavy menstrual discharge, trichomonas infection, and paracites)

The nurse is caring for a patient scheduled for an infusion drip pyelography. What patient preparation is necessary for this test?

Fluids are not restricted.

39. When taking an admission history of a patient with possible asthma who has new-onset wheezing and shortness of breath, the nurse will be most concerned about which information? a. The patient has a history of pneumonia 2 years ago. b. The patient takes propranolol (Inderal) for hypertension. c. The patient uses acetaminophen (Tylenol) for headaches. d. The patient has chronic inflammatory bowel disease.

B Rationale: -blockers such as propranolol can cause bronchospasm in some patients. The other information will be documented in the health history but does not indicate a need for a change in therapy. Cognitive Level: Application Text Reference: p. 610 Nursing Process: Assessment NCLEX: Physiological Integrity

22. When caring for a patient who is hospitalized with active TB, the nurse observes a family member who is visiting the patient. The nurse will need to intervene if the family member a. washes the hands before entering the patient's room. b. puts on a surgical face mask before visiting the patient. c. brings food from a "fast-food" restaurant to the patient. d. hands the patient a tissue from the box at the bedside.

B Rationale: A high-efficiency particulate-absorbing (HEPA) mask, rather than a standard surgical mask, should be used when entering the patient's room because the HEPA mask can filter out 100% of small airborne particles. Handwashing before visiting the patient is not necessary, but there is no reason for the nurse to stop the family member from doing this. Because anorexia and weight loss are frequent problems in patients with TB, bringing food from outside the hospital is appropriate. The family member should wash the hands after handling a tissue that the patient has used, but no precautions are necessary when giving the patient an unused tissue. Cognitive Level: Application Text Reference: p. 574 Nursing Process: Implementation NCLEX: Physiological Integrity

32. The nurse identifies a nursing diagnosis of ineffective airway clearance for a patient who has incisional pain, a poor cough effort, and scattered rhonchi after having a pneumonectomy. To promote airway clearance, the nurse's first action should be to a. have the patient use the incentive spirometer. b. medicate the patient with the ordered morphine. c. splint the patient's chest during coughing. d. assist the patient to sit up at the bedside.

B Rationale: A major reason for atelectasis and poor airway clearance in patients after chest surgery is incisional pain (which increases with deep breathing and coughing). The first action by the nurse should be to medicate the patient to minimize incisional pain. The other actions are all appropriate ways to improve airway clearance but should be done after the morphine is given. Cognitive Level: Application Text Reference: pp. 591, 594 Nursing Process: Implementation NCLEX: Physiological Integrity

33. The nurse caring for a patient with CF recognizes that the manifestations of the disease are caused by the pathophysiologic processes of a. inflammation and fibrosis of lung tissue. b. altered function of exocrine glands. c. failure of the mucus-producing goblet cells. d. thickening and fibrosis of the pleural linings.

B Rationale: CF is characterized by abnormal secretions of exocrine glands, mainly of the lungs, pancreas, and sweat glands. Damage to lung tissue develops late in the disease. The goblet cells continue to produce mucus. Cognitive Level: Comprehension Text Reference: pp. 655-656 Nursing Process: Assessment NCLEX: Physiological Integrity

35. A 68-year-old man has a long history of COPD and is admitted to the hospital with cor pulmonale. Which clinical manifestation noted by the nurse is consistent with the cor pulmonale diagnosis? a. Audible crackles at both lung bases b. 3+ edema in the lower extremities c. Loud murmur at the mitral area d. High systemic BP

B Rationale: Cor pulmonale is right ventricular failure caused by pulmonary hypertension, so clinical manifestations of right ventricular failure such as peripheral edema, jugular vein distension, and right upper-quadrant abdominal tenderness would be expected. Lung crackles, a murmur, and numbness and tingling are not caused by cor pulmonale. Cognitive Level: Application Text Reference: p. 602 Nursing Process: Assessment NCLEX: Physiological Integrity

8. A patient who has mild persistent asthma uses an albuterol (Proventil) inhaler for chest tightness and wheezing has a new prescription for cromolyn (Intal). To increase the patient's management and control of the asthma, the nurse should teach the patient to a. use the cromolyn when the albuterol does not relieve symptoms. b. use the cromolyn to prevent inflammatory airway changes. c. administer the cromolyn first for chest tightness or wheezing. d. administer the albuterol regularly to prevent airway inflammation.

B Rationale: Cromolyn is prescribed to reduce airway inflammation. It takes several weeks for maximal effect and is not used to treat acute asthma symptoms Albuterol is used as a rescue medication in mild persistent asthma and will not decrease inflammation. Cognitive Level: Application Text Reference: p. 620 Nursing Process: Implementation NCLEX: Physiological Integrity

16. The nurse identifies the nursing diagnosis of imbalanced nutrition: less than body requirements for a patient with COPD. An appropriate intervention for this problem is to a. have the patient exercise for 10 minutes before meals. b. offer high calorie snacks between meals and at bedtime. c. assist the patient in choosing foods with a lot of texture. d. increase the patient's intake of fruits and fruit juices.

B Rationale: Eating small amounts more frequently (as occurs with snacking) will increase caloric intake by decreasing the fatigue and feelings of fullness associated with large meals. Patients with COPD should rest before meals. Foods that have a lot of texture may take more energy to eat and lead to decreased intake. Although fruits and juices are not contraindicated, foods high in protein are a better choice. Cognitive Level: Application Text Reference: pp. 649,652 Nursing Process: Planning NCLEX: Physiological Integrity

35. Which statement by the COPD patient indicates that the nurse's teaching about nutrition has been effective? a. "I will drink lots of fluids with my meals." b. "I will have ice cream as a snack every day." c. "I should exercise for 15 minutes before meals." d. "I should avoid much meat or dairy products."

B Rationale: High-calorie foods like ice cream are an appropriate snack for patients with COPD. Fluid intake of 3 L/day is recommended, but fluids should be taken between meals rather than with meals to improve oral intake of solid foods. The patient should avoid exercise for an hour before meals to prevent fatigue while eating. Meat and dairy products are high in protein and are good choices for the patient with COPD. Cognitive Level: Application Text Reference: p. 649 Nursing Process: Evaluation NCLEX: Physiological Integrity

38. Which information given by an asthmatic patient during the admission assessment will be of most concern to the nurse? a. The patient says that the asthma symptoms are worse every spring. b. The patient's only asthma medications are albuterol (Proventil) and salmeterol (Serevent). c. The patient uses cromolyn (Intal) before any aerobic exercise. d. The patient's heart rate increases after using the albuterol (Proventil) inhaler.

B Rationale: Long-acting 2-agonists should be used only in patients who are also using another medication for long-term control (typically an inhaled corticosteroid). Salmeterol should not be used as the first-line therapy for long-term control. The other information given by the patient requires further assessment by the nurse but is not unusual for a patient with asthma. Cognitive Level: Application Text Reference: pp. 615, 621 Nursing Process: Assessment NCLEX: Physiological Integrity

25. A lobectomy is scheduled for a patient with stage I non-small cell lung cancer. The patient tells the nurse, "I would rather have radiation than surgery." Which response by the nurse is most appropriate? a. "Are you afraid that the surgery will be very painful?" b. "Tell me what you know about the various treatments available." c. "Surgery is the treatment of choice for stage I lung cancer." d. "Did you have bad experiences with previous surgeries?"

B Rationale: More assessment of the patient's concerns about surgery is indicated; an open-ended response will elicit the most information from the patient. The answer beginning, "Surgery is the treatment of choice" is accurate, but it discourages the patient from sharing concerns about surgery. The remaining two answers indicate that the nurse has jumped to conclusions about the patient's reasons for not wanting surgery. Cognitive Level: Application Text Reference: pp. 583-584 Nursing Process: Implementation NCLEX: Psychosocial Integrity

4. The health care provider has prescribed triamcinolone (Azmacort) metered-dose inhaler (MDI) two puffs every 8 hours and pirbuterol (Maxair) MDI 2 puffs four times a day for a patient with asthma. In teaching the patient about the use of the inhalers, the best instruction by the nurse is a. "Use the Maxair inhaler first, wait a few minutes, then use the Azmacort inhaler." b. "Using a spacer with the MDIs will improve the inhalation of the medications." c. "To avoid side effects, the inhalers should not be used within 1 hour of each other." d. "To maximize the effectiveness of the drugs, inhale quickly when using the inhalers."

B Rationale: More medication reaches the bronchioles when a spacer is used along with an MDI. There is no evidence that using a bronchodilator before a corticosteroid inhaler is helpful. The medications can be used at the same time. The patient should inhale slowly when using an MDI. Cognitive Level: Application Text Reference: p. 621 Nursing Process: Implementation NCLEX: Physiological Integrity

7. The nurse notes new-onset confusion in an 89-year-old patient in a long-term-care facility; the patient is normally alert and oriented. Which action should the nurse take next? a. Check the patient's pulse rate. b. Obtain an oxygen saturation. c. Notify the health care provider. d. Document the change.

B Rationale: New-onset confusion caused by hypoxia may be the first sign of pneumonia in older patients. The other actions are also appropriate in this order: check the pulse, notify the health care provider, and document the change in status. Cognitive Level: Application Text Reference: p. 565 Nursing Process: Implementation NCLEX: Physiological Integrity

3. During assessment of the chest in a patient with pneumococcal pneumonia, the nurse would expect to find a. hyperresonance on percussion. b. increased vocal fremitus on palpation. c. fine crackles in all lobes on auscultation. d. asymmetric chest expansion on inspection.

B Rationale: Pneumonias caused by Streptococcus pneumoniae are typically lobar or segmental. The nurse would expect to find increased vocal fremitus over the affected area of the lungs. The area would be dull to percussion. Fine crackles in all lobes would indicate a diffuse infection, which is more typical of viral pneumonias. Asymmetric chest expansion is not typical with pneumonia. Cognitive Level: Application Text Reference: p. 565 Nursing Process: Assessment NCLEX: Physiological Integrity

25. When developing a teaching plan to help increase activity tolerance at home for a 70-year-old with severe COPD, the nurse should teach the patient that an appropriate exercise goal is to a. exercise until shortness of breath occurs. b. walk for a total of 20 minutes daily. c. limit exercise to activities of daily living (ADLs). d. walk until pulse rate exceeds 150.

B Rationale: The goal for exercise programs for patients with COPD is to increase exercise time gradually to a total of 20 minutes daily. Shortness of breath is normal with exercise and not an indication that the patient should stop. Limiting exercise to ADLs will not improve the patient's exercise tolerance. A 70-year-old patient should have a pulse rate of 120 or less with exercise (80% of the maximal heart rate of 150). Cognitive Level: Application Text Reference: p. 653 Nursing Process: Planning NCLEX: Physiological Integrity

36. The nurse is caring for a patient with primary pulmonary hypertension who is receiving epoprostenol (Flolan). Which assessment information requires the most immediate action? a. The international normalized ratio (INR) is prolonged. b. The central line is disconnected. c. The oxygen saturation is 90%. d. The BP is 88/56.

B Rationale: The half-life of this drug is 6 minutes, so the nurse will need to restart the infusion as soon as possible to prevent rapid clinical deterioration. The other data also indicate a need for ongoing monitoring or intervention, but the priority action is to reconnect the infusion. Cognitive Level: Application Text Reference: p. 601 Nursing Process: Assessment NCLEX: Physiological Integrity

10. A patient with an acute attack of asthma comes to the emergency department, where ABGs are drawn. The nurse determines the patient is in the early phase of the attack, based on the ABG results of a. pH 7.0, PaCO2 50 mm Hg, and PaO2 74 mm Hg. b. pH 7.4, PaCO2 32 mm Hg, and PaO2 70 mm Hg. c. pH 7.36, PaCO2 40 mm Hg, and PaO2 80 mm Hg. d. pH 7.32, PaCO2 58 mm Hg, and PaO2 60 mm Hg.

B Rationale: The initial response to hypoxemia caused by airway narrowing in a patient having an acute asthma attack is an increase in respiratory rate, which causes a drop in PaCO2. The other PaCO2 levels are normal or elevated, which would indicate that the attack was progressing and that the patient is decompensating. Cognitive Level: Application Text Reference: pp. 614, 626 Nursing Process: Assessment NCLEX: Physiological Integrity

29. The health carre provider inserts two chest tubes connected with a Y-connecter in a patient with a hemopneumothorax. When monitoring the patient after the chest tube placement, the nurse will be most concerned about a. a large air leak in the water-seal chamber. b. 400 ml of blood in the collection chamber. c. severe pain with each deep patient inspiration. d. subcutaneous emphysema at the insertion site.

B Rationale: The large amount of blood may indicate that the patient is in danger of developing hypovolemic shock. A large air leak would be expected immediately after chest tube placement for pneumothorax. The severe pain should be treated but is not as urgent a concern as the possibility of continued hemorrhage. Subcutaneous emphysema should be monitored but is not unusual in a patient with pneumothorax. Cognitive Level: Application Text Reference: p. 586 Nursing Process: Assessment NCLEX: Physiological Integrity

42. All of the following information is obtained by the nurse who is caring for a patient receiving subcutaneous heparin injections to treat a pulmonary embolus. Which assessment data is most important to communicate to the health care provider? a. The patient has many abdominal bruises. b. The patient's BP is 90/46. c. The activated partial thromboplastin time is 2 times the patient baseline. d. The patient's stool is dark green and liquid.

B Rationale: The low BP may indicate that the patient is experiencing bleeding, a possible adverse effect of heparin therapy. Subcutaneous heparin administration is given into the subcutaneous tissue of the abdomen and abdominal bruising is not unusual. An aPTT 2 times the baseline indicates a therapeutic heparin level. The patient should be monitored for gastrointestinal bleeding, which would be indicated by black or red stools. Cognitive Level: Application Text Reference: p. 600 Nursing Process: Assessment NCLEX: Physiological Integrity

27. A patient with newly diagnosed lung cancer tells the nurse, "I think I am going to die pretty soon, maybe this week." The best response by the nurse is a. "Are you afraid that the treatment for your cancer will not be effective?" b. "Can you tell me what it is that makes you think you will die so soon?" c. "Would you like to talk to the hospital chaplain about your feelings?" d. "Do you think that taking an antidepressant medication would be helpful?"

B Rationale: The nurse's initial response should be to collect more assessment data about the patient's statement. The answer beginning "Can you tell me what it is" is the most open-ended question and will offer the best opportunity for obtaining more data. The answer beginning, "Are you afraid" implies that the patient thinks that the cancer will be immediately fatal, although the patient's statement may not be related to the cancer diagnosis. The remaining two answers offer interventions that may be helpful to the patient, but more assessment is needed to determine whether these interventions are appropriate. Cognitive Level: Application Text Reference: pp. 583-584 Nursing Process: Implementation NCLEX: Psychosocial Integrity

13. A patient who has active TB has just been started on drug therapy for TB. The nurse informs the patient that the disease can be transmitted to others until a. the chest x-ray shows resolution of the tuberculosis. b. three sputum smears for acid-fast bacilli are negative. c. TB medications have been taken for 6 months. d. sputum cultures on 3 consecutive days are negative.

B Rationale: The patient is considered infectious until three sputum smears are negative for acid-fast bacilli. Chest x-rays help to determine the presence of active TB but are not utilized to monitor the effectiveness of treatment. Taking the medications for 6 months is necessary, but the multidrug-resistant forms of the disease might not be eradicated after 6 months of therapy. Sputum cultures are used to diagnose the presence of active TB, but sputum smears are usually done to establish that treatment has been effective. Cognitive Level: Application Text Reference: p. 574 Nursing Process: Implementation NCLEX: Health Promotion and Maintenance

9. To protect susceptible patients in the hospital from aspiration pneumonia, the nurse will plan to a. turn and reposition immobile patients at least every 2 hours. b. position patients with altered consciousness in lateral positions. c. monitor frequently for respiratory symptoms in patients who are immunosuppressed. d. provide for continuous subglottic aspiration in patients receiving enteral feedings.

B Rationale: The risk for aspiration is decreased when patients with a decreased level of consciousness are placed in a side-lying or upright position. Frequent turning prevents pooling of secretions in immobilized patients but will not decrease the risk for aspiration in patients at risk. Monitoring of parameters such as breath sounds and oxygen saturation will help detect pneumonias in immune compromised patients, but it will not decrease the risk for aspiration. Continuous subglottic suction is recommended for intubated patients but not for all patients receiving enteral feedings. Cognitive Level: Application Text Reference: p. 567 Nursing Process: Planning NCLEX: Safe and Effective Care Environment

13. When teaching a patient with chronic obstructive pulmonary disease (COPD) about reasons to quit smoking, the nurse will explain that long-term exposure to tobacco smoke leads to a a. weakening of the smooth muscle lining the airways. b. decrease in the area available for oxygen absorption. c. lesser number of red blood cells for oxygen delivery. d. decreased production of protective respiratory secretions.

B Rationale: Tobacco smoke leads to an increase in proteolytic enzymes, which break down alveolar walls and lead to less alveolar surface area for gas exchange. Bronchial smooth muscle is not weakened by chronic smoking. Polycythemia is a common compensatory mechanism for patients with COPD. The quantity of respiratory secretions increases as a result of smoking. Cognitive Level: Application Text Reference: p. 633 Nursing Process: Implementation NCLEX: Physiological Integrity

36. When teaching the patient with COPD about exercise, which information should the nurse include? a. "Stop exercising if you start to feel short of breath." b. "Use the bronchodilator before you start to exercise." c. "Breathe in and out through the mouth while you exercise." d. "Upper body exercise should be avoided to prevent dyspnea."

B Rationale: Use of a bronchodilator before exercise improves airflow for some patients and is recommended. Shortness of breath is normal with exercise and not a reason to stop. Patients should be taught to breathe in through the nose and out through the mouth (using a pursed-lip technique). Upper-body exercise can improve the mechanics of breathing in patients with COPD. Cognitive Level: Application Text Reference: p. 653 Nursing Process: Implementation NCLEX: Physiological Integrity

WHEN ADMINISTERING ORAL MEDICATIONS, WHICH OF THE FOLLOWING PRACTICES SHOULD THE NURSE FOLLOW(SELECT ALL THAT APPLYS) A DISPENSE MULTIPLE LIQUID MEDICATIONS INTO A SINGLE CUP TO REDUCE THE NUMBER OF CONTAINERS THE PATIENT MUST HANDLE B PERFORM HAND HYGIENE BEFORE AND AFTER MEDICATION ADMINISTRATION C STAY AT THE BEDSIDE UNTIL THE PATIENT HAS FINISHED ALL MEDICATIONS D KEEP THE PATIENTS MAR AT THE BEDSTIME AT ALL TIMES E VERIFY THE PATIENTS RESPONSE TO THE MEDICATION 30 MINUTES AFTER ADMINISTRATION, OR AS APPROPRIATE FOR THE DRUG

B C E

Lydia is scheduled for elective splenectomy. Before the clients goes to surgery, the nurse in charge final assessment would be: "a. signed consent b. vital signs c. name band d. empty bladder"

B, VS: An elective procedure is scheduled in advance so that all preparations can be completed ahead of time. The vital signs are the final check that must be completed before the client leaves the room so that continuity of care and assessment is provided for.

2. One method of developing a theory is by first examining a general idea and then considering specific actions or ideas. What is the method called? a. Inductive reasoning b. Deductive reasoning c. Conceptual modeling d. Conceptive development

B. Moving from a general idea to specific ideas is deductive reasoning.

When are cultures obtained:

BEFORE starting IV antibiotics

Dopamine increases:

BP

Urea Nitrogen - abbreviation

BUN

Dystocia:

Baby cannot make it down the canal

Source of potassium:

Bananas, potatoes, and citrus fruits

In COPD:

Baroreceptors that detect the CO2 level are destroyed.

Emphysema

Barrel chest

GERD

Barrett's Esophagus (erosion of the lower portion of the esophageal mucosa)

Give Carafate (GI med):

Before meals to coat the stomach

What is the first priority when a client with an unwitnessed cardiac arrest is found?

Begin CPR

A 48 y/o male presents to the ED c/o severe substernal chest pain radiating down his left arm. He's admitted to the CCU with a diagnosis of MI. What nursing assessment activity is priority on admission to CCU?

Begin ECG monitoring

10-11 months:

Belly to butt

You are caring for a patient with biliary colic and are aware that the patient may experience referred abdominal pain. When you assess for referred pain on this patient what is the common location you would assess for referred pain r/t the biliary colic?

Below the right nipple

Low back pain:

Bend knees to relieve pain

Small and Frequent meals are:

Better than Larger meals

A patient returns to the floor after a laparoscopic cholecystectomy. The nurse caring for the patient is aware that the most serious potential complication is what?

Bile duct injury

Hirschsprung's:

Bile is lower obstruction; no bile is upper obstruction; produces ribbon-like stools

Long-term use of amphogel:

Binds to phosphates; increases Ca and robs the bones... leads to increased Ca absorption from bones and WEAKENS BONES!

Thrombocytopenia

Bleeding precautions! 1)Soft bristled toothbrush 2)No insertion of anything! (c/i suppositories, douche) 26 3)No IM meds as much as possible! *Iron deficiency anemia - easily fatigued 1)Fe PO - give with Vitamin C or on an empty stomach 2)Fe via IM- Inferon via Ztrack

Pulse Check on a infant during CPR:

Brachial

Fat Embolism:

Blood tinged sputum (r/t inflammation), Respiratory alkalosis (r/t tachypnea), hypocalcemia, increased serum lipids, "SNOW STORM" effect on chest x-ray.

Hydrocephalus

Bossing Sign (prominent forehead)

Hypervolemia

Bounding pulse, SOB, dyspnea, rales/crackles, peripheral edema, HTN, urine specific gravity < 1.010, semi-Fowler's

Intal (inhaler used to treat allergy induced asthma) may cause:

Bronchospasms

Addison's Disease

Bronze like skin pigmentation

Prednisone toxicity (cushing's syndrome):

Buffalo hump, moon face, hyperglycemia, hypertension

The nurse is assessing a patient with Graves' disease. What physical characteristics of Graves' disease would the nurse expect to find?

Bulging eyes

Lyme's Disease

Bull's eye rash

Cystitis

Burning on urination

SLE

Butterfly rash

A nurse is performing wound care. Which of the following practices violates surgical asepsis? a. Holding sterile objects above the waist b. Considering a 1″ edge around the sterile field as being contaminated c. Pouring solution onto a sterile field cloth d. Opening the outermost flap of a sterile package away from the body

C

Nurse Kate is changing a dressing and providing wound care. Which activity should she perform first? a. Assess the drainage in the dressing. b. Slowly remove the soiled dressing c. Wash hands thoroughly. d. Put on latex gloves.

C

WHICH MEDICATION WILL DELAY HEALING OF A POST-OP WOUND A LAXATIVE B ANTIHYPERTENSIVE C CORTICOSTEROID D K+ SUPPLEMENT

C

Urine: Glucose - Nursing Implications - general

Glucose is nto excreted by the kidney unless the blood levels exceed approxmiately 180 mg/dl, so can reflect the degree od glucose elevation in the blood.

Which of the following clients would least likely be at risk of developing skin breakdown? a. A client incontinent of urine feces b. A client with chronic nutritional deficiencies c. A client with decreased sensory perception d. A client who is unable to move about and is confined to bed

C

Duchenne's Muscular Dystrophy

Gower's Sign (use of hands to push one's self from the floor)

4-5 months:

Grasps, switch and roll

Perform a amniocentesis before 20 weeks gestation to check for:

Cardiac and pulmonary abnormalities

Rheumatic fever can lead to:

Cardiac valve malfunction

Edema is in the INTERSTITIAL SPACE and NOT in the:

Cardiovascular space

LPN cannot:

Handle Blood

The first step in helping family caregivers is to identify them. They must first be identifed either by health care professionals or self- identified.

Care givers have been referred to as "hidden patients" ... this is a common characteristic of care givers because they put the concern for their loved ones first and ignore their own needs

Retino Blastoma

Cat's eye reflex (grayish discoloration of pupils)

During CBI

Catheter is taped to the thigh so the leg should be kept straight. No other positioning restrictions are present.

40. A patient who is experiencing an acute asthma attack is admitted to the emergency department. The nurse's first action should be to a. determine when the dyspnea started. b. obtain the forced expiratory flow rate. c. listen to the patient's breath sounds. d. ask about inhaled corticosteroid use.

C Rationale: Assessment of the patient's breath sounds will help to determine how effectively the patient is ventilating and whether rapid intubation may be necessary. The length of time the attack has persisted is not as important as determining the patient's status at present. Most patients having an acute attack will be unable to cooperate with a FEV measurement. It is important to know about the medications the patient is using but not as important as assessing the breath sounds. Cognitive Level: Application Text Reference: p. 626 Nursing Process: Assessment NCLEX: Physiological Integrity

5. When preparing a patient with possible asthma for pulmonary function testing, the nurse will teach the patient to a. avoid eating or drinking for 4 hours before the forced expiratory volume in 1 second (FEV1)/forced expiratory volume (FEV) test. b. take oral corticosteroids at least 2 hours before the examination. c. withhold bronchodilators for 6 to 12 hours before the examination. d. use rescue medications immediately before the FEV1/FEV testing.

C Rationale: Bronchodilators are held before pulmonary function testing so that a baseline assessment of airway function can be determined. Testing is repeated after bronchodilator use to determine whether the decrease in lung function is reversible. There is no need for the patient to be NPO. Oral corticosteroids should also be held before the examination and corticosteroids given 2 hours before the examination would be at a high level. Rescue medications (which are bronchodilators) would not be given until after the baseline pulmonary function was assessed. Cognitive Level: Application Text Reference: p. 614 Nursing Process: Planning NCLEX: Physiological Integrity

28. A patient with COPD tells the nurse, "At home, I only have to use an albuterol (Proventil) inhaler. Why did the doctor add an ipratropium (Atrovent) inhaler while I'm in the hospital? The appropriate response by the nurse is a. "Atrovent will dilate the airways and allow the Proventil to penetrate more deeply." b. "Atrovent is being used to decrease airway inflammation and sputum production." c. "Atrovent works differently to dilate the bronchi, and the two drugs together are more effective." d. "Atrovent is a potent bronchodilator and patients need to be hospitalized when receiving it."

C Rationale: Combining bronchodilators improves effectiveness. Atrovent does not have to be used before Proventil, it does not decrease airway inflammation, and it does not require hospitalization. Cognitive Level: Application Text Reference: p. 640 Nursing Process: Implementation NCLEX: Physiological Integrity

33. A patient who has a right-sided chest tube following a thoracotomy has continuous bubbling in the suction-control chamber of the collection device. The most appropriate action by the nurse is to a. document the presence of a large air leak. b. obtain and attach a new collection device. c. notify the health care provider of a possible pneumothorax. d. take no further action with the collection device.

C Rationale: Continuous bubbling is expected in the suction-control chamber and indicates that the suction-control chamber is connected to suction. An air leak would be detected in the water-seal chamber. There is no evidence of pneumothorax. A new collection device is needed when the collection chamber is filled. Cognitive Level: Application Text Reference: p. 591 Nursing Process: Implementation NCLEX: Physiological Integrity

20. When reading the chart for a patient with COPD, the nurse notes that the patient has cor pulmonale. To assess for cor pulmonale, the nurse will monitor the patient for a. elevated temperature. b. complaints of chest pain. c. jugular vein distension. d. clubbing of the fingers.

C Rationale: Cor pulmonale causes clinical manifestations of right ventricular failure, such as jugular vein distension. The other clinical manifestations may occur in the patient with other complications of COPD but are not indicators of cor pulmonale. Cognitive Level: Application Text Reference: pp. 635-636 Nursing Process: Assessment NCLEX: Physiological Integrity

1. A patient with a history of asthma is admitted to the hospital in acute respiratory distress. During assessment of the patient, the nurse would notify the health care provider immediately about a. a pulse oximetry reading of 90%. b. a peak expiratory flow rate of 240 ml/min. c. decreased breath sounds and wheezing. d. a respiratory rate of 26 breaths/min.

C Rationale: Decreased breath sounds and wheezing would indicate that the patient was experiencing an asthma attack, and immediate bronchodilator treatment would be indicated. The other data indicate that the patient needs ongoing monitoring and assessment but do not indicate a need for immediate treatment. Cognitive Level: Application Text Reference: pp. 608, 612, 614 Nursing Process: Assessment NCLEX: Physiological Integrity

17. An alcoholic and homeless patient is diagnosed with active TB. Which intervention by the nurse will be most effective in ensuring adherence with the treatment regimen? a. Giving the patient written instructions about how to take the medications b. Teaching the patient about the high risk for infecting others unless treatment is followed c. Arranging for a daily noontime meal at a community center and give the medication then d. Educating the patient about the long-term impact of TB on health

C Rationale: Directly observed therapy is the most effective means for ensuring compliance with the treatment regimen, and arranging a daily meal will help to ensure that the patient is available to receive the medication. The other nursing interventions may be appropriate for some patients, but are not likely to be as helpful with this patient. Cognitive Level: Application Text Reference: pp. 572, 575 Nursing Process: Implementation NCLEX: Physiological Integrity

46. The nurse has received change-of-shift report about these four patients. Which one will the nurse plan to assess first? a. A 23-year-year-old patient with cystic fibrosis who has pulmonary function testing scheduled in 30 minutes b. A 35-year-old patient who was admitted the previous day with bacterial pneumonia and has a temperature of 100.2° F c. A 46-year-old patient who is complaining of dyspnea after having a thoracentesis an hour previously d. A 77-year-old patient with TB who has four antitubercular medications due in 15 minutes

C Rationale: Dyspnea after a thoracentesis may indicate a pneumothorax or hemothorax and requires immediate evaluation by the nurse. The other patients should also be assessed as soon as possible, but there is no indication that they may need immediate action to prevent clinical deterioration. Cognitive Level: Application Text Reference: p. 596 Nursing Process: Planning NCLEX: Physiological Integrity

44. Which assessment information obtained by the nurse when caring for a patient who has just had a thoracentesis is most important to communicate to the health care provider? a. BP is 150/90. b. Pain level is 5/10 with a deep breath. c. Oxygen saturation is 89%. d. Respiratory rate is 24 when lying flat.

C Rationale: Oxygen saturation would be expected to improve after a thoracentesis; a saturation of 89 indicates that a complication such as pneumothorax may be occurring. The other assessment data also indicate a need for ongoing assessment or intervention, but the low oxygen saturation is the priority. Cognitive Level: Application Text Reference: p. 596 Nursing Process: Assessment NCLEX: Physiological Integrity

21. The nurse is performing TB screening in a clinic that has many patients who have immigrated to the United States. Before doing a TB skin test on a patient, which question is most important for the nurse to ask? a. "How long have you lived in the United States?" b. "Is there any family history of TB?" c. "Have you received the BCG vaccine for TB?" d. "Do you take any over-the-counter (OTC) medications?"

C Rationale: Patients who have received the BCG vaccine will have a positive Mantoux test; another method for screening (such as a chest x-ray) will be used in determining whether the patient has a TB infection. The other information may also be valuable but is not as pertinent to the decision about doing TB skin testing. Cognitive Level: Application Text Reference: p. 572 Nursing Process: Assessment NCLEX: Physiological Integrity

18. The nurse teaches a patient with COPD how to perform pursed-lip breathing, explaining that this technique will assist respiration by a. loosening secretions so that they may be coughed up more easily. b. promoting maximal inhalation for better oxygenation of the lungs. c. preventing airway collapse and air trapping in the lungs during expiration. d. decreasing anxiety by giving the patient control of respiratory patterns.

C Rationale: Pursed-lip breathing increases the airway pressure during the expiratory phase and prevents collapse of the airways, allowing for more complete exhalation. Although loosening of secretions, improving inhalation, and decreasing anxiety are desirable outcomes for the patient with COPD, pursed-lip breathing does not directly impact these. Cognitive Level: Comprehension Text Reference: p. 646 Nursing Process: Implementation NCLEX: Physiological Integrity

3. An asthmatic patient who has a new prescription for Advair Diskus (combined fluticasone and salmeterol) asks the nurse the purpose of using two drugs. The nurse explains that a. Advair is a combination of long-acting and slow-acting bronchodilators. b. the two drugs work together to block the effects of histamine on the bronchioles. c. one drug decreases inflammation, and the other is a bronchodilator. d. the combination of two drugs works more quickly in an acute asthma attack.

C Rationale: Salmeterol is a long-acting bronchodilator, and fluticasone is a corticosteroid; they work together to prevent asthma attacks. Neither medication is an antihistamine. Advair is not used during an acute attack because the medications do not work rapidly. Cognitive Level: Application Text Reference: pp. 621 Nursing Process: Implementation NCLEX: Physiological Integrity

12. A hospitalized patient who may have tuberculosis (TB) has an order for a sputum specimen. When will be the best time for the nurse to collect the specimen? a. After the patient rinses the mouth with mouthwash b. As soon as the order is received from the health care provider c. Right after the patient gets up in the morning d. After the skin test is administered

C Rationale: Sputum specimens are ideally collected in the morning because mucus is likely to accumulate during the night. The patient should rinse the mouth with water; mouthwash may inhibit the growth of the bacilli. There is no need to wait until the tuberculin skin test is administered. Cognitive Level: Application Text Reference: p. 572 Nursing Process: Implementation NCLEX: Physiological Integrity

5. The nurse will anticipate discharge today for which of these patients with community-acquired-pneumonia? a. 24-year-old patient who has had temperatures ranging from 100.6° to 101° F b. 35-year-old patient who has had 600 ml of oral fluids in the last 24 hours c. 50-year-old patient who has an oxygen saturation of 91% on room air d. 72-year-old patient with a pulse of 102 and a blood pressure (BP) of 90/56

C Rationale: The 50-year-old meets the Infectious Diseases Society of America (IDSA) hospital discharge criteria. The other patients do not meet the criteria for discharge. Cognitive Level: Application Text Reference: p. 563 Nursing Process: Planning NCLEX: Physiological Integrity

6. The nurse identifies the nursing diagnosis of activity intolerance for a patient with asthma. A common etiologic factor for this nursing diagnosis in patients with asthma is a. anxiety about dyspnea. b. side effects of medications. c. work of breathing. d. fear of suffocation.

C Rationale: The activity intolerance patients with asthma experience is related to the increased effort needed to breathe when airways are inflamed and narrowed and interventions are focused on decreasing inflammation and bronchoconstriction. The other listed etiologies are not as appropriate for this diagnosis but would be appropriate for diagnoses seen in patients with asthma, such as social isolation, knowledge deficit, and anxiety. Cognitive Level: Application Text Reference: pp. 612, 624 Nursing Process: Diagnosis NCLEX: Physiological Integrity

15. A patient with an acute exacerbation of COPD has the following ABG analysis: pH 7.32, PaO2 58 mm Hg, PaCO2 55 mm Hg, and SaO2 86%. The nurse recognizes these values as evidence of a. normal acid-base balance with hypoxemia. b. normal acid-base balance with hypercapnia. c. respiratory acidosis. d. respiratory alkalosis.

C Rationale: The elevated PaCO2 and low pH indicate respiratory acidosis. The patient is hypoxemic and hypercapnic, but the pH indicates acidosis, not a normal acid-base balance. Cognitive Level: Comprehension Text Reference: p. 650 Nursing Process: Assessment NCLEX: Physiological Integrity

14. Which of these is the best goal for the patient admitted with chronic bronchitis who has a nursing diagnosis of ineffective airway clearance? a. Patient denies having dyspnea. b. Patient's mental status is improved. c. Patient has a productive cough. d. Patient's O2 saturation is 90%.

C Rationale: The goal for the nursing diagnosis of ineffective airway clearance is to maintain a clear airway by coughing effectively. The other goals may be appropriate for the patient with COPD, but they do not address the problem of ineffective airway clearance. Cognitive Level: Application Text Reference: p. 660 Nursing Process: Evaluation NCLEX: Physiological Integrity

26. An hour after a left upper lobectomy, a patient complains of incisional pain at a level 7 out of 10 and has decreased left-sided breath sounds. The pleural drainage system has 100 ml of bloody drainage and a large air leak. Which action should the nurse take first? a. Assist the patient to deep breathe and cough. b. Milk the chest tube gently to remove any clots. c. Medicate the patient with the ordered morphine. d. Notify the surgeon about the large air leak.

C Rationale: The patient is unlikely to take deep breaths or cough until the pain level is lower. A chest tube output of 100 ml is not unusual in the first hour after thoracotomy and would not require milking of the chest tube. An air leak is expected in the initial postoperative period after thoracotomy. Cognitive Level: Application Text Reference: p. 594 Nursing Process: Implementation NCLEX: Physiological Integrity

31. A 26-year-old patient has had CF since birth and has severe lung changes and cor pulmonale as a result of the disease. An appropriate expected outcome is that the patient will a. engage in aerobic exercise without dyspnea. b. be weaned from home oxygen use. c. achieve a realistic attitude toward treatment. d. develop no CF-related complications.

C Rationale: The patient's severe lung disease and cor pulmonale are late complications of CF, and a realistic attitude about what outcome can be expected from treatment is an appropriate outcome. Exercising without dyspnea and weaning from home oxygen therapy are not realistic outcomes for this patient with end-stage disease. The patient already has multiple CF-related complications and is likely to continue to develop complications. Cognitive Level: Application Text Reference: pp. 657-658 Nursing Process: Planning NCLEX: Health Promotion and Maintenance

9. During assessment of a patient with a history of asthma, the nurse notes wheezing and dyspnea. The nurse will anticipate giving medications to reduce a. laryngospasm. b. pulmonary edema. c. airway narrowing. d. alveolar distention.

C Rationale: The symptoms of asthma are caused by inflammation and spasm of the bronchioles, leading to airway narrowing. Treatment for laryngospasm or pulmonary edema would not be appropriate. There are no medications used to treat alveolar distention. Cognitive Level: Comprehension Text Reference: pp. 608, 611-612 Nursing Process: Assessment NCLEX: Physiological Integrity

29. The nurse has completed teaching a patient about MDI use. Which patient statement indicates that further patient teaching is needed? a. "I will shake the MDI each time before giving the medication." b. "I will take a slow, deep breath in after pushing down on the MDI." c. "I will float the canister in water to decide whether I need to get a new MDI." d. "I will attach a spacer to the MDI to make it easier for me to use."

C Rationale: This method is no longer recommended as a means of determining whether the medication needs replacement. The other patient statements are accurate and indicate the patient understands how to use the MDI. Cognitive Level: Application Text Reference: p. 622 Nursing Process: Evaluation NCLEX: Physiological Integrity

Bryant's Traction:

Children < 3 years or < 35 lbs with a Femur Fxr

Bulimia

Chipmunk face

"Mary received AtropineSO4 as a pre-medication 30 minutes ago and is now complaining of dry mouth and her PR is higher, than before the medication was administered. "A. The patient is having an allergic reaction to the drug. B. The patient needs a higher dose of this drug C. This is normal side-effect of AtSO4 D. The patient is anxious about upcoming surgery"

C Atropine lowers body fluid in throat and mouth pre-surgery

5.) Which of the following nursing actionsshould be given highest priority whenadmitting the patient into the operatingroom? A.) Level of consciousness B.) Vital signs C.) Patient identification and correctoperative consent D.) Positioning and skin preparation

C.) Patient identification and correctoperative consent

"A client is to have NPO for at least 12 hrs before surgery that same day. A nurse learns the client has half a glass of orange juice 3hr prior to admission. The nurse should... "A: Report the incident to the nursing supervisor. B: Inform the surgery department. C: Notify the anesthesiologist. D: Reschedule the surgery."

C: Notify the anesthesiologist. Restriction of fluids and food is designed to minimize the potential risk of aspiration and to decrease the risk of postoperative nausea and vomiting. A client who has not followed this instruction may have surgery delayed of cancelled.

Hodgkin's disease:

CA of the lymph that is very curable in the early stages

Hypocalcemia

CATS - convulsions, arrhythmias, tetany, spasms and stridor

Hypoparathyroid

CATS - convulsions, arrhythmias, tetany, spasms, stridor (decreased calcium), high Ca, low phosphorus diet

Hypocalcemia

CATS: Convulsions, arrhythmias, tetany, spasms, and stridor

Hypo-parathyroid

CATS: Convulsions, arrhythmias, tetany, spasms, stridor (decreased calcium); high Ca and low phosphorus diet

(4) options for CA Management:

CHEMO RADIATION SURGERY Allow to die with dignity

Laparoscopy

CO2 used to enhances visual, general anesthesia, foley. Post- walk patient to decrease CO2 build up used for procedure.

Cane walking:

COAL: Cane Opposite Affected Leg

Myxedema coma is:

COLD (hypothermia)

A 20 year old female client who tried lysergic acid diethylamide (LSD) as a teen tells the nurse that she has had bad dreams that make her want to kill herself. Which is the explanation for this occurence? A. These occurences are referred to as hold-over reactions to the drugs. B. These are flashbacks to a time of a "bad trip". C. The drug is still in the body and causing these reactions. D. The client is suicidal and should be on one-to-one precautions.

CORRECT ANSWER: BA. These reactons are called "flashbacks" B. Flashback reactions occur after the use of hallucinogens in which the client relives a bad episode that occurred while using the drug. C. The drug is gone from the body, but the mind-altering effects can occur at any time in the form of memory flashbacks. D. The client stated that the dreams are causing her distress. She is asking for help with the dreams, not planning her suicide.

Creatine Phosphokinase - Abbreviation

CPK

Left off at 35

CPR

Definitive diagnosis for AAA:

CT scan

Calan (verapamil)

Calcium channel blocker; treatment for HTN, angina; assess for constipation

Most spinal cord injuries are at the:

Cervical or lumbar regions

Dilantin

Can NOT be taken during pregnancy

Rubella (German measles)

Can be harmful to the fetus.

Phenobarbitol

Can be taken during pregnancy

SIADH (increased ADH)

Change in LOC, decreased deep tendon reflexes, tachycardia, N/V/D, HA Administer Declomycin and diuretics

Myesthenia Gravis:

Caused by a disorder in the transmission of impulses from nerve to muscle cell.

Cholinergic Crisis

Caused by excessive medication STOP medications Tensilon will make it WORSE

Ampho B:

Causes hypokalemia; must pre-medicate prior to giving and client's will more likely develop a fever

MS

Charcot's Triad (IAN)

5 foot care interventions that should be taught to a new diabetic patient:

Check feet daily and report any breaks, sores or blisters to health care provider, wear well0fitting shoes, never go to barefoot or wear sandals, never personally remove corns or calluses, cut or file nails straight across, wash feet daily with mild soap and warm water

For Meningitis:

Check for Kernig's/Brudzinski's signs

Digoxin

Check pulse and HOLD if < 60 BPM Check digoxin and potassium levels

After endoscopy:

Check the gag reflex

Acid Ash Diet:

Cheese, corn, cranberries, plums, prunes, meat, poultry, pastry, bread

Urine: Nitrates Nursing Implications - testing

Chemical testing done with a dipstick containing a reagent that reacts with nitrites to produce a pink color

The "3 C's" of TEF in the newborn:

Choking Coughing Cyanosis

Multiple Sclerosis:

Chronic, progressive disease with demyelinating lesions in the CNS which affect the white matter of the brain and spinal cord Motor S/S: limb weakness, paralysis, slow speech Sensory S/S: numbness, tingling, tinnitus Cerebral S/S: nystagmus, ataxia, dysphagia, dysarthria

Alzheimer's disease is:

Chronic, progressive, degenerative cognitive disorder that accounts for more than 60% of all dementias

Hypocalcemia

Chvostek and Trosseaus's Signs

Which of the orders should a nurse question as part of the plan of care for a pt with a stage III pressure ulcer?

Clean the ulcer q shift with Dakins solution (active ingredient in bleach) . Should not use more than twice a day

Urea Nitrogen - Nursing Implications

Client NPO 12hr proir to test and no alcohol 24 hours proir to test

Glucose - Lab Values: Nursing implications

Client to be NPO except water 8 hr proir to testing. Caffein may cause increased values

Cataract Surgery

Client will sleep on the UNAFFECTED side with a NIGHT shield for 1-4 weeks

Cataracts:

Cloudy, blurry vision. Treated by lens-removal surgery

A patient has a nursing diagnosis of disturbed sensory perception related to sleep deprivation. Which action will the nurse include in the plan of care?

Cluster nursing activities so that the patient has uninterrupted rest periods. rationale: Clustering nursing activities and providing uninterrupted rest periods will minimize sleep-cycle disruption. Sedative and opioid medications tend to decrease the amount of rapid eye movement (REM) sleep and can contribute to sleep disturbance and disturbed sensory perception. Silencing the alarms on the cardiac monitors would be unsafe in a critically ill patient, as would discontinuing assessments during the night.

Acromegaly

Coarse facial feature

Heat or Cold? Decreased blood flow to injured site

Cold

Heat or Cold? Helps prevent edema from forming

Cold

Heat or Cold? Increased Blood Viscosity

Cold

Heat or Cold? Local Anesthesia

Cold

Heat or Cold? Promotes blood coagulation at injury site

Cold

Heat or Cold? Reduced cell metabolism

Cold

Heat or Cold? Reduces Inflammation

Cold

Heat or Cold? Reduces O2 needs of tissues

Cold

Heat or Cold? Relieves Pain

Cold

Heat or Cold? Vasoconstriction

Cold

Urine: Glucose - Nursing Implications - Testing

Collect fresh double voided specimen.

Where are most uncomplicated UTIs acquired?

Community

MRSA

Contact precaution... ONLY!

Amyotrophic Lateral Sclerosis (ALS) is a:

Condition in which there is a degeneration of motor neurons in both the upper and lower motor neuron systems.

Lyme's Disease is mostly found in:

Conneticut

VRSA

Contact and airborne precaution

A patient presents at the free clinic c/o urticaria and red rash. She is diagnosed with a delayed hypersensitive reaction. The nurse caring for this patient knows that an example of hypersensitivity reactions characterized by delayed reaction that occurs 24-72 hours after exposure to an antigen is what?

Contact dermatitis from tape adhesive

Treatment for Knee Replacement:

Continuous passive motion machine

Levodopa (antiparkinson):

Contraindicated with MAOIs

Sinemet (antiparkinson):

Contraindicated with MAOIs

Which client statement demonstrates positive progress toward recovery from substance abuse? A) "I have completed detox and therefore am in control of my drug use." B) "I will faithfully attend Narcotic Anonymous (NA) when I can't control my carvings." C) "As a church deacon, my focus will now be on spiritual renewal." D) "Taking those pills got out of control. It cost me my job, marriage, and children."

Correct Answer: D --- clients who takes responsibility for the consequences of substance abuse/dependence is making positive progress toward recovery. This client would most likely be in the working phase of the counseling process in which acceptance of the fact that substance abuse causes problems occurs.

"An 85-year-old woman with a hip fracture is scheduled for surgery. She has Alzheimer's disease and is only oriented to her name. Which of the following should the nurse look for on the informed consent? "a. The patient's mark witnessed by the surgeon. b. The patient's explanation of the operative procedure. c. A signature of a person who has legal guardianship of the patient. d. The surgeon's note stating that the surgery was explained to the patient.

Correct answer is c"Rationale: If the patient is mentally incompetent to sign the permit for surgery, then written permission may be given by a legally appointed representative or responsible family member.

A nurse is monitoring a client who abuses alcohol for signs of alcohol withdrawal. Which of the following would alert the nurse to the potential for withdrawal delirium? 1. Hypotension, ataxia, hunger 2. Stupor, agitation, muscular rigidity, 3. hypotension, coarse hand tremors, aitation 4. hypertension, changes in level of consciousness, hallucinations

Correct answer=4. Rationale: Symptoms associated with withdrawal delirium typically include anxiety, insomnia, anorexia, hypertension, disorientation, hallucinations, changes in level of consciousness, agitation, fever, and delusions.

The nurse would expect a cocaine overdose in a patient who is experiencing a. craving, restlessness, irritability b. agitation, cardiac dysrhythmia, and seizures c. diarrhea, nausea and vomiting, and confusion d. slow, shallow respirations, hyporeflexia and blurred vision" "a. craving, restlessness, irritability

Correct, B = agitation, cardiac dysrhythmia and seizures "a. Acute overdose of cocaine does not include cravings as a symptom b. Cocaine is a stimulant and will result in psychological, cardiac and neurological problems c. Cocaine typically does not have adverse GI effects d. These are symptoms of alcohol overdose."

According to psychological theorists, personality traits associated with drug abuse include: 1. difficulty dealing with intimacy 2. poor impulse control 3. low self-esteem 4. narcisstic behavior 5. introverted personality

Correct: 3, 4, 2, 1"Rationale: 1. Low self-esteem. Clients have a diminished sense of self-worth and value. Alcohol can numb these feelings and clients perceives themselves as more social, more relaxed, and enjoying life more. 2. Difficulty dealing with intimacy. The client may feel inadequate, insecure, and have low self-esteem. Personal intimacy threatens the alcoholic's sense of self. Alcohol is thought to be an avenue to alleviate anxiety and feelings of inadequacy. 3. Introverted personality: Behavioral traits associated with substance abuse include overt extroversion, not introversion. 4. Poor impulse control. The need for immediate gratification can drive alcohol and drug abuse, which is perceived by the client as a path to feel good immediately. 5. Narcissistic behaviors. The narcissistic person tends to feel unique or superior to others and is in control of the situation, including behaviors involving substance consumption. Cognitive Level: Application Client Need: Psychological Integrity Nursing Process: Assessment Learning Outcome: Discuss the major theoretic explanations for substance-related disorders."

PT/PTT are elevated when client is on:

Coumadin

Cognitive Therapy:

Counseling

Oh (Olfactory I) Some Oh (Optic II) Say Oh (Oculomotor III) Marry To (Trochlear IV) Money Touch (Trigeminal V) But And (Abducens VI) My Feel (Facial VII) Brother A (Auditory VIII) Says Girls (Glossopharyngeal IX) Big Vagina (Vagus X) Bras And (Accessory XI) Matter Hymen (Hypoglassal XII) More

Cranial nerves tips

MI

Crushing, stabbing pain that is NOT relieved by Nitro; pain typically radiates to the left shoulder, neck, or arms

Angina

Crushing, stabbing pain that is relieved by Nitro

Urine: Crystals - Nursing Implications - general

Crystals found on microscopic examination indicate renal stone formation is imminent, if not already present.

Pancreatitis

Cullen's Sign (ecchymosis of umbilicus); + Grey Turner's Spots

Prothrombin Time - Used to help regulate which drug therapy?

Cumadin Thereputic range is 1.35-2 times normal or control

PRIOR TO STARTING A TUBE FEEDING, THE NURSE ASSESSES THE PH AND COLOR OF THE PATIENT'S GASTRIC CONTENTS AND RECEIVES A PH READING OF 6.2 AND THE ASPIRATE IS OFF-WHITE COLOR. A STOMACH B SMALL INTESTINE C COLON D RESPIRATORY TRACT

D

THE DRESSING CHANGE ON A DEEP UPPER-ARM WOUND IS PAINFUL FOR THE PATIENT. WHEN PREPARING A CARE PLAN FOR THE PATIENT, THE NURSE WILL INCORPORATE WHICH OF THE FOLLOWING MEASURES: A ADMINISTER ANALGESIC IMMEDIATELY BEFORE DRESSING CHANGE B PERFORM DRESSING CHANGE WHEN PATIENT IS FATIGUED FROM PT C PERFORM DRESSING CHANGE DURING MEALTIME SO PATIENT IS DISTRACTED D ADMINISTER ANALGESIC 30-45 MIN BEFORE DRESSING CHANGE

D

The evening nurse reviews the nursing documentation in the male client's chart and notes that the day nurse has documented that the client has a stage II pressure ulcer in the sacral area. Which of the following would the nurse expect to note on assessment of the client's sacral area? a. Intact skin b. Full-thickness skin loss c. Exposed bone, tendon, or muscle d. Partial-thickness skin loss of the dermis

D

The nurse is caring for a patient who had knee replacement surgery 5 days ago. The patient's knee appears red and is very warm to the touch. The patient requests pain medication. Which of the following would be a correct explanation of what the nurse is noticing? A) These are expected findings for this postoperative time period. B) The patient may becoming dependent upon pain medication. C) The nurse should observe the patient more closely for wound dehiscence. D) The patient is demonstrating signs of a postoperative wound infection.

D

WHICH ONE OF THE NUTRITIONAL GUIDELINES SHOULD THE NURSE GIVE A WOMAN IN HER 2ND TRIMESTER OF PREGNANCY A EAT NORMAL NUMBER OF CALORIES BUT INCREASE FRUITS AND VEGETABLES B MAINTAIN REG CALORIE INTAKE, BUT TAKE SUPPLEMENTS C EAT AS MUCH AS YOU CAN D MORE CALORIES AND HIGH IN NUTRIENTS

D

34. All of these orders are received for a patient having an acute asthma attack. Which one will the nurse administer first? a. IV methylprednisolone (Solu-Medrol) 60 mg b. triamcinolone (Azmacort) 2 puffs per MDI c. salmeterol (Serevent) 50 mcg per DPI d. albuterol (Ventolin) 2.5 mg per nebulizer

D Rationale: Albuterol is a rapidly acting bronchodilator and is the first-line medication to reverse airway narrowing in acute asthma attacks. The other medications work more slowly. Cognitive Level: Application Text Reference: p. 620 Nursing Process: Implementation NCLEX: Physiological Integrity

17. A patient is seen in the clinic with COPD. Which information given by the patient would help most in confirming a diagnosis of chronic bronchitis? a. The patient tells the nurse about a family history of bronchitis. b. The patient denies having any respiratory problems until the last 6 months. c. The patient's history indicates a 40 pack-year cigarette history. d. The patient complains about having a productive cough every winter for 2 months.

D Rationale: A diagnosis of chronic bronchitis is based on a history of having a productive cough for several months for at least 2 consecutive years. There is no familial tendency for chronic bronchitis. Although smoking is the major risk factor for chronic bronchitis, a smoking history does not confirm the diagnosis. Cognitive Level: Application Text Reference: p. 629 Nursing Process: Assessment NCLEX: Physiological Integrity

37. The nurse has received a change-of-shift report about these patients with COPD. Which patient should the nurse assess first? a. A patient with loud expiratory wheezes b. A patient who has a cough productive of thick, green mucus c. A patient with jugular vein distension and peripheral edema d. A patient with a respiratory rate of 38

D Rationale: A respiratory rate of 38 indicates severe respiratory distress, and the patient needs immediate assessment and intervention to prevent possible respiratory arrest. The other patients also need assessment as soon as possible, but they do not need to be assessed as urgently as the tachypneic patient. Cognitive Level: Application Text Reference: pp. 612, 626 Nursing Process: Assessment NCLEX: Safe and Effective Care Environment

20. During IV administration of amphotericin B ordered for treatment of coccidioidomycosis, the nurse increases the patient's tolerance of the drug by a. cooling the solution to 80° F before administration. b. keeping the patient flat in bed for 1 hour after the infusion is completed. c. diluting the amphotericin B in 500 ml of sterile water. d. giving diphenhydramine (Benadryl) 1 hour before starting the infusion.

D Rationale: Administration of an antihistamine before giving the amphotericin B will reduce the incidence of hypersensitivity reactions. Cooling the solution and keeping the patient flat after infusion are not indicated. Amphotericin B does not need to be diluted in 500 ml of fluid, although the nurse should ensure adequate hydration in the patient receiving this drug. Cognitive Level: Application Text Reference: p. 575 Nursing Process: Implementation NCLEX: Physiological Integrity

39. After discharge teaching has been completed for a patient who has had a lung transplant, the nurse will evaluate that the teaching has been effective if the patient states a. "I will make an appointment to see the doctor every year." b. "I will not turn the home oxygen up higher than 2 L/minute." c. "I will be careful to use sterile technique with my central line." d. "I will write down my medications and spirometry in a journal."

D Rationale: After lung transplant, patients are taught to keep logs of medications, spirometry, and laboratory results. Patients require frequent follow-up visits with the transplant team; annual health care provider visits would not be sufficient. Home oxygen use is not an expectation after lung transplant and patients would not usually have a central IV line. Cognitive Level: Application Text Reference: p. 604 Nursing Process: Evaluation NCLEX: Physiological Integrity

6. A 77-year-old patient with pneumonia has a fever of 101.2° F (38.5° C), a nonproductive cough, and an oxygen saturation of 89%. The patient is very weak and needs assistance to get out of bed. The priority nursing diagnosis for the patient is a. hyperthermia related to infectious illness. b. ineffective airway clearance related to thick secretions. c. impaired transfer ability related to weakness. d. impaired gas exchange related to respiratory congestion.

D Rationale: All these nursing diagnoses are appropriate for the patient, but the patient's oxygen saturation indicates that all body tissues are at risk for hypoxia unless the gas exchange is improved. Cognitive Level: Application Text Reference: p. 566 Nursing Process: Diagnosis NCLEX: Physiological Integrity

2. The nurse recognizes that intubation and mechanical ventilation are indicated for a patient in status asthmaticus when a. ventricular dysrhythmias and dyspnea occur. b. loud wheezes are audible throughout the lungs. c. pulsus paradoxus is greater than 40 mm Hg. d. fatigue and an O2 saturation of 88% develop.

D Rationale: Although all of the assessment data indicate the need for rapid intervention, the fatigue and hypoxia indicate that the patient is no longer able to maintain an adequate respiratory effort and needs mechanical ventilation. The initial treatment for the other clinical manifestations would initially be administration of rapidly acting bronchodilators and oxygen. Cognitive Level: Application Text Reference: pp. 612-613 Nursing Process: Assessment NCLEX: Physiological Integrity

24. When developing a teaching plan for a patient with a 42 pack-year history of cigarette smoking, it will be most important for the nurse to include information about a. reasons for annual sputum cytology testing. b. CT screening for lung cancer. c. erlotinib (Tarceva) therapy to prevent tumor risk. d. options for smoking cessation.

D Rationale: Because smoking is the major cause of lung cancer, the most important role for the nurse is educating patients about the benefits of and means of smoking cessation. Early screening of at-risk patients using sputum cytology, chest x-ray, or CT scanning has not been effective in reducing mortality. Tarceva may be used to in patients who have lung cancer, but not to reduce risk for developing tumors. Cognitive Level: Application Text Reference: pp. 582, 584 Nursing Process: Planning NCLEX: Health Promotion and Maintenance

14. The nurse recognizes that the goals of teaching regarding the transmission of TB have been met when the patient with TB a. demonstrates correct use of a nebulizer. b. reports daily to the public health department. c. washes dishes and personal items after use. d. covers the mouth and nose when coughing.

D Rationale: Covering the mouth and nose will help decrease airborne transmission of TB. The other actions will not be effective in decreasing the spread of TB. Cognitive Level: Application Text Reference: p. 574 Nursing Process: Evaluation NCLEX: Health Promotion and Maintenance

34. When providing preoperative instruction for a patient scheduled for a left pneumonectomy for cancer of the lung, the nurse informs the patient that the postoperative care includes a. positioning on the right side. b. chest tubes to water-seal chest drainage. c. bedrest for the first 24 hours. d. frequent use of an incentive spirometer.

D Rationale: Frequent deep breathing and coughing are needed after chest surgery to prevent atelectasis. To promote gas exchange, patients after pneumonectomy are positioned on the surgical side. Chest tubes are not usually used after pneumonectomy because the affected side is allowed to fill with fluid. Early mobilization decreases the risk for postoperative complications such as pneumonia and deep vein thrombosis. Cognitive Level: Application Text Reference: pp. 596-597 Nursing Process: Planning NCLEX: Physiological Integrity

30. A patient experiences a steering wheel injury as a result of an automobile accident. During the initial assessment, the emergency department nurse would be most concerned about a. complaints of severe pain. b. heart rate of 110 beats/min. c. a large bruised area on the chest. d. paradoxic chest movement.

D Rationale: Paradoxic chest movement indicates that the patient may have flail chest, which will severely compromise gas exchange and can rapidly lead to hypoxemia. Severe pain, a slightly elevated pulse rate, and chest bruising all require further assessment or intervention, but the priority concern is poor gas exchange. Cognitive Level: Application Text Reference: pp. 586, 588 Nursing Process: Assessment NCLEX: Physiological Integrity

8. Following discharge teaching, the nurse evaluates that the patient who was admitted with pneumonia understands measures to prevent a reoccurrence of the pneumonia when the patient states, a. "I will increase my food intake to 3000 calories a day." b. "I will need to use home oxygen therapy for 3 months." c. "I will seek medical treatment for any upper respiratory infections." d. "I will do deep-breathing and coughing exercises for the next 6 weeks."

D Rationale: Patients at risk for recurrent pneumonia should use the incentive spirometer or do deep breathing and coughing exercises or both for 6 to 8 weeks after discharge. Although caloric needs are increased during the acute infection, 3000 calories daily will lead to obesity and increase the risk for pneumonia. Patients with acute lower respiratory infections do not usually require home oxygen therapy. Upper respiratory infections require medical treatment only when they fail to resolve in 7 days. Cognitive Level: Application Text Reference: p. 569 Nursing Process: Evaluation NCLEX: Health Promotion and Maintenance

41. After teaching the patient with asthma about home care, the nurse will evaluate that the teaching has been successful if the patient states, a. "I will use my corticosteroid inhaler as soon as I start to get short of breath." b. "I will only turn the home oxygen level up after checking with the doctor first." c. "My medications are working if I wake up short of breath only once during the night." d. "No changes in my medications are needed if my peak flow is at 80% of normal."

D Rationale: Peak flows of 80% or greater indicate that the asthma is well controlled. Corticosteroids are long-acting, prophylactic therapy for asthma and are not used to treat acute dyspnea. Because asthma is an acute and intermittent process, home oxygen is not used. The patient who has effective treatment should sleep throughout the night without waking up with dyspnea. Cognitive Level: Application Text Reference: p. 628 Nursing Process: Evaluation NCLEX: Physiological Integrity

30. A 23-year-old with cystic fibrosis (CF) is admitted to the hospital. Which intervention will be included in the plan of care? a. Schedule sweat chloride test to evaluate the effectiveness of therapy. b. Arrange for a hospice nurse to visit with the patient regarding home care. c. Place the patient on a low-sodium diet to prevent cor pulmonale. d. Perform chest physiotherapy every 4 hours to mobilize secretions.

D Rationale: Routine scheduling of airway clearance techniques is an essential intervention for patients with CF. A sweat chloride test is used to diagnose CF, but it does not provide any information about the effectiveness of therapy. There is no indication that the patient is terminally ill. Patients with CF lose excessive sodium in their sweat and require high amounts of dietary sodium. Cognitive Level: Application Text Reference: p. 658 Nursing Process: Planning NCLEX: Physiological Integrity

23. A patient is receiving 35% oxygen via a Venturi mask. To ensure the correct amount of oxygen delivery, it is most important that the nurse a. give a high enough flow rate to keep the bag from collapsing. b. use an appropriate adaptor to ensure adequate oxygen delivery. c. drain moisture condensation from the oxygen tubing every hour. d. keep the air entrainment ports clean and unobstructed.

D Rationale: The air entrainment ports regulate the oxygen percentage delivered to the patient, so they must be unobstructed. A high oxygen flow rate is needed when giving oxygen by partial rebreather or non-rebreather masks. The use of an adaptor can improve humidification but not oxygen delivery. Draining oxygen tubing is necessary when caring for a patient receiving mechanical ventilation. Cognitive Level: Comprehension Text Reference: p. 642 Nursing Process: Implementation NCLEX: Physiological Integrity

21. When a patient with COPD is receiving oxygen, the best action by the nurse is to a. avoid administration of oxygen at a rate of more than 2 L/min. b. minimize oxygen use to avoid oxygen dependency. c. administer oxygen according to the patient's level of dyspnea. d. maintain the pulse oximetry level at 90% or greater.

D Rationale: The best way to determine the appropriate oxygen flow rate is by monitoring the patient's oxygenation either by ABGs or pulse oximetry; an oxygen saturation of 90% indicates adequate blood oxygen level without the danger of suppressing the respiratory drive. For patients with an exacerbation of COPD, an oxygen flow rate of 2 L/min may not be adequate. Because oxygen use improves survival rate in patients with COPD, there is not a concern about oxygen dependency. The patient's perceived dyspnea level may be affected by other factors (such as anxiety) besides blood oxygen level. Cognitive Level: Application Text Reference: p. 640 Nursing Process: Implementation NCLEX: Physiological Integrity

28. A patient is admitted to the emergency department with a stab wound to the right chest. Air can be heard entering his chest with each inspiration. To decrease the possibility of a tension pneumothorax in the patient, the nurse should a. position the patient so that the right chest is dependent. b. administer high-flow oxygen using a non-rebreathing mask. c. cover the sucking chest wound with an occlusive dressing. d. tape a nonporous dressing on three sides over the chest wound.

D Rationale: The dressing taped on three sides will allow air to escape when intrapleural pressure increases during expiration, but it will prevent air from moving into the pleural space during inspiration. Placing the patient on the right side or covering the chest wound with an occlusive dressing will allow trapped air in the pleural space and cause tension pneumothorax. The patient should receive oxygen, but this will have no effect on the development of tension pneumothorax. Cognitive Level: Application Text Reference: p. 586 Nursing Process: Implementation NCLEX: Physiological Integrity

10. After a patient with right lower-lobe pneumonia has been treated with intravenous (IV) antibiotics for 2 days, which assessment data obtained by the nurse indicates that the treatment has been effective? a. Bronchial breath sounds are heard at the right base. b. Increased vocal fremitus is palpable over the right chest. c. The patient coughs up small amounts of green mucous. d. The patient's white blood cell (WBC) count is 9000/µl.

D Rationale: The normal WBC count indicates that the antibiotics have been effective. All the other data suggest that a change in treatment is needed. Cognitive Level: Application Text Reference: p. 569 Nursing Process: Evaluation NCLEX: Physiological Integrity

43. In developing a teaching plan for a patient who is being discharged with a warfarin (Coumadin) prescription after having a pulmonary embolus, the nurse will include information about a. where to schedule activated partial thromboplastin time testing. b. avoidance of a high protein diet. c. how to obtain enteric-coated aspirin. d. foods that are high in vitamin K.

D Rationale: The patient who is taking Coumadin should have a consistent vitamin K intake, since vitamin K interferes with the effect of the medication. INR testing, rather than aPTT testing, is used to monitor for a therapeutic level of Coumadin. Aside from vitamin K, there are no other dietary requirements associated with Coumadin use. Aspirin should be avoided when taking anticoagulant medications because of the effect on platelet function. Cognitive Level: Application Text Reference: p. 600 Nursing Process: Planning NCLEX: Physiological Integrity

26. A patient with severe COPD tells the nurse, "I wish I were dead! I cannot do anything for myself anymore." Based on this information, the nurse identifies the nursing diagnosis of a. hopelessness related to presence of long-term stress. b. anticipatory grieving related to expectation of death. c. ineffective coping related to unknown outcome of illness. d. disturbed self-esteem related to physical dependence.

D Rationale: The patient's statement about not being able to do anything for himself or herself supports this diagnosis. Although hopelessness, anticipatory grieving, and ineffective coping may also be appropriate diagnoses for patients with COPD, the patient does not mention long-term stress, death, or an unknown outcome as being concerns. Cognitive Level: Application Text Reference: p. 655 Nursing Process: Diagnosis NCLEX: Psychosocial Integrity

1. Following assessment of a patient with pneumonia, the nurse identifies a nursing diagnosis of ineffective airway clearance. Which information best supports this diagnosis? a. Resting pulse oximetry (SpO2) of 85% b. Respiratory rate of 28 c. Large amounts of greenish sputum d. Weak, nonproductive cough effort

D Rationale: The weak, nonproductive cough indicates that the patient is unable to clear the airway effectively. The other data would be used to support diagnoses such as impaired gas exchange and ineffective breathing pattern. Cognitive Level: Application Text Reference: p. 568 Nursing Process: Diagnosis NCLEX: Physiological Integrity

Tetralogy of Fallot

DROP: Defect (septal) Right ventricular hypertrophy Overriding aortas Pulmonary stenosis

Push fluids with Allopurinol:

Flushes the uric acid out of the system

A nurse is conducting preoperative teaching with a client about the use of incentive spirometer. The nurse should include which piece of information in discussions with the client? A. Inhale as rapidly as possible. B. Keep a loose seal between the lips and mouth piece. C. After maximum inspiration, hold teh breath for 15 seconds and exhale. D. The best results are achieved when sitting up or with the head of the bed elevated 45-90 degrees.

D is correct, must be in high fowler's position for optimal lung expansion. The mouthpiece should be covered tightly and breath holds for 5 seconds.

Heat or Cold? Decreased Blood Viscosity

Heat

Heat or Cold? Decreases Spasmodic Pain

Heat

"The wife of the client diagnosed with chronic alcoholism tells the nurse, ""I have to call his work just about every Monday to let them know he is ill or he will lose his job."" Which would be the nurse's best response? "A. ""I'm sure that this must be hard for you. Tell me about your concerns."" B. ""You are afraid he will lose his source of income."" C. ""Why would you call in for your husband? Can't he do this?"" D. ""Are you aware that when you do this your are enabling him?""

D. ""Are you aware that when you do this your are enabling him?"" "A and B. These are therapeutic responses. The spouse is not expressing feelings but is stating a fact. The nurse should address the problem. C. The spouse is not required to give an explanation to the nurse. D. The spouse's behavior is enabling the client to continue to drink until he cannot function."

The nurse assesses a preoperative client. Which question should the nurse ask the client, to help determine the client's risk for developing malignant hyperthermia in the periperative period? A. Have you ever had heat exhaustion or heat stroke? B. What is the normal range for you body temperature? C. Do you or any of your family members have frequent infections? D. Do you or any of your family members have problems with anesthesia?

D. Do you or any of your family members have problems with anesthesia? Malignat hyperthermia is a genetic disorder in which a combination of anesthetic agents triggers uncontrolled skeletal muscle contractions that lead to a potentially fatal hyperthermia. Questioning the client about family history of general anesthesia problems reveal this as a risk for the client. Options A, B, and C are unrelated to this surgical complication.

Chapter 5: 1. When you ask an older student why it is necessary to change the pt's bed every day, he says, " I guess we have always done it that way." This answer is example of which of the following? a. Unsubstantiated knowledge b. scientific knowledge c. Authoritative knowledge d. Traditional knowledge

D. Traditional knowledge is a the part of nursing practice passed down from generation to generation, often without research data to support it.

A patient is scheduled for a laparoscopic cholecystectomy at an ambulatory surgery center. What do you expect? "A. Curative surgery for cancer of the pancreas. B. Palliative surgery for a resection of a tumor. C. Surgery with small incisions for removal of the liver. D. Removal of the gallbladder using a minimally invasive technique.

D: Most surgical procedures are being performed as ambulatory surgery (also called same-day or outpatient surgery). Many of these operations use minimally invasive techniques (e.g., laparoscopic techniques). Cholecystectomy is removal of the gallbladder.

Client's with DELUSIONS:

DISTRACT them

The nursing instructor is discussing diabetes mellitis with the junior nursing class. What would the instructor tell the class may develop in the patient when ketone bodies accumulate in excessive amounts?

DKA, diabetic ketoacidosis.

DKA is rare in:

DM Type II because there is still enough insulin to prevent breakdown of fats

E - evaluate A - assess T - teach

DO NOT delegate what you can EAT!

Allopurinol:

DO NOT give Vitamin C

Yogurt has live cultures:

DO NOT give to immunosuppressed clients

Jewish Clients

DO NOT serve MEAT and MILK together

Child age 4 to 5 years needs:

DPT, MMR, OPV

Dakins Solution

Dakin's solution is used to prevent and treat skin and tissue infections that could result from cuts, scrapes and pressure sores. It is also used before and after surgery to prevent surgical wound infections. Dakin's solution is a type of hypochlorite solution. It is made from bleach that has been diluted and treated to decrease irritation. Chlorine, the active ingredient in Dakin's solution, is a strong antiseptic that kills most forms of bacteria and viruses.

AMI:

Dead heart tissue is PRESENT

Myasthenia Gravis:

Decrease in receptor sites for acetylcholine. Since smallest concentration of ACTH receptors are in cranial nerves, expect fatigue and weakness in the eyes, mastication, and pharyngeal muscles.

A geriatric nurse is preforming an assessment of body systems on an 85 y/o patient. The nurse realizes that what particular change is age related affecting the renal or urinary system?

Decreased GFR

What does a -4 mean on the RASS scale?

Deep sedation, not responding to voice; one of the worst scores

Magnesium Sulfate (used to halt preterm labor) is contraindicated if:

Deep tendon reflexes are reduced.

Iron antedote:

Deferoxamine

A clinet in cardiac arrest is noted on bedside monitor to be in pulseless vetricular tachycardia. What is the first action that should be taken?

Defibrilation

IM sites for Children:

Deltoid and Gluteus Maximus

What medication is used to treat pain caused by pancreatitis:

Demerol

Heat or Cold? Improves delivery of antibiotics to wound

Heat

Hypermagnesemia

Depresses the CNS, hypotension, facial flushing, muscle weakness, absent deep tendon reflexes, shallow respirations (this is a medical emergency)

MG

Descending muscle weakness

The family members of a patient who has just been admitted to the intensive care unit (ICU) with multiple traumatic injuries have just arrived in the ICU waiting room. Which action should the nurse take first?

Describe the patient's injuries and the care that is being provided. rationale: Lack of information is a major source of anxiety for family members and should be addressed first. Family members should be prepared for the patient's appearance and the ICU environment before visiting the patient for the first time. ICU visiting should be individualized to each patient and family rather than being dictated by rigid visitation policies. Inviting the family to participate in a multidisciplinary conference is appropriate but should not be the initial action by the nurse.

Multiple Sclerosis (myelin sheath destruction) leads to:

Destruction in nerve impulse conduction

2-3 months: turns head side to side 4-5 months: grasps, switch & roll 6-7 months: sit at 6 and waves bye-bye 8-9 months: stands straight at eight 10-11 months: belly to butt (phrase has 10 letters) 12-13 months: twelve and up, drink from a cup

Developmental stages

Caput Succedaneum:

Diffuse edema of the fetal scalp that crosses the suture lines. Swelling reabsorbs within 1 to 3 days.

Digoxin antedote:

Digibind

Potassium potentiates:

Digoxin Toxicity

Theophylline increases the risk of:

Digoxin toxicity and DECREASES the effects of LITHIUM and DILANTIN

Do NOT given INH with:

Dilantin (can cause phenytoin toxicity)

Heat or Cold? Improves delivery of leukocytes to wound

Heat

Heat or Cold? Increased Capillary Permeability

Heat

Your patient is on tube feedings and is experiencing diarrhea. You suspect the patient is experiencing dumping syndrome. How might the nurse help to alleviate this problem?

Dilute the feeding solution

What are the essential steps of effective supervison?

Direction, evealuation and follow-up

"A 65 year old man has been admitted to the hospital for spinal stenosis surgery. When does the discharge training and planning begin for this patient? "A: Following surgery B: Upon admit C: Within 48 hours of discharge D: Preoperative discussion"

Discharge planning for any surgery begins upon admit.

Cholesterol - Nursing Implication

Do not collect a sample from the same arm as an infusing IV

Sodium - Nursing Implications

Do not collect a sample from the same arm as an infusing IV

NO PEE... NO K+

Do not give potassium without adequate urine output

Total Hip Replacement

Don't sleep on the operated side Don't flex the hip more than 45-60 degrees Don't elevate the HOB more than 45 degrees Maintain hip ABDUCTION by separating the thighs with pillows

Risperdal (antipsychotic):

Doses over 6 mg can cause tardive dyskinesia (used as the 1st line antipsychotic in children)

Position of Ear for child

Down and Back

Red Blood Cell Count - in medication administration - what should you never do when drawing a sample for this test?

Draw from an arm that is being infused with an IV

Describe the method of drawing up a mixed dose of insulin, regular with NPH

Draw regular insulin first then NPH

think of SPIDERMAN! S - sepsis S - scarlet fever S - streptococcal pharyngitis P - parvovirus B19 P - pneumonia P - pertussis I - influenza D - diptheria (pharyngeal) E - epiglottitis R - rubella M - mumps M - meningitis M - mycoplasma or meningeal pneumonia An - Adenovirus Private Room or cohort Mask

Droplet precautions

Urine: Specific Gravity - Nursing Implications - drugs

Drugs that may cause increased specific gravity include dextran and sucrose.

Anticholinergic Effects:

Dry mouth, urinary retention, constipation, blurred vision

Stomas

Dusky stoma means poor blood supply; protruding stoma means prolapsed; sharp pain and rigidity mean peritonitis; mucus in the ileal conduit is expected

Cardiac output decreases with:

Dysrhythmias

Thorazine, Haldol (antipsychotic) can lead:

EPS

S/S of a Fractured Hip:

EXTERNAL ROTATION, SHORTENING, and ADDUCTION.

A patient comes to the clinic c/o pain in the epigastric region. The nurse suspects that the patient's pain is r/t a peptic ulcer when the patient states the pain is relieved by what?

Eating

Cullen's Sign:

Ecchymosis in the umbilical area, see with Pancreatitis

Head Injury

Elevate HOB 30 degrees to decrease ICP

After Supratentorial Surgery (incision behind the hairline)

Elevate HOB 30-45 degrees

Above Knee Amputation (AKA)

Elevate for the first 24 hours on a pillow Position prone daily to provide for hip extension

Buck's Traction (skin traction)

Elevate the foot of the bed for counter-traction

Below Knee Amputation (BKA)

Elevate the foot of the bed for the first 24 hours Position prone daily to provide for hip extension

A nurse is caring for a patient with Crohn's disease. The patient is scheduled for a barium enema. What is an appropriate nursing intervention the day before the test?

Encourage plenty of fluids

Chp. 9 Maria is a 6 y.o. child from Mexico. Maria's socialization into the Mexican culture is best described as: 1. Assimilation 2. Acculturation 3. Biculturalism 4. Enculturation

Enculturation

What does the Glasgow Coma Scale assess?

Eye opening, motor response, verbal response; 15 is normal, <7 is coma

Heat or Cold? Increased Tissue metabolism

Heat

Heat or Cold? Increases Blood flow

Heat

Medication of choice for anaphylactic shock:

Epinephrine

Medication of choice for asystole:

Epinephrine

What is the first drug most likely to be used for an in-hospital cardiac arrest?

Epinephrine

The nurse is doing triage at the community clinic when a middle-age patient presents with abdominal pains and heartburn. The patient states the symptoms have persisted for several days following a particularly spicy meal. When assessing the patient, the nurse notes the patient has a history of acute gastritis. What complication would the nurse be particularly likely to assess for?

Esophageal or pyloric obstruction r/t scarring.

Transesophageal Fistula (TEF):

Esophagus doesn't fully develop (this is a surgical emergency)

Premarin

Estrogen replacement for post-menopause

Urine: Leukocyte esterase - Nursing Implications - general

False-negative results may occur in specimines that contain high levels of protiens or ascorbic acid

The nurse is monitoring a client's blood pressure an hour after administering an antihypertensive medication. What is the purpose of this monitoring?

Evaluation of therapeutic effect Therapeutic effect occurs after the administration of the medication, and the nurse should assess for expected outcomes. Tolerance cannot be assessed by taking postadministration blood pressure. Toxicity cannot be assessed by taking blood pressure after administration. Need is determined before the administration of the drug.

DO NOT delegate what you EAT!

Evaluations Assessments Initial Teaching

KEY WORDS are very important. Avoid answers with absolutes:

Examples: ALWAYS NEVER MUST

A nurse is preforming blood pressure screenings at a local healthfair. While obtaining subjective assessment data from a patient with HTN, the patient tells the nurse that she has a family history of HTN and she herself has high cholesterol and lipids. The patient says she smokes 1 pack of cigarettes daily and drinks "about a pack of beer" everyday. The nurse notes that a modifiable risk factor of this patient is what?

Excessive alcohol intake

Diabetes Insipidus (decreased ADH)

Excessive urine output and thirst, dehydration, weakness Administer Pitressin

Hyperthyroidisms/Grave's Disease

Exopthalmus

Fibrin Hyalin

Expiratory Grunt

What is a deep full thickness (4th degree burn)?

Extends into subcutaneous tissue and may involve muscle and bone; has no blood flow or feeling, appeared blackened with eschar

V = variable decels; C = cord compression caused E = early decels; H = head compression caused A = accels; O = okay, not a problem! L = late decels = placental insufficiency, can't fill

FHR patterns in OBGYN, (VEAL CHOP)

-safety -setting limits 11-establish trusting relationship -meds -leas restrictive methods/environment.

FIVE INTERVENTIONS FOR PSYCH PATIENTS

What is Cranial Nerve VII?

Facial; facial movement, as well as sensing taste

Hyper-parathyroid

Fatigue, muscle weakness, renal calculi, back and joint pain (increased calcium), low Ca and high phosphorus diet

Russell Traction:

Femur or lower leg

Lumbar Puncture:

Fetal position. S/P: Neuro assessment q 15-30 minutes until stable; have client lay flat for 2-3 hours and encourage fluids

FiO2 over what value can cause oxygen toxicity?

FiO2 of 60% over a prolonged period

Fluid Retention:

First concern is heart problems

may be meconium ileus at birth. Baby is inconsolable, do not eat, not passing meconium

First sign of Cystic Fibrosis

Peptic ulcers caused by H. pylori are treated with:

Flagyl, Prilosec, and Biaxin. This treatment kills the bacteria and stops production of stomach acid; it DOES NOT heal the ulcer

Hepatic Encephalopathy

Flapping tremors

Heat or Cold? Promotes Muscle Relaxation

Heat

What does FiO2 represent?

Fraction of inspired O2; represented either as a percentage or decimal

Urine: Glucose - Lab Values

Fresh specimine: None - 24 hour Specimen 50-300

Stage III pressure Ulcer

Full thickness skin loss involving damage to or necrosis of subcutaneous tissue that may extend down to, but not through, underlying fascia. The ulcer presents clinically as a deep crater with or without undermining of adjacent tissue.

Stage IV pressure Ulcer

Full thickness skin loss with extensive destruction, tissue necrosis, or damage to muscle, bone, or supporting structures (e.g., tendon, joint capsule). Undermining and sinus tracts also may be associated with this type of pressure ulcer.

4 common causes of fluid volume DEFICIT

GI causes: vomiting, diarrhea, GI suctioning, decreased fluid intake, increase in fluid output like sweating, massive edema, ascites

A staff nurse is exposed to hepatitis B. Hospital protocol is the staff nurse report to the ED for the administration of what substance to provide passive acquired immunity?

Gamma Globulin

Pancreatitis (for pain control)

Give Demerol DO NOT give Morphine Sulfate

For ANY kind of bad FHR pattern:

Give O2--often by mask

Prior to any invasive procedure:

Give antibiotic therapy (prophylactic)

Rhogam:

Given at 28 weeks, 72 hours post partum; given IM. Only given to Rh NEGATIVE mothers. If the indirect Coomb's test is POSITIVE, then you DO NOT need to give Rhogam because she has antibody; only give if NEGATIVE Coombs.

Versed

Given for conscious sedation Assess for respiratory depression and hypotension

What is Cranial Nerve IX?

Glossopharyngeal; senses taste and responsible for the gag reflex

Heat or Cold? Promotes movement of wastes and nutrients

Heat

Turner's Sign:

Grayish-blue flank seen in Pancreatitis

Identify the ways HIV is transmitted

HIV is transmitted through blood and body fluids, unprotected sex, sharing needles, infected blood products, breastmilk, and needlesticks

Standard precautions for

HIV, MRSA or VRSA (but contact if more severe), tetanus, E Coli, mono (from 2nd day), impetigo (from 2nd day), lyme disease, thrush

Ventilator Alarms:

HOLD: High alarm--Obstruction due to increased secretions, kink, client cough, gag, or bites Low pressure alarm--Disconnection or leak in the ventilator or in the client airway cuff; or if the client stops spontaneous breathing

Thyroid storm is:

HOT (hyperthermia)

what should you monitor for in a dialysis patient?

HOTN, headache, malaise, vomitting, dizziness, muscle cramps

When doing an epidural anesthesia:

HYDRATION is a PRIORITY INTERVENTION prior to the procedure.

Preferred antipsychotic in elderly:

Haldol; but still high risk of EPS (dystonia, tardive dyskinesia, tightening of jaw, stiff neck, swollen tongue); monitor for early signs of reaction and treat with Benadryl (IM)

The nurse reads that the half-life of the medication being administered is 12 hours. What assumption will guide the nurse's care of this client?

Half-life refers to the time it takes to excrete a drug from the body. Administering the medication every 6 hours would not be appropriate; it would be too soon. Half-life does not refer to onset of action or to the number of doses in 24 hours.

Infectious Mononucleosis

Hallmark: sore throat, cervical lymph adenoapathy, and fever

Halo Traction:

Have a screwdriver nearby

Cushing's:

Have extra "cushion" of hormones

The nurse is teaching a newly diagnosed diabetic about his insulin regimen. When administering Humalog insulin the nurse should teach the patient what?

Have the patient's meal tray available.

Hematocrit - Abbreviation

Hct

Primary Intention

Healing takes place when the wound margins are neatly approximated. Examples: paper cut, surgery

4 common causes of fluid volume OVERLOAD

Heart failure, cirrhosis, excess ingestion of table salt or overhydration with sodium0containing fluids

What are the normal neonate vital signs?

Heart rate: 100-160; BP: >60/40-50 systolic; Respirations: 30-60; Temp: 97.7-98.9 (axillary)

The difference between community-based nursing and community- oriented nursing is that in community based nursing the nurse:

Helps individuals and families manage acute or chronic health problems in the community and home setting.

A nursing instructor is talking with her clinical group about patients with acute glomerulonephritis. The instructor tells the students that the patient may exhibit which of the following clinical manifestations?

Hematuria

Urine: RBC - Nursing Implications - general

Hematuria can be microsopic or gross. Bladder, ureteral, and urethral diseases are the most comon causes of RBCs in the urine. Traumatic urethral catherization may cause RBCs in the urine

Platelet Count - What might decrease the numbers?

Hemmorrhage, DIC, reduction in production, infections, prostetic heart valves, drugs

Potassium - Nursing Implications

Hemolysis of a specimen can result in a falsely elevated respoce

The patient asks the nursing assistant for a bedpan. When the patient is finished, the nursing assistant notifies the nurse that the patient has bright red streaking blood in the stool. What is this most likely the result of?

Hemorrhoids

Hemoglobin - what causes a decrease?

High Altitude Living

DVT

Homan's Sign

In one primary immunodeficiency disease WBC cannot initiate an inflammatory response to infectious organisms. What disease is it?

Hyperimmunoglobulinemia E Syndrome

Cushing's (up, up, up, down, up)

Hypernatremia Hypertension Hypervolemia HYPOkalemia Hyperglycemia

Pheochromocytoma

Hypersecretion of epinephrine/norepinephrine; persistent HTN, Tachycardia, hyperglycemia, diaphoresis, tremor, pounding HA Client should avoid stress; provide frequent bathings and rest breaks; avoid cold and stimulating foods Treatment: Surgical removal of tumor

Increased ICP

Hypertension, bradypnea, bradycardia (Cushing's Triad)

Don't use Kayexalate if client has:

Hypoactive bowel sounds

Trousseau and Tchovoski signs:

Hypocalcemia

Tetany

Hypocalcemia + Trousseau's Sign/carpopedal spasm; Chvostek Sign (facial spasm)

What is Cranial Nerve XII?

Hypoglossal; controls most of the tongue movements, important for speech and swallowing

If the U wave is the most prominent, what condition might the nurse suspect?

Hypokalemia

Addison's (down, down, down, up, down)

Hyponatremia Hypotension Hypovolemia HYPERkalemia Hypoglycemia

Shock

Hypotension, tachycardia, tachypnea

Cardinal sign of ARDS is:

Hypoxemia (low oxygen level in tissues)

Trendelenberg test for varicose veins:

If they fill proximally--varicosity

Spinal shock occurs:

Immediately after surgery

A COLD is NOT a contraindication to:

Immunizations

A client is breathing 40 breaths/min. He is diaphoretic and confused. What nursing diagnosis should be the priority for the client at this time?

Impaired gas exchange

A nurse is caring for a pt with diabetes and a necrotic left great toe who is scheduled for amputation of the affected toe. The Pts WBC count is 15.0 x 10 to the 6th power/ UL, and he has coolness of the lower extremities, weighs 75 lbs more than his ideal body weight, and smokes 2 packs of cigarettes per day Which priority nursing dx addresses the primary factor affecting the pts ability to heal?

Impaired skin integrity related to decreased blood flow secondary to diabetes and smoking.

A 65 yo stroke pt with limited mobility has a purple area of suspected deep tissue injury on the left greater trochanter. Which of the following nursing dx is/are most appropriate ? (select all that apply)

Impaired skin integrity related to immobility and decreased sensation Impaired tissue integrity related to inadequate circulation secondary to pressure.

An 85 yo pt is assessed to have a score of 16 on the Braden scale. Based on this information, how should the nurse plan for this pts care?

Implement a q2 hr turning schedule with skin assessment

A UAP may perform care that falls within which component of the nursing process?

Implementation

Prior to a Liver Biopsy

Important to be aware of the lab results for Prothrombin Time (PTT)

The nurse is caring for a patient who is scheduled for a colonscopy. The nurse is preparing to instruct the patient on a colon preparation procedure that will include polyethylene glycol electrolyte lavage prior to the procedure. What is the nurse aware of about the use of lavage solutions and when they are contraindicated?

In a patient with inflammatory bowel disease

Urine: Glucose - Nursing Implications - - Pregnancy

In pregnancy glycosuria is common, but persistant and signifiganly high levels may indicate gestationnal diabetes.

You are caring for a patient with a hematologic disorder. The patient asks you where the body forms blood cells. Where would you tell the patient that blood cells are formed?

In the bone marrow

The placenta is located:

In the upper part of the uterus

Antipsychotics (Haldol, Thorazine, Prolixin)

Incompatible with caffeine and apple juice

The nurse is assessing a patient's bladder by percussion. The nurse elicits dullness after voiding. What does this finding indicate?

Incomplete bladder emptying

Glucose - Lab Values: Aged Adult

Increase in normal range after 50

Following surgery, a patient's central venous pressure (CVP) monitor indicates low pressures. Which action will the nurse anticipate taking?

Increase the IV fluid infusion rate. rationale: A low CVP indicates hypovolemia and a need for an increase in the infusion rate. Diuretic administration will contribute to hypovolemia and elevation of the head may decrease cerebral perfusion. Documentation and continued monitoring is an inadequate response to the low CVP.

What is Cushing's Triad?

Increased SBP and decreased DBP (widening pulse pressure), bradypnea, and bradycardia

Atropine OD

Increased temp, Decreased LOC, Flushed face, and Increased thirst

Thyroid Storm

Increased temperature, Tachycardia, and HTN

Hypovolemia

Increased temperature, rapid/weak pulse, tachypnea, hypotension, anxiety, urine specific gravity > 1.030

Hypernatremia

Increased temperature, weakness, disorientation/delusions, hypotension, tachycardia Administer hypotonic solution

A 40 y/o male newly diagnosed with HTN is discussing risk factors with the nurse. The nurse talks about lifestyle changes with the patient and advises that the patient should avoid tobacco use. What is the rationale behind that advise to the patient?

Increases the risk of heart disease.

Osteomyelitis

Infectious bone disease; attain blood cultures and give antibiotics; then, if necessary, surgery is utilized to drain the abscesses

Side effect of Thyroid hormones:

Insomnia

LTB

Inspiratory stridor

The nurse manager on the orthopedic unit is reviewing a report that indicates that in the last month 5 clients were diagnosed with pressure ulcers. The nurse manager should:

Institute a quality improvement plan that identifies contributing factors, proposes solutions, and sets improvement outcomes.

Beta cells of the pancreas produce:

Insulin

The nurse understands that there are several mechanisms by which drugs can exert their action on the body, including which mechanisms? (Select all that apply.)

Interacting with specific receptors Inhibiting the action of a specific enzyme Altering metabolic chemical processes Nonspecific binding to a macromolecular receptor

Ammonia antedote:

Lactulose

Glaucoma:

Intraocular pressure is greater than the normal (22 mm Hg); treat with miotics (pilocarpine) to constrict. DO NOT treat with ATROPINE!

The nurse realizes that a drug administered by this route will require the most immediate evaluation of therapeutic effect.

Intravenous medications are not altered by first pass effect and enter the system quickly. Oral medications are absorbed in the stomach and small intestine, travel through the portal system, and are metabolized by the liver before they reach general circulation. Subcutaneous medications need to be absorbed into the bloodstream before entering the circulation to exert effect. Topical medications need to be absorbed through the skin before entering the blood stream and exerting an effect.

What is deep partial thickness (2nd degree) burn?

Involves the epidermal layer and the 2nd third of the dermal; it appears white-red and waxy, blanching occurs slowly or not at all; moderate edema and less blistering d/t more dead skin

Tardive Dyskinesia:

Irreversible--involuntary movements of the tongue, face, and extremities; may happen after prolonged use of antipsychotics

Wilm's tumor:

Is usually encapsulated above the Kidney's causing flank pain

If the Peritoneal Dialysis catheter (tenkhoff) was placed in the last 1-2 weeks:

It is ok to have: Abdominal cramps, blood tinged outflow, and leaking around the site CLOUDY OUTFLOW is NEVER NORMAL.

Iron - Nursing Implications

It is preferred but not necessary to be NPO for 8 hours

Creatinine Nursing Implications

It is preferred but not necessary to be NPO for 8 hours. A ratio of 20:1 BUN to creatinine indicates adequate kidney function

What is the phlebostatic axis?

It is the reference point on the chest that is used as a baseline for consistent tranducer height placement. It is the 4th intercostal point and midaxillary line.

What is stroke volume and it's normal range?

It is the volume of blood ejected from the heart with each beat. Normal is 50-100 ml/beat

What does the CVP measure?

It measures the filling pressure of the right side of the heart.

Medication of choice for bipolar:

Lithium

Stage II pressure Ulcer

Partial thickness skin loss involving epidermis, dermis, or both. The ulcer is superficial and presents clinically as an abrasion, blister, or shallow crater.

Take iron elixir with:

Juice or water... NEVER with MILK!!!

Meningitis

Kernig's Sign (leg flex then leg pain on extension), Brudzinski Sign (neck flex = lower leg flex)

Urine: Ketones - Nursing Implications: general - diabetes

Ketones spill over into the urine when blood glucose levels in dibetic patients are elevated. Ketonuria is associated with poorly controled diabetes.

Urine: Ketones - Nursing Implications: general - Children

Ketonuria may occur with acute febrile illnesses, especially in infants and children.

Buck's T4raction:

Knee immobility

Measles

Koplik's spots

DKA

Kussmauls breathing (Deep, Rapid RR)

Potassium - Abbreviation

K⁺

Kawasaki's

Leads to cardiac problems

A critical car nurse s caring for a patient with pulmonary artery catheter in place. What does this catheter measure that is particularly important in critically ill patients?

Left Ventricular preload

Red Blood Cell Count - what can cause a rise in numbers?

Living in a high altitude and exercize

Differentiate between the symptoms of left sided heart failure and right sided heart failure:

Left sided: pulmonary congestion due to back up of circulation in the left ventricle, Right sided: peripheral congestion due to back up of circulation in the right ventricle

Alcohol withdrawal:

Librium

Medication of choice for VTach:

Lidocaine

Leprosy

Lion face

The nurse notes that a patient's endotracheal tube (ET), which was at the 21-cm mark, is now at the 24-cm mark and the patient appears anxious and restless. Which action should the nurse take first?

Listen to the patient's lungs. rationale: The nurse should first determine whether the ET tube has been displaced into the right mainstem bronchus by listening for unilateral breath sounds. If so, assistance will be needed to reposition the tube immediately. The other actions also are appropriate, but detection and correction of tube malposition are the most critical actions.

With LEFT sided cardiac cath:

Look for CORONARY complications

With RIGHT sided cardiac cath:

Look for VALVE problems

Cystic Fibrosis Diet:

Low fat, high sodium, fat soluble vitamins (ADEK); treat with aerosal bronchodilators, mucolytics, and pancreatic enzymes

Low residue diet means:

Low fiber

A client with leukemia may have epitaxis b/c of:

Low platelets

PTB

Low-grade afternoon fever

In pH regulation, the (2) organs of concern are the:

Lungs and Kidneys

Angiotensin II:

Lungs--Potent Vasodilator

A woman has been diagnosed with breast cancer and is being treated aggressively with a chemotherapeutic regimen. As a result of this regimen she has an inablilty to fight infection due to the fact her bone marrow is unable to produce a sufficient amount of what?

Lymphocytes

High Density Lipoprotien - Lab Values: Adult

M: >45 F: >55

PArnate MArplan NArdil

MAIOs used for depression (PANAMA)

MAOIs that are used as antidepressants:

MAOIs which are used to treat depression all have an "arrrr" sound in the middle: Parnate, Marplan, and Nardil

How is CPP determined?

MAP - ICP

Contact Precautions:

MRSA, Respiratory, skin, wound enteric and eye infections

Hyperkalemia

MURDER - muscle weakness, urine (oliguria/anuria), respiratory depression, decreased cardiac contractility, ECG changes, reflexes

Hyperkalemia

MURDER: Muscle weakness, urine (oliguria/anuria), respiratory depression, decreased cardiac contractility, ECG changes, reflexes

PDA

Machine like murmur

Head Injury

Mannitol (osmotic diuretic) crystallizes at room temperature so ALWAYS use a FILTER set.

When the ventilator alarm sounds, the nurse finds the patient lying in bed holding the endotracheal tube (ET). Which action should the nurse take first?

Manually ventilate the patient with 100% oxygen. rationale: The nurse should ensure maximal patient oxygenation by manually ventilating with a bag-valve-mask system. Offering reassurance to the patient, notifying the health care provider about the need to reinsert the tube, and activating the rapid response team also are appropriate after the nurse has stabilized the patient's oxygenation.

Which nursing intervention will best enhance the absorption of an intramuscular injection?

Massaging the site increases circulation to the area and thus increases absorption. Cold will cause vasoconstriction and will not enhance absorption. Administration in the leg and the Z-track method will not enhance absorption.

Airborne Precautions:

Measles, Chicken Pox, and TB; client's need private room, negative airflow w/ 6-12 air exchanges/hr, and N95 usage

When caring for a patient who has an intraaortic balloon pump in place, which action will be included in the plan of care?

Measure the patient's urinary output every hour. rationale: Monitoring urine output will help determine whether the patient's cardiac output has improved and also help monitor for balloon displacement. The head of the bed should be no higher than 30 degrees. Heparin is used to prevent thrombus formation. Limited movement is allowed for the extremity with the balloon insertion site to prevent displacement of the balloon.

Po2

Measures the absolute amt. of oxygen in the blood

Activated Partial Thromboplastin Time - definition

Measures time required for clot formation; used to evaluate clotting factors

First sign of Cystic Fibrosis:

Meconium ileus at birth. Baby is inconsolable, will not eat, and passing meconium

What are the 10 Rights of Medication Administration

Medication Assessment Dose Documentation Route Patient Education Timing Evaluation Refusal (MADDRPETER)

When a client is in DISTRESS:

Medication administration is RARELY the PRIORITY

Kernig's/ Brudzinski's signs

Meningitis signs

Ethambutol:

Messes with your eyes

The pharmacist states that the client's biotransformation of a drug was altered. What does the nurse realize has affected the drug?

Metabolism connotes a breakdown of a product. Biotransformation is actually a more accurate term because some drugs are actually changed into an active form in the liver in contrast to being broken down for excretion.

How does 1+ pitting edema appear?

Mild edema (0"-1/4" indentation), disappears rapidly

Alkaline Ash Diet:

Milk, veggies, rhubarb, and salmon

Treatment for Addison's:

Minearl corticoids

TIA

Mini-stroke with NO DEAD brain tissue

How does 2+ pitting edema appear?

Moderate Pitting (1/4"-1/2" indentation), disappears in 10-15 seconds

When giving Kayexalate:

Monitor for DEHYDRATION (potassium has an inverse relationship with sodium)

When a client is on Nitroprusside:

Monitor thiocynate (cyanide) levels. Normal value should be 1; > 1 is heading towards toxicity.

Cushing's Syndrome

Moon face appearance and buffalo hump

Autonomic Dysreflexia/Hyperreflexia

S/S: Pounding HA, profuse sweating, nasal congestion, goose flesh, bradycardia, hypertension Place client in SITTING position (elevate the HOB) FIRST before any other intervention

A client is complaining of pain rated "10" on a scale of 1 to 10. The nurse has several choices of pain medication to administer. Which order is the best for the nurse to administer at this time?

Morphine sulfate 1 mg IV (intravenous) When a drug is administered intravenously, it does not need to be absorbed because it is placed directly into general circulation.

A 45 y/o adult male patient is admitted to emergency following a fall from a tree stand in which he sustained a large laceration to the occipital area of his head. The patient relates that prior to the fall, he had developed unrelieved chest pain that was present for approximately 20 minutes before he fell and is still present. The DR. diagnoses the patient with an MI and deep laceration to the head. To minimize cardiac damage, what physician's order will the nurse expect to see for this patient?

Morphine sulphate, Oxygen administration, and bed rest

Normal heart sound in CHF:

S3

What heart sound is NOT normal during a AMI?

S3

Akathisia:

Motor restlessness, need to keep going; treat with antiparkinson medications; can be mistaken for agitation

Hypokalemia

Muscle weakness, dysrhythmias Increase K (raisins, bananas, apricots, oranges, beans, potatoes, carrots, celery)

Hypercalcemia

Muscle weakness, lack of coordination, abdominal pain, confusion, absent tendon reflexes, sedative effect on CNS

Blood Tests for MI:

Myoglobin, CK, and Troponin

Glucose - Lab Values: Newborn

N 30-60 I 40-90 Preemie - 20-60

Addisonian Crisis

N/V, confusion, abdominal pain, extreme weakness, hypoglycemia, dehydration, hypotension

Neutropenic clients:

NO: Flowers, Live Vaccines, or Fresh Fruits

Bilirubin - Proir to pulling a sample the nurse should ensure that the patient is:

NPO except for water for 8-12 hours proir to drawing a sample

Cardiac Cathether:

NPO for 8-12 hours, empty bladder, pulses, inform client that they may feel heart palpitations or desire to cough when the dye is given. S/P: V/S, keep leg straight, and bedrest for 6-8 hours.

What mechanism is involved in the reduction of pain through the administration of NSAID's

NSAID's act by a peripheral mechanism at the level of damaged tissue by inhibiting protaglandin synthesis and other chemical mediators involved in pain transmission.

Give with food:

NSAIDS Corticosteroids Bipolar medications Cephalosporins Sulfanomides

"The client presents in the emergency room with constricted pupils, slurred speech, drowsiness, and respirations of 8/min. The person who accompanied the client to the emergency room reports the client had taken an unknown quantity of meperidine (Demerol) tablets 30 minutes earlier. Which medication should the nurse anticipate giving the client?" "Methadone Phenytoin (Dilantin) Naloxone (Narcan) Diazepam (Valium)

Naloxone (Narcan)

Hyponatremia

Nausea, muscle cramps, increased ICP, muscular twitching, convulsion Administer osmotic diuretics and fluids

Sodium - Abbreviation

Na⁺

Define myocardial infarction:

Necrosis of the heart muscle due to poor perfusion of the heart

Cold and Clammy

Needs some Candy (hypoglycemia)

Urine: Ketones - Lab Values:

Negative

Urine: Leukocyte esterase - Lab Values

Negative

Urine: Nitrates - Lab Values

Negative

Dilantin level:

Normal is 10-20; can cause GINGIVAL HYPERPLASIA

What is the normal range for Cardiac Index?

Normal is 2.4-4 L/min/m^2

Aminoglycosides (like Vancomycin) cause:

Nephrotoxicity and ototoxicity

Normal differential?

Neutrophil: 55-70% (2500-8000), Lymphocytes: 20-40% (1000-4000), Monocytes: 2-8% (100-700), Eosinophils: 1-4% (50-500), Basophils 0.5-1% (25-100)

Creatinine - Lab Values: Newbor

Newborn .2-.4 Infant .5-1

Sodium - Lab Values: Infant

Newborn 134-144 Infant 134-150

Cholesterol - Lab Values: Newborn

Newborn: 53-135 Infant: 70-175

Thyroxine (T-4) - Nursin Implications : Newborn

Newborns screened to prevent hypothyroidism to prevent mental retardation - a heel stick is used to collect the blood.

How to draw up Insulin:

Nicole Richie RN: Air into NPH, then air into Regular, draw up regular then draw up NPH.

Pemphigus Vulgaris

Nikolsky's sign (separation of epidermis caused by rubbing of the skin)

Unstable Angina is NOT relieved by:

Nitroglycerin

Angina (low oxygen to heart tissues):

No DEAD heart tissue

RHUBARB, SARDINES, COLLARD GREENS

Non dairy sources of calcium include

Orange Tag in Triage:

Non-emergent Psych

Stage I pressure Ulcer

Nonblanchable erythema of intact skin, the heralding lesion of skin ulceration. In individuals with darker skin, discoloration of the skin, warmth, edema, induration, or hardness may also be indicators

Urine: Biliruben - Lab Values:

None

Urine: Casts- Lab Values

None

Urine: Crystals - Lab Values

None

A patient is scheduled to have a fecal occult blood test. Before the test, the nurse should instruct the patient to avoid:

Nonsteroidal anti-inflammatory drugs

When caring for the patient with a pulmonary artery pressure catheter, the nurse notes that the PA waveform indicates that the catheter is in the wedged position. Which action should the nurse take?

Notify the health care provider. rationale: When the catheter is in the wedge position, blood flow past the catheter is obstructed, placing the patient at risk for pulmonary infarction. A health care provider or specially trained nurse should be called to reposition the catheter. The other actions will not correct the wedging of the PA catheter.

The nurse is caring for a patient who underwent percutaneous lithrotripsy earlier in the day. In this procedure, an ultrasonic probe inserted into a nephrostomy tube into the renal pelvis generates ultra-high-frequency sound waves to shatter renal calculi. What instruction should the nurse give the patient?

Notify the physician about cloudy of foul-smelling urine

Myelogram

Npo 4-6hr, allergy hx, phenothiazines, cns depressants, and stimulants withheld 48hr prior, table will be moved to various postions during test. Post- neuro q2-4, water soluble HOB up, oil soluble HOB down, oral analgesics for h/a, encourage po fluids, assess for distended bladder, inspect site.

The nurse is caring for a patient diagnosed with Hashimoto's thyroiditis. When assessing this patient, what symptom would the nurse expect in a patient with hypothyroidism?

Numbness and tingling in the fingers

Right eye:

OD

Left eye:

OS

Both eyes:

OU

Admission height and weight is whose responsibility

OUR RESPONSIBILITY (Basis for all drug calculations)

A 47 y/o male patient calls the nurse and asks about risk factors of HTN. What should the nurse list as risk factors for primary HTN.

Obesity, high intake of sodium and saturated fat

Urine: Biliruben - Nurising Implications - general

Obstruction of the gbile duct by a gall stone causes conjugated hyperbulirubinemia, and unlike the unconjugated form, conjugated bilirubin is water soluable and can be excreted into the urine. Bilirubin is not stable in urine, especially when exposed to the light.

Allen's Test:

Occlude both ulnar and radial artery until the hand blanches and then release the ulnar. If hand pinks up, then the ulnar artery is good and you can carry on with ABG/radial stick as planned. ABGs must be placed on ice and taken to the lab.

Physiological jaundice:

Occurs after 24 hours

Pathological jaundice:

Occurs before 24 hours ad lasts 7 days.

What is Cranial Nerve I?

Olfactory; smell

Pyloric Stenosis

Olive like mass

A patient with a liver mass is undergoing a percutaneous liver biopsy. After the procedure the nurse assists the physician in position the patient. What position should they position the patient in?

On the right side with a pillow under the costal margin

Epispadias:

Opening of the urethra is on the dorsal (front) surface of the penis

Methadone:

Opioid analgesic used to detoxify/treat pain in narcotic addicts

What is Cranial Nerve II?

Optic; sight

In an ECG reading which wave represents depolarization of the atrium?

P wave

MAOIs that are used to treat depression:

PANAMA: PA--Parnate NA--Nardil MA--Marplan

The nurse is preforming a shift assessment on a patient with aldosteronism. What would the nurse recognize that the kidney's response to this condition would lead to?

Polyuria

DM

Polyuria, polydypsia, and polyphagia

Partial Thromboplastin Time - Abbreviation

PTT

DEAD tissues can NOT have:

PVCs; if left untreated, PVCs can lead to VF

McBurney's Point:

Pain in the RLQ indicative of appendicitis

Placenta Abruptio:

Pain is present, but no bleeding.

Murphy's Sign:

Pain with palpation of the gall bladder area seen with Cholecystitis

Priaprism:

Painful erection lasting longer than 6 hours

Bladder CA

Painless hematuria

Hodgkin's DSE/Lymphoma

Painless, progressive enlargement of the spleen and lymph tissues; Reedstenberg Cells

With chronic pancreatitis:

Pancreatic enzymes are given with meals

A 55 y/o male with acute pancreatitis is being admitted. What is the critical care nurse aware of concerning the common description of acute pancreatitis?

Pancreatic enzymes digest the pancreas.

Morphine is contraindicated in:

Pancreatitis (it causes spasm of the Sphincter of Oddi)

Compartment Syndrome:

Paresthesias and increased pain are classic symptoms. Neuromuscular damage is irreversible 4-6 hours after onset.

The pharmacology instructor is talking with the pre-nursing class about drugs used to control HTN. Propranolol hydrochloride, like other anti-hypertensive drugs, has side effects. Patients are often anxious about the side effects of anti-hypertensives. A nursing diagnosis of Risk for noncompliance with therapeutic regimen r/t side effects of prescribed therapy would be evaluated how?

Patient takes medications as prescribed and reports any side effects.

Separation anxiety:

Peaks in toddlerhood

Coffee-Brown Emesis:

Peptic Ulcer

Flagyl, Prilosec and Biaxin

Peptic ulcers caused by H. pylori meds

Isoniazid causes:

Peripheral neuritis

INH can cause:

Peripheral neuritis; take Vitamin B6 to prevent

Glaucoma clients lose:

Peripheral vision

Sign of fat embolism:

Petechiae and treated with Heparin

Dengue

Petechiae or + Herman's sign

A basic concept of pharmacology that the nurse must understand is how the drug influences cell physiology. What is the term for this concept?

Pharmacodynamics:Pharmacodynamics refers to what the drug does to the body; that is, how it influences cellular physiology. Pharmacokinetics is the study of what the body does to the drug. Pharmacotherapeutics refers to the study of the therapeutic use of drugs. Pharmacology is the study of drugs.

Parkinson's

Pill-Rolling tremors

Client with Heat Stroke

Place client FLAT with LEGS ELEVATED

Tube feeding with Decrease LOC (and oil-based Myelogram)

Place client in FLAT SUPINE (to prevent HA and leaking of CSF)

Prolapsed Cord

Place client in KNEE-CHEST position or TRENDELENBURG

Thyroidectomy

Place client in LOW or SEMI-FOWLER's Support the head, neck, and shoulders

Pancreatitis:

Place client in the fetal position, NPO, gut rest, prepare antecubital site for PICC because they'll probably be getting TPN/Lipids.

Woman in Labor w/ Un-reassuring FHR (late decels, decreased variability, fetal bradycardia)

Place client on LEFT side Administer oxygen Stop pitocin Increase IV fluids

During Internal Radiation

Place client on bedrest while implant is in place

Shock

Place client on bedrest with extremities elevated 20 degrees, knees straight, head slightly elevated (modified Trendelenburg)

Client's with a LOWER AMPUTATION:

Place in a PRONE position

Treatment of Cord Compression

Place the mother in the TRENDELENBERG position because this removes pressure off the presenting part of the cord (if her head is down, the baby is not longer being pulled out of the body by gravity) If the cord is prolapsed, cover it with sterile saline gauze to prevent drying of the cord and to minimize infection.

A patient has undergone surgery for oral cancer and has just been extubated. When the anesthesia wears off, what nursing action would promote comfort and facilitate breathing for this patient?

Placing the patient in Fowler's position.

Heparin prevents:

Platelet aggregation

Chest tubes are placed in the:

Pleural Space

Cerebral Palsy:

Poor muscle control due to birth injuries and/or decreased oxygen to the brain tissue

After Infratentorial Surgery (incision at nape of the neck)

Position client FLAT and LATERAL on either side

Enema Administration

Position client in LEFT side-lying (Sim's) with knee flexed

Infant with Cleft Lip

Position client on back or in infant seat to prevent trauma to suture line. While feeding, hold client in upright position.

Myringotomy

Position client on side of the AFFECTED ear after surgery (allows drainage of secretions)

Urine: Leukocyte esterase - Nursing Implications - general

Positive results indicate UTI

Infant with Spina Bifida

Postion client PRONE (on abdomen) so that the sac does not rupture

Salt Substitutes may contain:

Potassium

Autonomic Dysreflexia

Potentially life threatening emergency. Interventions: Elevate the HOB to 90 degrees; Loosen restrictive clothing; Assess for bladder distension and bowel impaction (triggers); Administer antihypertensives

Preload affects amount of blood that goes to the R ventricle. Afterload is the resistance the blood has to overcome when leaving the heart.

Preload vs Afterload

The nurse has administered several oral medications to the client which may alter the rate of absorption?

Presence of food in the stomach pH of the stomach Pain Explanantion: The presence of food in the stomach usually decreases absorption of drugs but may increase absorption for a few specific medications. The pH of the stomach affects absorption of drugs dependent on the pH of the drug. Alkaline drugs are absorbed more readily in an alkaline environment, and acidic drugs are absorbed more readily in an acidic environment. The form of the drug also affects absorption, with liquid drugs being absorbed the fastest and enteric-coated tablets the slowest. Pain can affect absorption by slowing gastric emptying time. Position will not influence absorption. Amount of saliva will not influence absorption.

PTU and Tapazole

Prevention of thyroid storm

Triiodothyronine (T-3) - Nursing Implications - pregnancy

Primary Diagnosis hyperthyroidism. Total T3 Values are increased in pregnancy due to serum protiens being increased.

A young woman is contacted by a public health nurse and informed that one of her sexual contacts has tested positive for HIV. The young woman comes into the clinic to be tested. She tells the nurse that she has heard that people who are infected with HIV have a period when they test negative even though they are infectious. What is this period called?

Primary infection

Gout Medications

Probenecid (Benemid), Colchicine, Allopurinol (Zyloprim)

Gout meds

Probenecid (Benemid), Colchicine, Allopurinol (Zyloprim)

Hematocrit - How can a tournaquet alter values?

Prolonged stasis from vasoconstrictipon secondary to a tournaquet.

Heparin antedote:

Protamine sulfate

The nurse is providing patient education for a patient with peptic ulcer disease secondary to chronic nonsteroidal anti-inflammatory drug (NSAID) use. The patient has been newly prescribed misoprostol (cytotec). What would the nurse be most accurate in informing the patient about the drug?

Protects the stomach's lining

Urine: Specific Gravity - Nursing Implications - general

Protienuria is an indicator of renal disease.

Down Syndrome

Protruding tongue

What would the nurse recognize as a common goal of discharge in all care settings?

Providing continuity of care for the client

contact

RSV, head lice (with isolation up to 24 hours), rotavirus, impetigo (for 24 hours), CDIF (MUST WASH WITH SOAP AND WATER)

Heroin withdrawal in a NEONATE:

S/S: Irritable and poor sucking

Diptheria

Pseudo membrane formation

Manchausen Syndrome:

Psychiatric disorder that causes an individual to self-inflict injury or illness or to fabricate symptoms of physical or mental illness--in order to receive medical care or hospitalization.

What is PAS?

Pulmonary artery systolic pressure, reflects the highest pressure generated by the right ventricle during systole

When caring for a patient with pulmonary hypertension, which parameter will the nurse monitor to evaluate whether treatment has been effective?

Pulmonary vascular resistance (PVR) rationale: PVR is a major contributor to pulmonary hypertension, and a decrease would indicate that pulmonary hypertension was improving. The other parameters also may be monitored, but do not directly assess for pulmonary hypertension.

brown green or yellow

Purulent

Metabolic Alkalosis

Remember: From the MOUTH (vomitus)

In an ECG reading what complex represents depolarization of the ventricle?

QRS complex

Woman in Labor w/ Un-reassuring FHR

R (late decels, decreased variability, fetal bradycardia, etc) --> turn on left side (and give O2, stop Pitocin, increase IV fluids)

LVNs can be delegated tasks BUT . . .

R.N. still is accountable and responsible for it

Parkinson's:

RAT: Rigidity Akinesia (loss of muscle movement) Tremors Treat with Levodopa

Parkinson's disease

RAT: rigidity, akinesia (loss of muscle mvt), tremors. Treat with levodopa.

Urine RBC Casts - Nursing Implications

RBC casts suggest glomerulonephritis interstitial nephritis, acute necrosis, pyelonephritis, renal trauma, or renal tumor. Strenour physical Exercize may also cause RBC casts.

Client's with HALLUCINATIONS:

REDIRECT them

Cor Pulmonae

RIGHT Side heart failure caused by LEFT ventricular failure--causing edema and JVD

Cor Pulmonale (s/s: fluid overload) is:

RIGHT sided heart failure caused by PULMONARY DISEASE; may also occur with BRONCHITIS or EMPHYSEMA.

Appendicitis:

RLQ pain and rebound tenderness

Appendicitis (inflammation of the appendix); pain is located:

RLQ w/ Rebound tenderness

If assigned task doesn't meet expectations

RN must step in and teach. Give credit and praise when due. Offer observations and share concerns and ask for their feedback, too.

describe the VS of a normal newborn:

RR 30-50/m HR 120-140/m BP 60-80/40-50 axillary T <97.6/36.5 may be caused by prematurity, infection, or low environmental temp

Urine: Crystals - Nursing Implications - general - testing

Radiographic contrast media may cause precipation of urinary crystals

What is the peak action time of the rapid acting regular insulin, intermediate acting insulin and long-acting insulin?

Rapid: 2-4 hours, Immediate acting: 6-12 hours, Long acting: 14-20 hours

What should be always be ready to do with our assigned clients

Re-evaluate and re-assign care as the client's needs or the nursing team's abilities change

When drawing up regular insulin and NPH together:

Remember: RN--Regular before NPH

Appendicitis

Rebound tenderness

Ulcerative Colitis

Recurrent bloody diarrhea

Pernicious Anemia

Red beefy tongue

Koplick's Spots:

Red spots with blue center characteristic of PRODROMAL stage of Measles--usually in the MOUTH.

Pernicious Anemia

Red, Beffy tongue; will take Vit.B12 for life!

Rifampin:

Red-orange tears and urine and contraceptives DO NOT work as well.

BPH

Reduced size and force of urine

A patient with primary HTN comes into the clinic c/o a change in vision- ie, blurring and decreased visual acuity. The nurse is aware that these symptoms could be indicative of what?

Retinal blood vessel damage

An ill child:

Regresses in behavior

Activated Partial Thromboplastin Time - What it is used for?

Regulating Heprin Therapy. Theraputic Range is 1.5-2.5 times normal or control

Partial Thromboplastin Time - What it is used for?

Regulating Heprin Therapy. Theraputic Range is 1.5-2.5 times normal or control

Cushings ulcers:

Related to BRAIN injury

Cushing's Triad:

Related to INCREASED ICP (HTN, bradycardia, and irregular respirations)

HYPERthyroidism:

Remember Michael Jackson in Thriller: SKINNY, NERVOUS, BULDGING EYES, UP ALL NIGHT, and HEART BEATING FAST

FHR Patterns

Remember VEAL CHOP: V C -- Variable decels; Cord compression E H -- Early decels; Head compression A O -- Accels; Okay, no problem L P -- Late decels; Placental insufficiency

Metabolic Acidosis

Remember: From the ASS (diarrhea)

Hydroxyurea (for sickle cell):

Report GI symptoms immediately; could be a sign of toxicity

Afterload is the:

Resistance that the blood has to overcome when leaving the heart

Hyperventilation:

Respiratory alkalosis

In delegating, we should always keep in mind

Right task, circumstances, person, direction, supervision AND evaluation. We are the manager of client care and we should be thinking cost-effective, too

Hypoxia

Restless, anxious, cyanotic, tachycardia, tachypnea (monitor ABGs)

Functions of Sleep

Restoration, reducing fatigue,stabilizing mood, improving blood flow to the brain, increasing protein synthesis, maintaining the disease-fighting mechanisms of the immune system,promoting cellular growth and repair, improving the capacity for learning and memory storage

Prior to an occult stool test:

Restrict Vitamin C (cantaloupe) as it causes a false positive for occult blood

Pancuronium:

Reversal agent: Neostigmine/Atropine (anticholinergic)

You are admitting a patient to your unit with diverticular diesease. What questions should the nurse ask during her assessment?

Review of dietary habits, about tenesmus, onset and duration of pain

ASA can cause:

Reye's syndrome (encephalopathy) when given to children

Rh- negative mothers receive:

Rhogam to protect their next baby

Non-dairy sources of Calcium:

Rhubarb,sardines, and collard greens

Cholera

Rice watery stool

What are the 5 rights of delegation?

Right task, circumstance, person, direction/communication and supervision

A patient with primary HTN complains of dizziness with ambulation. The patient is currently on an alpha-adrenergic blocker. The nurse assesses postural hypotension. When teaching this patient about risks associated with postural hypotension, what should the emphasis be placed on?

Rising slowly from a lying or sitting position

Tetanus

Risus sardonicus

Typhoid

Rose spots in abdomen

Head and Neck= 9% Each upper ext= 9% Each lower ext= 18% 10Front trunk= 18% Back trunk= 18% Genitalia= 1% ?

Rules of 9's for burns

Pneumonia

Rusty sputum

Zoloft:

S/E: Agitation, sleep disturbance, and dry mouth

Clozapine:

S/E: Agranulocytosis, tachycardia, and seizures

Air/Pulmonary Embolism

S/S: CP, SOB, Tachycardia, Pale/Cyanotic, Sense of Impending Doom Intervention: Place client on LEFT side and LOWER the HOB.

Hypernatremia (greater than 145):

SALT: Skin flushed Agitation Low grade fever Thirst

What route is the MMR given?

SQ

A pt has exhausted the number of hospital days allowed by the reimbursement provided by Medicare and is to be discharged from the hospital's acute unit with an open infected abdominal wound requiring IV antibiotics and dressing changes. The nurse case manager plans transfer of this pt to a :

SUBACUTE CARE UNIT

Cystic Fibrosis

Salty Skin

HCO3 - Lab Values: Nursing Implications

Sample must be heprinized. Specimen must be iced for transport. No air bubbles allowed in specimen. Direct pressure should be applied to the puncture site.

O2 Saturation - Lab Values: Nursing Implications

Sample must be heprinized. Specimen must be iced for transport. No air bubbles allowed in specimen. Direct pressure should be applied to the puncture site.

Pco2 - Nursing Implications

Sample must be heprinized. Specimen must be iced for transport. No air bubbles allowed in specimen. Direct pressure should be applied to the puncture site.

Po2 - Lab Values: Nursing Implications

Sample must be heprinized. Specimen must be iced for transport. No air bubbles allowed in specimen. Direct pressure should be applied to the puncture site.

pH of Blood: Nursing Implications

Sample must be heprinized. Specimen must be iced for transport. No air bubbles allowed in specimen. Direct pressure should be applied to the puncture site.

Creatine Phosphokinase - Nursing Implications

Samples must be stored proir to running test

blood

Sanguinous

Intussusception

Sausage shaped mass; Dance Sign (empty portion of RLQ)

abrasion

Scraping or wearing away of the skin by friction; irritation

A nursing student asks when you formula feeding can begin on a patient you are caring for. The patient has just returned to your floor after having a PEG tube placed. What would be your best response.

Second day after placement of tube

A nurse is caring for a pt who has a pressure ulcer that is treated with debridement, irrigations, and moist gauze dressings. How should the nurse anticipate healing to occur?

Secondary intention - Wounds that occur from trauma, ulceration, and infection have large amounts of exudate and wide, irregular wound margins with extensive tissue loss These wounds may have edges that can not be approximated (brought together) .

ARDS (fluids in alveoli) and DIC (disseminated intravascular coagulation) are always:

Secondary to something else (another disease process)

Autonomic dysreflexia (life threatening inhibited sympathetic response of nervous system to a noxious stimuli) occurs in client with spinal cord injuries at:

T-7 or above and is usually caused by a FULL BLADDER.

A nurse is caring for a 16 y/o who as sickle cell anemia. The nurse is concerned about substance abuse in this patient. What would the nurse encourage the patient to do to prevent substance abuse of analgesics in the patient?

Seek care from a single provider for pain relief

William's position:

Semi-Fowler's with knees flexed (knee gatch) to relieve lower back pain.

Post-Thyroidectomy

Semi-Fowler's; prevent head/neck flexion/hyperextension; have trach ready at the bedside

Droplet Precautions:

Sepsis, scarlet fever, streptococcal pharyngitis, parovirus B19, pneumonia, pertusis, influenza, diptheria, epiglottis, rubella, mumps, meningitis, mycoplama and adenovirus.

blood+water

Serosanguinous

watery drainage

Serous

A student asks the pathophysiology instructor what causes ascites. What would the instructor tell the student is involved in the physiologic process that causes ascites? A. Sodium and water retention B. A damaged liver C. Insufficient renal flow D.Increased lymphatic flow E. Decreased synthesis of Immunoglobulin G

Sodium and Water retention, A damaged liver, Increased lymphatic flow

How does 3+ pitting edema appear?

Severe Pitting (1/2" - 1" indentation), disappears in 1-2 minutes

How does 4+ pitting edema appear?

Severe Pitting (>1" indentation), may be present after 5 minutes

A 37 y/o male patient presents at the emergency department complaining of nausea and vomiting and severe abdominal pain. While the nurse is assessing the patient, the patient;s wife informs the nurse that the patient had ingested 24 ounces of alcohol last evening. The patients abdomen is rigid, and there is bruising to the patients flank. What is the patient exhibiting signs of?

Severe pancreatitis with possible peritonitis.

Crisis Intervention:

Short Term

Do NOT give Demerol to client's with:

Sickle Cell Crisis

What diagnostic test is used to determine thyroid activity?

T3, T4

6-7 months

Sits at 6 and waves bye-bye

Dunlap Traction:

Skeletal or skin

VCHIPS V - varicella zoster C - cutaneous diphtheria H - herpez simplex I - impetigo P - pediculosis S - scabies

Skin Infections

Best way to warm a newborn:

Skin to skin contact covered with a blanket and on the mother's chest

Myxedema/Hypothyroidism

Slowed physical and mental function; sensitivity to the cold; skin and hair are dry

Uremic fetor:

Smell of urine on the breath

Aldosterone attracts:

Sodium

Urine: Specific Gravity - Nursing Implications - general

Specific Gravity is a measurement of hydration status, with overhydration the urine is more dilute, with underhydration the urine is more concentrated.

Liver Cirrhosis

Spider-like varices

You are caring for a patient who is scheduled for a gastroscopy. What preparation is needed for this test?

Spray or gargle the back of the throat with local anesthetic

what can LVNs help us with

Stable clients only. Can handle data collection but never the actual steps in nursing process. They can implement tasks on careplan and SPECIFIC tasks for us on our unstable client. Can't start but can remove IVs

A patient has a glomular filtration rate of 43 mL/min/1.73m2. Based upon this GFR, the nurse interprets that the patient's chronic kidney disease is at what stage?

Stage 3

An 82 yo man is being cared for at home by his family. A pressure ulcer on his right buttock measures 1cm x 2 cm x 0.8 cm in depth, and pink subcutaneous tissue is completely visible on the wound bed. Which stage would the nurse document on the wound assessment form?

Stage III

8-9 months:

Stands straight at eight

By what authority may nurses delegate nursing care to others?

State Nurse Practice Act

Malaria

Stepladder like fever with chills

What are the complications that can occur following gastric surgery?

Stomal ulcers, pneumonia, metabolic imbalances

Premature ventricular contractions (PVCs) occur while the nurse is suctioning a patient's endotracheal tube. Which action by the nurse is best?

Stop and ventilate the patient with 100% oxygen. rationale: Dysrhythmias during suctioning may indicate hypoxemia or sympathetic nervous system stimulation, and the nurse should stop suctioning and ventilate the patient with 100% oxygen. Lowering the suction pressure will decrease the effectiveness of suctioning without improving the hypoxemia. Because the PVCs occurred during suctioning, there is no need for antidysrhythmic medications (which may have adverse effects) unless they recur when the patient is well oxygenated.

* INH can cause peripheral neuritis, take Vit B6 to prevent also hepatotoxic 14 Rifampin - Red orange tears and urine, also contraceptives don't work as well * Ethambutol - messes with your Eyes

TB meds side effects

Epinephrine is given via:

TB syringe

Kawasaki Syndrome

Strawberry tongue

A 42 y/o women comes to the clinic complaining of intermittent urinary incontinence when she sneezes. The clinic nurse is aware this is what type of incontinence?

Stress incontinence

CVA

Stroke with DEAD brain tissue

TPN is given via a:

Subclavian line

What defines pancreatitis as acute?

Sudden onset with high risk of mortality

Hot and Dry:

Sugar High (hyperglycemia)

airborne

TB with pulmonary involvement

Bethamethasone (celestone):

Surfactant--medication for lung expansion

Client's with Asthma and Arthritis:

Swimming is the best exercise

To determine the effectiveness of medications that a patient has received to reduce left ventricular afterload, which hemodynamic parameter will the nurse monitor?

Systemic vascular resistance (SVR) rationale: Systemic vascular resistance reflects the resistance to ventricular ejection, or afterload. The other parameters will be monitored, but do not reflect afterload as directly.

The nurse is caring for a patient receiving a continuous norepinephrine (Levophed) IV infusion. Which patient assessment information indicates that the infusion rate may be too high?

Systemic vascular resistance (SVR) is elevated. rationale: Vasoconstrictors such as norepinephrine (Levophed) will increase SVR, and this will increase the work of the heart and decrease peripheral perfusion. Bradycardia, hypotension, and low PAWP are not associated with norepinephrine infusion.

Glomerulonephritis

Take V/S q 4 hours and attain daily weights

Thoracentesis Prep:

Take V/S, shave area around needle insertion site, position client with arms on pillow/over bed table/lying on side, no more than 1000 cc at one time. S/P: Listen for bilateral breath sounds, V/S, check leakage, and apply a sterile dressing.

Zocor (anti-hyperlipidemia):

Take on a empty stomach to enhance absorption; report any unexplained muscle pain--especially fever

Urine: Ketones - Nursing Implications: general - testing

Tesing for ketones can be preformed immediately after urine collectiomn. Dip reagent stick into the urine specimen. Read the strip in 15 seconds by comparing it to a color chart.

Vermox:

Take with a high fat diet (increases absorption)

Amphogel and Renegal:

Take with meals.

Milieu Therapy:

Taking care of the client/environment

Shilling Test:

Test for pernicious anemia (how well one absorbs Vitamin B12.

Guthrie Test:

Tests for PKU; baby should have eaten source of protein first

If we identify a weakness of our NAP, what do we do to remedy that?

Teach, teach, teach and DOCUMENT WHAT WE TAUGHT -- it's our responsibility

If a staff member performs your assigned tasks which were not delegated to them and a problem occurs, what do you do

Teach, teach, teach and document what you taught. ALWAYS fill out an incident report and then go home and document the incident for yourself. The hospital will protect its interests and you need to protect yours.

Myasthenia Crisis: a positive reaction to Tensilon--will improve symptoms Cholinergic Crisis: caused by excessive medication-stop med-giving Tensilon will make it worse

Tensilon test outcomes for Myasthenia Gravis

When a client comes in and is in active labor:

The FIRST action of the nurse is to listen to FHR/tone

The impact of preload on contractility is explained by what law?

The Frank-Starling Law sometimes called just the Starling Law

DO NOT give digoxin to children if:

The HR < 100

When the charge nurse is evaluating the care that a new RN staff member provides to a patient receiving mechanical ventilation, which action by the new RN indicates the need for more education?

The RN positions the patient with the head of bed at 10 degrees. rationale: The head of the patient's bed should be positioned at 30 to 45 degrees to prevent ventilator-acquired pneumonia. The other actions by the new RN are appropriate.

When the nursing supervisor is evaluating the performance of a new RN, which action indicates that the new RN is safe in providing care to a patient who is receiving mechanical ventilation with 10 cm of peak end-expiratory pressure (PEEP)?

The RN uses a closed-suction technique to suction the patient. rationale: The closed-suction technique is suggested when patients require high levels of PEEP to prevent the loss of PEEP that occurs when disconnecting the patient from the ventilator. Suctioning should not be scheduled routinely, but it should be done only when patient assessment data indicate the need for suctioning. Taping connections between the ET and the ventilator tubing would restrict the ability of the tubing to swivel in response to patient repositioning. Ventilator tubing changes increase the risk for ventilator-associated pneumonia (VAP) and are not indicated routinely.

A patient with respiratory failure has hemodynamic monitoring and is receiving mechanical ventilation with peak end-expiratory pressure (PEEP) of 10 cm H2O. Which information indicates that a change in the ventilator settings may be required?

The arterial line shows a blood pressure of 90/46. rationale: The hypotension indicates that the high intrathoracic pressure caused by the PEEP may be decreasing venous return and cardiac output (CO). The other assessment data would not be caused by mechanical ventilation.

Diabetic Ketoacidosis (DKA):

The body is breaking down fat instead of sugar for energy. Fats leave ketones (acids) that cause the pH to decrease

A new nurse on the cardiac unit is taking a class in the anatomy and physiology of the heart. What does the nurse learn is a cardiac action potential?

The cycle of depolarization and repolarization

Radioactive iodine

The key word here is flush. Flush substance out of body w/3-4 liters/day for 2 days, and flush the toilet twice after using for 2 days. Limit contact w/patient to 30 minutes/day. No pregnant visitors/nurses, and no kids.

The nurse is administering medications to a client with chronic renal failure. What is a priority action of the nurse?

The kidneys are responsible for the majority of drug excretion. With excretion impaired, the medication can remain in the system longer and there is more chance for toxicity to develop.

Which assessment data obtained by the nurse when caring for a patient with a left radial arterial line indicates a need for the nurse to take action?

The left hand is cooler than the right hand. rationale: The change in temperature of the left hand suggests that blood flow to the left hand is impaired. The flush system needs to be changed every 96 hours. A mean arterial pressure (MAP) of 75 mm Hg is normal. Flush systems for hemodynamic monitoring are set up to deliver 3 to 6 mL/hour of flush solution.

HCO3

The metabolic component of pH. It determines how well the kidneys are working to maintain adequate levels in the body system.

During the preoperative interview the nurse obtains information about the client's medication history. Which of the following is not necessary to record about the client? 1) Current use of medications, herbs, and vitamins. 2) Over the counter medication use in the last 6 weeks. 3) Steroid use in the last year. 4) Use of all drugs taken in the last 18 months.

The nurse does not need to ask about all drugs used in the last 18 months unless the client is still taking them. The nurse does need to know all drugs the client is currently taking, including herbs and vitamins, over the counter medications such as aspirin taken in the past 6 weeks, the amount of alcohol consumed, and illegal use of drugs, because these can interfere with the anesthetic and analgesic agents. Steroid use is of concern because it can suppress the adrenal cortex for up to 1 year, and supplemental steroids may need to be administered in times of stress such as surgery.

*Thyroid storm is HOT (hyperthermia) *Myxedema coma is COLD (hypothermia)

Thyroid storm vs myxedema crisis

A patient is prescribed corticosteroid therapy. What would be important information for the nurse to give the patient who is prescribed corticosteroid therapy?

The patient is at in increased risk for developing infection.

When the nurse is weaning a patient who has chronic obstructive pulmonary disease (COPD) from mechanical ventilation, which patient assessment indicates that the weaning protocol should be discontinued?

The patient respiratory rate is 32 breaths/min. rationale: Tachypnea is a sign that the patient's work of breathing is too high to allow weaning to proceed. The patient's heart rate is within normal limits, although the nurse should continue to monitor it. An oxygen saturation of 93% is acceptable for a patient with COPD. A spontaneous tidal volume of 500 mL is within the acceptable range.

A patient who has undergone liver transplantation is ready to be discharged home. The nurse is providing discharge teaching. What topic will the nurse emphasize the most related to discharge teaching?

The patient will take immunosuppressive agents as required.

Urine:WBC - Nursing Implications

The prescence of 5 or more WBCs in the urine indicae a UTI involving the bladder or kidneys, OR BOTH. A clean catch culture Vaginal discharge may contaminate the urine specimin and factitously cause WBC in the culture

What is the definition of cerebral perfusion pressure?

The pressure at which brain tissue is perfused which is 80-100 mmHg or at least 70

"35. The nurse provides a referral to Alcoholics Anonymous to a client who describes a 20-year history of alcohol abuse. The primary function of this group is to: "a. Encourage the use of a 12-step program. b. Help members maintain sobriety. c. Provide fellowship among members. d. Teach positive coping mechanisms."

The primary purpose of Alcoholics Anonymous is to help members achieve and maintain sobriety. Although each of the remaining answer choices may be an outcome of attendance at Alcoholics Anonymous, the primary purpose is directed toward sobriety of members.

"Five minutes after receiving a preoperative sedative medication by IV injection, a patient asks the nurse to get up to go to the bathroom to urinate. Which of the following is the most appropriate action for the nurse to take? "A.) Assist patient to bathroom and stay next to door to assist patient back to bed when done. B. Allow patient to go to the bathroom since the onset of the medication will be more than 5 minutes. C. Offer the patient to use the urinal/bedpan after explaining the need to maintain safety. D. Ask patient to hold the urine for a short period of time since a urinary catheter will be placed in the operating room."

The prime issue after administration of either sedative or opioid analgesic medications is safety. Because the medications affect the central nervous system, the patient is at risk for falls and should not be allowed out of bed, even with assistance.

A pt is 1 day post op after having abdominal surgery. She has incisional pain, a 99.5F temp, slight erythema at the incision margins and 30mL of serous sanguineous drainage in the Jackson Pratt drain. Based on this assessment data, what conlusion would the nurse make?

The pt is experiencing normal inflammatory response.

Pco2

The respiratory component of pH. It determines how well the lungs are eliminating carbon dioxide from the body.

Which assessment information obtained by the nurse when caring for a patient receiving mechanical ventilation indicates the need for suctioning?

The respiratory rate is 32 breaths/min. rationale: The increase in respiratory rate indicates that the patient may have decreased airway clearance and requires suctioning. Suctioning is done when patient assessment data indicate that it is needed, not on a scheduled basis. Occasional expiratory wheezes do not indicate poor airway clearance, and suctioning the patient may induce bronchospasm and increase wheezing. An SpO2 of 93% is acceptable and does not suggest that immediate suctioning is needed.

If the baby is breech:

The sounds are HIGH up in the FUNDUS near the UMBILICUS.

If the baby is vertex:

The sounds are SLIGHTLY ABOVE the SYMPHYSIS PUBIS.

If the baby is a POSTERIOR presentation:

The sounds are heard at the SIDES.

If the baby is an anterior presentation:

The sounds are heard closer to the MIDLINE, between the UMBILICUS and the SIDES.

What does the PR interval represent?

The time required for the impulse to travel from the atria through the A-V node

"O"

The universal donor (remember "O" in donor)

"AB"

The universal receipient

A patient newly admitted to your unit has an ileal conduit. You need to determine the appliance size to order for this patient. What does the nurse measure to determine the size of the appliance needed?

The widest part of the stoma

don't assume someone is competent to do something just because of what

Their job description. It's our job to find out our staff's strength and weakness.

Placenta Previa:

There is no pain, bleeding is present.

When caring for a patient who has an arterial catheter in the radial artery to monitor blood pressure, which information obtained by the nurse is most important to report to the health care provider?

There is redness at the catheter insertion site. rationale: Redness at the catheter insertion site indicates possible infection. The Allen test is performed before arterial line insertion, and a positive test indicates normal ulnar artery perfusion. A MAP of 86 is normal and the dicrotic notch is normally present on the arterial waveform.

The nurse is caring for a teenage girl who as had an anaphylactic reaction after a bee sting. The nurse is providing patient teaching prior to the patient's discharge. In the event of an anaphylactic reaction, the nurse informs the patient that she should self-administer epinephrine in what site?

Thigh

Obsession is:

Thought

The nurse is caring for a patient with hyperthyroidism. What would the nurse closely monitor for?

Thyroid storm

4 year old cannot interpret:

Time... need to explain time in relationship to a known COMMON EVENT (ex. Mom will be back after supper).

Paget's Disease:

Tinnitus, bone pain, enlargement of bone, thick bones

Protonix is given prophylactically:

To prevent stress ulcers

Dumping Syndrome (post-operative ulcer/stomach surgeries)

To prevent: Eat in a reclining position Lie down for 20-30 minutes after meals Restrict fluids during meals Low CHO and low fiber diet Encourage small but frequent meals

Apply eye drops:

To the conjunctival sac and then apply pressure to the nasolacrimal duct/inner canthus.

Babinski Sign

Toes curl (good sign); Toes fan (bad sign)

A patient admitted with inflammatory bowel disease asks the nurse for help with menu selections. What menu selection is the best choice for this patient?

Tofu

Decorticate:

Towards the CORD

What is Cranial Nerve V?

Trigeminal; facial sensation and facial movement

Itching under the cast area:

Treat with cool air via a blow dryer, ice pack for 10-15 minutes. NEVER use a QTip or anything to scratch the area.

Diamox

Treatment for Glaucoma and high altitude sickness; don't take if allergic to sufla

Vistaril

Treatment for anxiety and itching (also given preop to dry up the mouth) Assess for dry mouth

Theophylline

Treatment for asthma or COPD; therapeutic level is 10-20

Carafate

Treatment for duodenal ulcers (coats the ulcers); client should take before meals

Dopamine (Intropin)

Treatment for hypotension, shock, low cardiac output, poor perfusion to vital organs; monitor EKG for arrythmias and monitor BP

Ritalin

Treatment of ADHD; assess for heart related side effects and report immediately; child may need a drug holiday b/c it stunts growth

Amphojel

Treatment of GERD and KIDNEY STONES Assess for constipation

Apresoline (hydralazine)

Treatment of HTN or CHF; report flu-like symptoms; rise slowly from a sitting/lying position; take with meals

Bentyl

Treatment of IBS; assess for anticholinergic side effects

Cogentin

Treatment of Parkinson's and EPS effects of other drugs

Artane

Treatment of Parkinson's; may cause a sedative effect

Sinemet

Treatment of Parkinson's; the client's sweat, saliva, and urine may turn reddish brown occassionally; may cause drowsiness

Librium

Treatment of alcohol W/D; don't take alcohol with this medication (causes severe N/V)

Indocin (NSAID)

Treatment of arthritis (osteo, RA, gouty), bursitis, and tendonitis

Timolol (Timoptic)

Treatment of glaucoma

Synthroid

Treatment of hypothyroidism; may take several weeks to take effect; notify doctor of CP; take in the AM on an empty stomach; could cause hyperthyroidism

Oncovin (vincristine)

Treatment of leukemia; given IV only

Tigan

Treatment of post-op N/V and for nausea associated with gastroenteritis

Kwell

Treatment of scabies and lice. Scabies: apply lotion once and leave on for 8-12 hours Lice: use the shampoo and leave on for 4 minutes with hair uncovered then rinse with warm water and comb with a fine tooth comb

Navane

Treatment of schizophrenia; assess for EPS

Dilantin

Treatment of seizures; therapeutic level is 10-20

Strabismus:

Treatment: Botox and patch the GOOD eye so that the weaker eye can get stronger.

Hypomagnesemia

Tremors, tetany, seizures, dysrhythmias, depression, confusion, dysphagia

What is Cranial Nerve IV?

Trochlear; eye movement

True or False The character of the exudate, in amount, color and odor, can help to identify the exact nature of the infection

True

Peritoneal Dialysis (when outflow is Inadequate)

Turn client from SIDE to SIDE--BEFORE checking for kinks in the tubing

Treatment for LATE DECELS

Turn the mother to her LEFT side, to allow more blood flow to the placenta.

For a CABG operation, when the GREAT SAPHENOUS vein is taken it is:

Turned INSIDE OUT due to the valves that are inside

2-3 months:

Turns head from side to side

12-13 months:

Twelve and up, drink from a cup

Which type of diabetic always requires insulin replacement?

Type 1 DM

Which type of diabetic sometimes requires no medication?

Type 2 DM

What criteria should alert a client with known angina who takes nitroglycerin tablets sublinguallty to call EMS?

Unrelieved chest pain after nytroglycerine

A new admit is considered stable or unstable?

Unstable and is our PRIORITY CLIENT!!! BUT DON'T FORGET that a client that's been on our floor can be COMPLEX AND STABLE at the same time.

Position of ear for adult

Up and Back

Fetal Alcohol Syndrome:

Upturned nose, flat nasal bridge, thin upper lip, and SGA

A nurse has just reassessed the condition of a postoperative patient admitted to the surgical unit 1 hour ago. The nurse plans to monitor which of the following parameters most carefully during the next hour? 1. urinary output of 20 ml/hr 2. Temperature of 37.6 Celsius 3. Blood Pressure of 100/70 mmhg 4. Serous drainage of the surgical dressing

Urine output should be maintained at a minimum of 30 ml/hr for an adult. An output of less than 30ml/hr for each consecutive 2 hours should be reported to a physician.

For ACUTE PAIN (Sprained ankle):

Use COLD

For CHRONIC PAIN (rheumatoid arthritis):

Use HEAT therapy

Phobic Disorders:

Use SYSTEMIC DESENSITIZATION

An 80 y/o male, newly diagnosed with primary HTN, has just been put on a beta-blocker. The nurse knows that in addition to teaching the patient about his medication, she should also focus her teaching on what?

Use of supportive devices such as handrails and walkers to prevent falls stemming from postural hypotension

Dexedrine:

Used for ADHD; may alter insulin needs; avoid taking with MAOIs; take in the morning (insomnia is a possible side effect)

Rifampin:

Used for TB; dyes body fluids orange

Diamox:

Used for glaucoma; can cause hypokalemia

Cytovene:

Used for retinitis caused by cytomegalovirus; client will need regular eye exams and should report: dizziness, confusion, or seizures immediately

Pitocin:

Used for uterine stimulation

Tensilon:

Used in Myesthenia Gravis to confirm the diagnosis.

Glucose Tolerance Test:

Used on client's who are pregnant; result of 140 or higher needs further evaluation

International Normalized Ratio - What is it used for?

Used to monitor Anticoagulation therapy

Greenstick Fracture:

Usually seen in children; bone breaks on one side and bends on the other

ICP-increased BP, decreased pulse, decreased resp. shock- decreased BP, increased pulse, increased resp.

VS in ICP and Shock

What is Cranial Nerve X?

Vagus; controls muscle of voice, innervates many organs, can cause lowering of the heart rate, and muscle movement

Medication of choice for status elipticus is:

Valium

CO2 causes:

Vasoconstriction

EleVate Veins; dAngle Arteries

Veins and arteries positions for better perfusion.

HOLD High alarm- Obstruction due to incr. secretions, kink, pt. coughs, gag or bites Low press alarm- Disconnection or leak in ventilatior or in pt. airway cuff, pt. stops spontaneous breathing

Ventilator alarms (HOLD)

A patient who is receiving mechanical ventilation is anxious and is "fighting" the ventilator. Which action should the nurse take first?

Verbally coach the patient to breathe with the ventilator. rationale: The initial response by the nurse should be to try to decrease the patient's anxiety by coaching the patient about how to coordinate respirations with the ventilator. The other actions also may be helpful if the verbal coaching is ineffective in reducing the patient's anxiety.

Meniere's Disease

Vertigo, Tinnitus

Chicken Pox

Vesicular Rash (central to distal) dew drop on rose petal

IM site for Toddlers above 18 months:

Vetrogluteal

The patient has been newly diagnosed with HIV. The patient asks the nurse what is going on inside his body. The nurse explains that the body produces antibody molecules in an effort to contain the free HIV particles and assist in their removal. The nurse goes on to explain that after this first inital immune response the remaining amount of virus in the body is called what?

Viral Set point

Retinal Detachment

Visual floaters, flashes of light, and blurring of vision

Coumadin antedote

Vitamin K

The nurse is caring for a patient with liver failure. The nurse understands that patients in liver failure often require vitamin therapy. Which vitamin does the liver require for the synthesis of prothrombin?

Vitamin K

What is tidal volume?

Vt is the amount of air that moves in and out of the lungs in one normal breath.

Urine: WBC Casts - Nursing Implications

WBC Casts are most frequently found in infections of the kidney, poststreptococcal glomerulonephritis or inflammitory nephriris

A pt is admitted with a chronic leg wound. The nurse assesses local manifestations of erythema and pain at the wound site. What would the nurse anticipate being ordered to assess the pts systemic response?

WBC and differential

Pancreatic enzyme are taken:

WITH meals

Myasthenia Gravis

WORSENS with exercise and IMPROVES with rest.

What is the best way for the nurse to begin the preoperative interview?

Walk in the client's room, sit down, maintain eye contact, and make an introduction.

A community health nurse is providing an educational event at the local high school The topic nurse is speaking about is varicose veins. What would the nurse suggest as a proactive preventative measure for varicose veins.

Walking for several minutes every hour to promote circulation

Where there is sugar (glucose) ...

Water follows.

The nursing instructor is discussing ulcerative colitis with her clinical group. What would she tell her students about the characteristics of the stools of these patients?

Watery with blood and mucus

When do PVC's present grave danger?

When they begin to occur more often than once in 10 beats, occur in twos or threes, land near the T wave, or take on multiple configurations

Prior to administration of IV immunoglobulin what interventions should the nurse implement?

Weigh the patient before administration to determine onset of edema.

What is the best indication of DEHYDRATION?

Weight

Cerebral Angio Prep:

Well hydrated, lie flat, shave the site, pulses marked. S/P: Keep flat for 12-14 hours, check site, pulses, and force fluids.

Asthma

Wheezing on expiration

A nurse is administering two highly protein-bound drugs. Which is the safest course of action for the nurse to take?

When administering two drugs that are protein-bound, one of the drugs will have fewer sites to which to bind and thus more drug available for activity, thereby increasing the risk of toxicity. Food or water will not change the outcome of administration. Hepatic function is a concern at this time.

Wilm's tumor

Wilm's tumor is usually encapsulated above the kidneys causing flank pain.

A comatose patient is receiving oral care. What oral regimen would be most effective in decreasing the client's risk of tooth decay and plaque accumulation?

Wiping the teeth and gums with gauze pad

The nurse is caring for a patient with a history of systemic lupus erythematosus who has been recently diagnosed with ESRD. The patient has an elevated phosphorus level and has been prescribed calcium acetate to bind the phosphorus. What is an important instruction that the nurse should give the patient about how to take the prescribed phosphorus-binding medication?

With each meal

Mevacor (anticholesterol) must be given with:

With evening meal if it is QD (per day)

Hemophilia:

X-linked; mother passes disease to the son

Urine: Biliruben - defination

Yellow pigment in RBCs that once broken down and recycled, is transferred into the liver and put into bile

What specific things do you have to tell the person you are delegating to?

You must make sure exactly which task you've assigned them, which should be done first, etc., and any other tasks you need completed and when. Give CLEAR directions indicating what ranges you want reported to you.

Addison's:

You need to "add" hormones

Iron injections should be given via:

Z-track so that they don't leak into the SQ tissues

A 30 y/o male patient comes into the emergency department with sever asthma-related signs and symptoms. What medication is recommended to prevent the signs and symptoms of asthma?

Zafirlukast (Accolate)

What is the formula for MAP?

[SBP + 2(DBP)] / 3 = MAP

when we get a nurse from another unit to our floor, what do we consider them

a brand new nurse. Cannot handle any specialized care.

reactive hyperemia

a bright red flush on the skin occurring after pressure is relieved

Undescended testis or cryptorchidism is

a known risk factor for testicular cancer later in life. Start teaching boys testicular self exam around 12, because most cases occur during adolescence.

What precautions are required for patients with TB when placed on respiratory isolation?

a mask for everyone entering the room, a private room, and patient must wear mask when leaving their room

Prothrombin Time - definition

a measure of the blood's coagulation abilities by measuring how long it takes for a clot to form after prothrombin has been activated

International Normalized Ratio - definition

a method developed to standardize prothrombin time results among laboratories by accounting for the different thromboplastin reagents used to determine PT

REM sleep

a recurring sleep state during which dreaming occurs

Cholesterol - definition

a steroid, which is both made in the liver and ingested in food from animal sources

Chapter 24 . A nurse assesses an oral temperature for a pt as 38.5 (101.3F). What term would the nurse use to report this temperature? a. Fever b. Hypthermia c. Hypertension d. Afebrile

a. Fever

14. It is important to have the appropriate cuff size when taking the bp. What error my result from a cuff that is too large or to small? a. An incorrect reading b. Injury to the pt c. Prolonged pressure on the arm d. Loss of Korotkoff sound

a. A bp cuff that is not the right size may cause an incorrect reading.

12. A pt has a bp reading of 130/90mm Hg when visiting a clinic. What would the nurse recommend to the pt? a. Follow-up measurements of blood pressure b. Immediate treatment by a physician c. nothing, because the nurse considers this reading is due to anxiety d. A change in dietary intake.

a. A single bp reading that is mildly elevated is not significant, but the measurement should be taken again over time to determine if hypertension is problem. The nurse would recommend a return visit to the clinic for are check.

3. Which of the following terms is defined as the sense of identification with a collective cultural group? a. Ethnicity b. Race c. Cultural acquisition d. Culture shock

a. Ethnicity is the sense of identification with a collective cultural group, largely based on the group's common heritage.

7. What group is the largest subculture of the healthcare system? a. Nurses b. Physicians c. Social workers d. Physical therapists

a. Nurses are the largest subculture of the healthcare system.

5. After reviewing information about the four concepts common to nursing theories, the students demonstrate understanding of the information when they identify which concept as most important? A. Person b. Environment c. Health d. Nursing

a. Of the four concepts, the most important is the person

4. Which type of theory focuses on clinical nursing practice? A. Prescriptive theory B. Descriptive theory C. Developmental theory D. Systems theory

a. Prescriptive theories address nursing interventions and are designed to control, promote, and change clinical nursing practice.

10. Who are the largest group of healthcare provides in the US? a. Registered nurses b. Physicians c. Physical therapists d. Social workers

a. Registered nurses are the largest group of healthcare provider in the US.

angina (low oxygen to heart tissues) = no dead heart tissues. MI= dead heart tissue present.

angina vs MI

7. Which nursing organization was the first international organization of professional women? a. ICN b. ANA c. NLN d. NSNA

a. The ICN, founded in 1899, was the first international organization of professional women.

7. Two nurses are taking an apical-radial pulse and not a different in pulse rate of 8 beats per min. The nurse would document this difference as which of the following? a. Pulse deficit b. Pulse amplitude c. ventricular rhythm d. heart arrhythmia

a. The difference between the apical and radial pulse rate is called the pulse deficit.

5. A pt complains of severe abdominal pain. When assessing the vital signs, the nurse would nt be surprised to find what assessment? a. An increase in pulse rate b. A decrease in body temperature c. A decrease in bp c. A increase in respiratory depth.

a. The pulse often increases when an individual is experiencing pain. Pain does not affect body temperature and may increase (not decrease) blood pressure. Acute pain may increase respiratory rate by decrease respiratory depth

fistula

abnormal passageway between two organs or between an internal organ and the body surface

Graves disease/hyperthyroidism

accelerated physical and mental function; sensitivity to heat, fine/soft hair

Platelet Count - Which drugs may cause a drcrease in lab values?

acetaminophen, asprin, chemotherepy, H2 -blockers, INH levaquin, streptomycin, sulfonmides, thiazide diuretics

A breast cancer patient treated with Tamoxifen should report changes in:

acuity, because the adverse effect could be irreversible.

Differentiate between acute renal failure and chronic renal failure:

acute is often reversable, caused by abrupt deterioration of kidney function, chronic is irreversable, slow deterioration of kidney function characterized by increasing BUN and creatinine, eventually dialysis is required

what are the nursing responsibilities during alcohol withdrawal?

administering sedation as needed, monitoring VS, seizure precautions, providing quiet, well-let environment

High Density Lipoprotien - Lab Values: Child

age: 1-9 53-56 10-14 52-55 15-19 46-52

Sodium - defination

an electrolyte regulated in the blood and urine by the kidneys

What does the Monro-Kellie Hypothesis state?

an increase of 1 or more intra-cranial contents must cause a reciprocal change in the remaining contents or increased ICP will occur

crushing injury

an injury caused by compression that involves both direct tissue injury caused by circulation disturbance resulting from pressure on blood vessels

open wound

an injury in which the skin is interrupted, exposing the tissue beneath

contusion

an injury to underlying tissues without breaking the skin and is characterized by discoloration and pain

A nurse is developing a plan of care for a client scheduled for surgery The nurse should include which activity in the nursing care plan for the client on the day of surgery? 1. Have the client void immediately before going into surgery 2. Avoid oral hygiene and rinsign with mouthwash 3. Verify that the client has not eaten for the last 24 hours 4. Report immediately any slight increase in blood pressure or pulse.

answer #1The nurse would assist the client to void immediately before surgery so that the bladder will be empty. A slight increase in blood pressure and pulse is common during the preoperative period and is usually the result of anxiety. The client usually has a restriction of food and fluids for 6 to 8 hours before surgery instead of 24 hours. Oral hygeine is allowed, but the client should not swallow any water.

Shapes of Fontanels?

anterior is diamond shaped, posterior is triangle shaped

anterior fontanelle-18 months. Posterior- 6 to 8 weeks.

anterior vs posterior fontanelle closed by

Bactrim

antibiotic..dont take if allergic to sulfa drugs...diarrhea common side effect...drink plenty of fluids

heparin= protamine sulfate coumadin= vitamin k morphine= narcan ammonia= lactulose acetaminophen= n-Acetylcysteine. Iron= deferoxamine Digitoxin, digoxin= digibind. Alcohol withdraw= Librium.

antidotes

what are the sx of cocaine withdrawal?

anxiety, depression, irritability, fatigue, insomnia/ hypersomnia, psychomotor agitation

Detached Retina

area of detachment should be in the dependent position

Guillain-Barre Syndrome (GBS)

ascending paralysis. Keep eye on respiratory system.

What is often the underlying cause of an abdominal aortic aneurysm?

atherosclerosis (hardening of the arteries) Occurs in the part of the aorta that feeds the abdomen and lower limbs

When preparing to assist with the insertion of a pulmonary artery catheter, the nurse will plan to ______________

attach cardiac monitoring leads before the procedure. rationale: Dysrhythmias can occur as the catheter is floated through the right atrium and ventricle, and it is important for the nurse to monitor for these during insertion. Pulmonary artery catheter insertion does not require anesthesia, and the patient will not need to be NPO. Changes in cardiac enzymes or heart sounds are not expected during pulmonary artery catheter insertion.

The nurse evaluates her teaching as effective when the pt recovering from acute renal failure states that he will do which of the follwing?

avoid takng durgs that may be nephrotoxic

Identify 3 nursing interventions that prevent postoperative urinary tract infections:

avoiding postoperative catheterization, increase oral fluid intake, Q4-Q6hours and early ambulation

5. A young Hispanic mother comes to the local clinic because her baby is sick. She speaks only Spanish and the nurse speaks only English. What should the nurse do? a. Use short words and talk more loudly b. Ask an interpreter for help c. Explain why care can't be provided d. Provide instructions in writing.

b .Many agencies have a qualified interpreter who understands the healthcare system and can reliably provide assistance.

7. Which type of quantitative research is conducted to directly influence or improve clinical practice? a. Basic research b. Applied research c. Experimental research d. Descriptive research

b. Applied research, a type of quantitative research, is designed to directly influence or improve clinical practice

10 A student nurse is learning to assess bp What does the bp measure? a. Flow of blood through he circulation b. Force of blood against arterial walls c. force of blood against venous wall d. flow of blood through the heart

b. Blood pressure is the measurement of the force of blood against arterial walls.

Chapter 2 1. Which of the following phrases best defines culture? a. A dominant group within a society b. A shared system of beliefs, values, and behaviors c. One's values are replaced by the values of the dominant culture d. Categories are based on specific physical characteristics

b. Culture may be defined as a shared system of beliefs, values, and behavioral expectations that provide social structure for daily living.

9. Of the following types of qualitative research, which method developed in anthropology?

b. Ethnographic research was developed by the discipline of anthropology and is used to examine issues of culture of interest to nursing

What are the side effects of amphotericin B?

can be quite severe, including anorexia, chills, cramping muscle and joint pain, circulatory problems

3. When assessing a temperature rectally, the nurse would use extreme care when inserting the thermometer to prevent which of the following? a. An increase in heart rate b. A. decrease in heart rate c. A decrease in blood pressure d. An increase in respirations

b. Insertion of a rectal thermometer may stimulate the vagus nerve, which, in turn, would decrease heart rate. This may potentially be harmful for pts with cardia problems

8. What is the purpose of the ANA's Scope and Standards of Practice? a. To describe the ethical responsibility of nurses b. To define the activities that are special and unique to nursing c. To establish nursing as an independent and free-standing profession.

b. The ANA's scope and standards of practice define the activities of nurses that are specific and unique to nursing.

15. A pt has IV fluids infusing in the R arm. When taking a bp on this pt what would the nurse do in this situation? a. Take the bp in the R arm b. Take the bp int he left arm c. Use the smallest possible cuff d. report inability to take th bp.

b. The bp should be taken in the arm opposite the one with the infusion.

Chapter 1. 1.Which phrase best describes the science of nursing? a. The skilled application of knowledge b. The knowledge base for care c. Hands-on care, such as giving a bath d. Respect for each individual pt.

b. The science of nursing is the knowledge base for care that is provided. In contrast, the skilled application of that knowledge is the art of nursing.

9. A nurse wants to acquire knowledge of a specific culture. What could be done first? a. Talk to coworkers b. Review literature c. Talk to family members of the pt d. Ask others with more experience for help.

b. reviewing literature about a specific culture can provide the nurse with a starting point for information about cultural values, dietary practices, family lines of authority, and health and illness beliefs and practices.

8. Before assessing respirations, the nurse reviews normal rates for adults. Which rate would the nurse identify as normal a. 1 to 6 breaths/min b. 12 to 20 breaths/min 3. 60 to 80 breaths/min d. 100 to 120 breaths/ min

b.The normal respiratory rate for adults is 12 to 20 breaths/min

Potassium

bananas, potatoes, citrus fruits, tomato, watermelon

COPD

baroreceptors that detect the CO2 level are destroyed. Therefore, O2 level must be low because high O2 concentration 8blows the patient's stimulus for breathing.

When examining a client with emphysema, what physical findings is the nurse likely to see?

barrel chest, dry or productive cough, decreased breath sounds, dyspnea, crackles in lung fields

Shock

bedrest with extremities elevated 20 degrees, knees straight, head slightly elevated (modified Trendelenburg)

AM- Steroids, Diuretics, Aricept PM- Growth Hormone before meals- Carafate (Sulcrafate) After meals- antacids

best time to take meds

What should the O2 flow rate be for the client with COPD?

between 1-2 liters per NC, too much O2 may eliminate the patients stimulus to breath since COPD patient's have a hypoxic drive to breathe

HCO3 - defination

bicarbonate, hydrogen carbonate; Bicarbonate, 70% of carbon dioxide in the blood is converted to a bicarbonate; The metabolic component of pH. It determines how well the kidneys are working to maintain adequate levels in the body system.

How does a Stage 2 pressure ulcer appear?

blistering, partial thickness skin loss involving the epidermis and dermis, shallow crater

With glomerulonephritis you should consider

blood pressure to be your most important assessment parameter. Dietary restrictions you can expect include fluids, protein, sodium, and potassium.

hot and dry-sugar high (hyperglycemia) cold and clammy-need some candy (hypoglycemia)

blood sugars

Creatine Phosphokinase - defination

blood test to measure an enzyme of heart and skeletal muscle released into the blood following muscle injury or necrosis

Hypervolemia

bounding pulse, SOB, dyspnea, rares/crackles, peripheral edema, HTN, urine specific gravity <1.010; Semi-Fowler's

The main hypersensitivity reaction seen with antiplatelet drugs is

bronchospasm (anaphylaxis)

Anterior fontanelle closes:

by 18 months

Posterior fontanelle closes:

by 6 to 8 weeks

"A preoperative client expresses anxiety to a nurse about upcoming surgery. Which response by the nurse is most likely to stimulate further discussion between the client and the nurse? "a) ""If its any help, everyone is nervous before surgery"" b) ""I will be happy to explain the entire surgical procedure to you"" c) ""Can you share with me what you've been told about your surgery?"" d) ""Let me tell you about the care you'll receive after surgery and the amount of pain you can anticipate"""

c) ""Can you share with me what you've been told about your surgery?""Explanations should begin with the info the client knows. By providing the client with individualized explanations of care and procedures, the nurse can assit the client in handling anxiety and fear for a smooth preoperative experience. Clients who are clam and emotionally prepared for surgery withstand anesthesia better and experience fewer postop complications. Options a, b, and d will produce anxiety in the patient.

Aspirin

can cause Reye's syndrome (encephalopathy) when given to children

3. When describing the term concept to a group of students, which word would the instructor most likely used? a. Fact b. Science c. Idea d. Truth

c. A concept,, like an idea, is an abstract impression of reality.

6. A nurse is interviewing a newly admitted pat. Which question would be considered culturally sensitive? a. "Do you think you will be able to eat the food we have here?" b. " Do you understand that we can't prepare special means?" c. "What types of food do you eat for meals?" d. "Why can't you just eat our food while you are here?"

c. Asking pts what types of foods they eat for meals is culturally sensitive.

Hodgkin's disease

cancer of lymph is very curable in early stage.

8. A nurse states, "I know I am cleaner than most of my pts." What does this statement indicate? a. Cultural assimilation b. Racism c. Ethnocentrism d. Stereotyping

c. Ethnocentrism occurs when one believes that one's own ideas and practices are superior to those of others.

10. Although all for the following are important to culturally competent nursing care, which one is most basic? a. Learning another language. b. Having significant information c. Treating each person as an individual d. Recognizing the importance of family.

c. In all aspects of nursing, it is important to treat each pt as an individual. This is also true in providing culturally competent care.

9. What type of authority regulates the practice of nursing? A. International standards and codes b. Federal guidelines and regulations c. State nurse practice acts d. Institutional policies

c. Nurse practice acts are established in ea. state to regulate the practice of nursing.

6. The nurse is taking an apical pulse. What equipment will he take into the pts room? a. Sphygmomanometer b. Electronic thermometer c. Stethoscope d. Doppler apparatus

c. The apical pulse can only be assessed by listening with a stethoscope.

8. Anurse researcher is studying the effects of exercise and sleep on blood pressure. The researcher identifies blood pressure as which type of variable? a. Exploratory b. Correlational c. Dependent d. Independent

c. The dependent variable is the variable being studied and is determined by manipulating conditions (independent variable)

13. A nurse is documenting a bp of 12/80 mm Hg. The nurse interprets the 120 to represent which of the following? a. Pulse rate b. Diastolic pressure c. Systolic pressure d. Pulse deficit

c. The systolic pressure is 120mm HG. The diastolic is the pressure 80mmHg.

2. Minority groups living within a dominant culture may lose the cultural characteristics that made them different. What is the process called? a. Cultural diversity. b. Cultural imposition c. Cultural assimilation d. Ethnocentrism

c. When minority groups live within a dominant group many members lose the cultural characteristics that once made them different.

2. A nurse is assessing vital signs on several hospitalized children. The nurse would plan to use the oral route to assess temperature for which pt? a. 6 month - old infant b. Patient receiving oxygen therapy by mask c. 15 yr old healthy adolescent d. Unconscious pt

c. a healthy adolescent would be an appropriate pt for assessing temperature by the oral route

Why is a client in renal failure being given antacids?

calcium and aluminum antacids bind phosphates and help to keep phosphates from being absorbed into the blood stream

Calan (verapamil)

calcium channel blocker: tx of HTN, angina...assess for constipation

An intraaortic balloon pump (IABP) is being used for a patient who is in cardiogenic shock. An assessment finding indicating to the nurse that the goals of treatment with the IABP are being met is a _________________

cardiac output (CO) of 5 L/min. rationale: A CO of 5 L/min is normal and indicates that the IABP has been successful in treating the shock. The low SV signifies continued cardiogenic shock. The tachycardia and low urine output also suggest continued cardiogenic shock.

hypoxemia (low oxygen level in tissues).

cardinal sign of ARDS

During Continuous Bladder Irrigation (CBI) -

catheter is taped to thigh so leg should be kept straight. No other positioning restrictions.

Amynoglycosides (like vancomycin) c

cause nephrotoxicity and ototoxicity.

Isoniazid

causes peripheral neuritis

What lifestyle changes can the patient who is at risk for hypertension initiate to reduce the likelihood of becoming hypertensive?

cease cigarette smoking, if applicable, control weight, exercise regularly and maintain a low fat, low cholesterol diet

cerebral palsy

cerebral palsy = poor muscle control due to birth injuries and/or decrease oxygen to brain tissues.

SIADH (increased ADH)

change in LOC, decreased deep tendon reflexes, tachycardia, n/v/a, HA; administer Declomycin, diuretics

what are the indications of a head injury?

changes in VS, confusion, disturbances of consciousness, sudden onset of neurologic deficits, pupillary abnormalities, sensory dysfunctions, visual and/or hearing impairment, vertigo, headache, spasticity, movement disorders, seizures, hypovolemic shock

Evaluation of pain therapy requires the consideration of the ________ character of pain, ____ to therapy, ___ to function, and patient's perception of a therapy's effectivness

changing response ability

what is the nursing care for a cholecystectomy?

check T tube drainage if present (500-1,000mL/day), protect skin around incision from bile, zinc oxide or water-soluble lubricant, keep drainage bag at same level as gallbladder (on bed), maintain patient in semi-Fowler's position after T-tube is removed, observe dressing for bile, notify physician if significant drainage is observed

what is the nursing care for a woman's first post-partum hour?

check VS, fundal height, and lochia (color, amount) q15m

Digoxin

check pulse, less than 60 hold, check dig levels and potassium levels.

what is the nursing care for a dialysis patient?

check the bruit or thrill q8h, don't take BP or blood specimens through extremities

4 options of cancer therapy

chemo, radiation, surgery, allow to die with dignity.

what is the nursing care for a head injury?

close observation, possible surgery, maintenance of head and neck alignment with body axis, preservation of brain homeostasis and prevention of secondary brain damage, lower ICP, prevent seizures with anticonvulsants, maintain fluid/ electrolyte balance, may intubate if comatose and mechanically ventilate, maintain NG tube, maintain airway, prevent injury, improve cognitive functioning, client and family education

Cataract

cloudy, blurry vision. Treated by lens removal-surgery

What is cluster breathing?

clusters of several breathes with irregular periods of apnea between clusters

use Cold for acute pain (eg. Sprain ankle) and Heat for chronic ( rheumatoid arthritis)

cold vs hot compress

secondary intention

complex healing of a larger wound involving sealing of the wound through scab formation, granulation or filling of the wound, and constriction of the wound. (full thickness)

What symptoms of pneumonia might the nurse expect to see in an older client?

confusion, lethargy, anorexia and rapid RR

what does meal planning for a diabetic include?

consistency in total calories, carbs, and timing of food. diet should be high in complex carbs, low intake of fat, high intake of fiber, and some protein

What items should the nurse assist the patient in removing before surgery?

contact lenses, glasses, dentures, partial plates, wigs, jewelry, prostheses, makeup and nail polish.

5 important teaching aspects for clients who are beginning corticosteroid therapy

continue medication until weaning plan has begun by physician, monitor serum potassium, glucose and sodium frequently, weigh daily, and report a weight gain of >5lb in a week, monitor BP and pulse closely, and teach signs of Cushing's syndrome.

At an initial visit to a pt in the home, the nurse explains that the pt and family can expect the nurse to :

coordinate services provided by all members of the home care team needed by the pt.

Case Manager

coordinates pt care during an entire episode of illness in all care settings.

"A pre-operative nurse prepares a client for surgery, which nursing interventions should be included in the plan of care? Mark all that apply. "a. Maintain NPO status to prevent aspiration. b. Verify the client's signature on the consent prior to surgery. c. Remove dentures and contact lenses prior to surgery. d. Check the client's allergy and blood bands for accuracy. e. Verify the client's mobility in all extremities prior to surgery.

correct answers: a,b,c,d"Maintaining NPO prevents the client from aspirating food particles into the lungs during and after surgery. Because of legal requirements, the surgical consent must be signed prior to surgery to verify the client's acknowledgement of the content on the consent. Dentures, hairpins, glasses, and contacts may interfere with client safety, or compromise the sterile field. Allergy, blood, and identification bands should all be checked prior to surgery to prevent medication errors, blood bank errors, and to facilitate proper identification of the client. Verifying the client's mobility in all extremities prior to surgery is part of the physical assessment, but not a necessary action prior to surgery.

4 components of teaching for a patient with TB:

cough into tissues and dispose into special bags, long term need for daily medication, good hand-washing technique, report symptoms of deterioration or blood in secretions

Prolapsed cord

cover it with sterile saline gauze to prevent drying of the cord and to minimize infection.

What are Cheyne-Stokes respirations?

crescendo-decrescendo pattern; increasing then decreasing rate and depth of respirations followed by apnea

Prednisone toxicity

cushing's syndrome= buffalo hump, moon face, high glucose, hypertension.

A nurse is preparing a plan of care for a client who is a Jehovah's Witness. The client has been told that the surgery is necessary. The nurse considers the client's religious preferences in developing the plan of care and documents that: a) faith healing is practiced primarily b) medication administration is not allowed c) surgery is prohibited in this religious group d) the administration of blood and blood products is forbidden"

d) the administration of blood and blood products is forbidden Among Jehovah's Witnesses, surgery is not prohibited, but the administration of blood and blood products is forbidden. Faith healing is forbidden in this religious group. Administration of medication is an acceptable practice, except if the medication is derived from blood products.

6. Which of the following nursing degrees prepares a nurse for advanced practice as a clinical specialist or nurse practitioner? a. LPN b. ADN c. BSN d. Master's

d. A master's degree prepares advanced practice nurses.

4. While taking an adult pt's pulse, a nurse finds the rate to be 140 beats/min. What should the nurse do next. a. Check the pulse again in 2 hr b. Check the bp c. Record the information d. Report the rate

d. A rate of 140/beats/min in an adult is an abnormal pulse and should be reported to the instructor or the nurse in charge of the pt.

2. Which nurse in history is credited with establishing nursing education? a. Clara Barton b. Lillian Wald c. Lavinia Dock d. Florence Nightingale

d. Florence Nightingale established nursing education

"While assessing a patient preoperatively, the nurse notices a history of latex allergy. Which of the following would be an appropriate intervention? "a. Notify the surgeon so that the case can be canceled b. Give the patient antihistamines to prevent anaphylactic shock c. Question the patient about how his or her latex allergy developed d. Notify the operating room staff so that latex-free supplies will be used

d. Notify the operating room staff so that latex-free supplies will be used Rationale: Latex precaution protocols should be used for patients identified as having a positive latex allergy test result or a history of signs and symptoms related to latex exposure. Many health care facilities have created latex-free product carts that can be used for patients with latex allergies.

4. Which of the following phrases describes one of the purposes of the ANA's Nursing's Social policy statement? a. To describe the nurse as a dependent caregiver b. To provide standards for nursing educational programs c. To regulate nursing research d. To describe nursing's values and social responsibility.

d. The Nursing's Social Policy Statement describes the values and social responsibility of nursing.

10. A nurse is formulating a clinical question in PICO format. Which of the following is represented by the letter P? a. Comparison to another similar treatment b. Clearly defined, focused literature review c. Specific identification of the desired outcome d. Explicit descriptions of population of interest

d. The P in the PICO format represents an explicit description of patient population of interest.

11. A nurse knows that the blood pressure is often higher in older adults based on the understanding that which of the following occurs with aging? a. Loss of muscle mass b. Changes in exercise level c. Decreased peripheral resistance d. Decreased elasticity in arterial walls.

d. With aging, elasticity in the arterial walls is decreased, contributing to an elevated blood pressure reading.

4. A nurse states, "That woman is 78 years old- too old to learn how to change a dressing." What is the nurse demonstrating? a. Cultural imposition b. Clustering c. Cultural competency d. Stereotyping

d. stereotyping is assuming that all members of a group are alike

A 77-year-old client with a history of COPD has undergone a hernia repair. Which of the following expected outcomes should be the priority focus for the nurse? " a.The client ambulates 10 feet with assistance. b.The client tolerates a clear liquid diet. c.The client rates his pain as 2 to 3 on a 10-point scale. d.The client has normal auscultated breath sounds"

d: client has normal breath sounds, is the the biggest priority after major surgery

How does a Stage 4 pressure ulcer appear?

damage to muscle, bone, tendon, or joint, full thickness skin loss with extensive destruction, tissue necrosis

German measles (rubella)

dangerous in pregnancy

Myasthenia Gravis

decrease in receptor sites for acetylcholine. Since smallest concentration of ACTH receptors are in cranial nerves, expect fatigue and weakness in eye, mastication, pharyngeal muscles.

Four hours after mechanical ventilation is initiated for a patient with chronic obstructive pulmonary disease (COPD), the patient's arterial blood gas (ABG) results include a pH of 7.50, PaO2 of 80 mm Hg, PaCO2 of 29 mm Hg, and HCO3- of 23 mEq/L (23 mmol/L). The nurse will anticipate the need to ____________

decrease the respiratory rate. rationale: The patient's PaCO2 and pH indicate respiratory alkalosis caused by too high a respiratory rate. The PaO2 is appropriate for a patient with COPD, increasing the tidal volume would further lower the PaCO2, and the PaCO2 and pH indicate a need to make the ventilator changes.

What is the first sign of tolerance to pain analgesics

decreased duration of drug effectiveness

How does a Stage 3 pressure ulcer appear?

deep crater with or without undermining of adjacent tissue, full thickness skin loss involving damage or necrosis of subcutaneous tissue that may extend down to, but not through underlying fascia

Hypermagnesemia

depresses the CNS, hypotension, facial flushing, muscle ewakness, absent deep tendon reflexes, shallow respirations, emergency

From the a** (diarrhea)= metabolic acidosis From the mouth (vomitus)=metabolic alkalosis

diarrhea vs vomitus

To Prevent Dumping syndrome

eat in reclining position, lie down after meals for 20-30 minutes (also restrict fluids during meals, low CHO and fiber diet, small frequent meals)

While assessing a patient with a central venous catheter, the nurse notes the catheter insertion site is red and tender and the patient's temperature is 101.8° F. The nurse will plan to _____________

discontinue the catheter and culture the tip. rationale: The information indicates that the patient has a local and systemic infection caused by the catheter and the catheter should be discontinued. Changing the flush system, administration of analgesics, and continued monitoring will not help prevent or treat the infection. Administration of antibiotics is appropriate, but the line should still be discontinued to avoid further complications such as endocarditis.

What instructions should be given to a client following radiation therapy?

do NOT wash off lines, wear soft cotton garments, avoid use of powders and creams on radiation site

Identify 2 nursing interventions for the client of hemodyalysis

do not take BP or preform venipunctures on the arm with the fistula or graft, and assess acess site for thriss and bruit

When does Lantus peak?

does not peak, starts working in 70 min

what is the nursing care for an amputation?

don't elevate the residual limb immediately post-op, lie patient prone 20-30 min every 3-4h, dressings changed daily until sutures are removed, limb wrapped in elastic bandage to shape and observe for adequate tissue perfusion

After total hip replacement

don't sleep on operated side, don't flex hip more than 45- 60 degrees, don't elevate HOB more than 45 degrees. Maintain hip abduction by separating thighs with pillows.

Birth weight

doubles by 6 month and triple by 1 year of age.

Nicole Richie RN Air into NPH, then air into regular, draw up regular then draw up NPH

drawing up insulin tip

Orthostasis is verified by:

drop in pressure with increasing heart rate.

Atropine

drug of choice for Asystole (no heart beat)

ACE inhibitor

drug of choice for CHF

adenosine or adenocard

drug of choice for SVT

Lidocaine

drug of choice for Ventricular Tachycardia

Epinephrine

drug of choice for anaphylactic shock

lithium.

drug of choice for bipolar

Valium

drug of choice for status epileptcius

What symptoms should the nurse expect to find in a client with hypokalemia?

dry mouth, and thirst, drowsiness, and lethargy, muscle weakness and aches, and tachycardia

List symptoms of digitalis toxicity:

dysrhythmias, headache, nausea and vomiting

In which order will the nurse take these actions when assisting with oral intubation of a patient who is having respiratory distress?

e) Oxygenate the patient with a bag-valve-mask system for several minutes. b) Place the patient in the supine position. c) Inflate the cuff of the d) Attach an end-tidal CO2 detector to the endotracheal a) Obtain a portable chest-x-ray.tube.endotracheal tube. rationale: The patient is pre-oxygenated with a bag-valve-mask system for 3 to 5 minutes before intubation and then placed in a supine position. Following the intubation, the cuff on the endotracheal tube is inflated to occlude and protect the airway. Tube placement is assessed first with an end-tidal CO2 sensor, then with a chest x-ray.

Differentiate between essential and secondary hypertension:

essential hypertension has no known cause, secondary hypertension develops in response to an identifiable mechanism

Diabetes Insipidus (decreased ADH)

excessive urine output and thirst, dehydration, weakness, administer Pitressin

what are the possible complications of a total hip arthroplasty?

excessive wound drainage, thromboembolism, infection, dislocation of prosthesis

What is the proper teaching plan for a patient taking anti-hypertensive medications:

explain how and when the patient should take the medication, reason for the medication, necessity of compliance, need for follow-up visits while on medications, need for certain lab tests, and vital sign parameters while initiating therapy

A contaminated or traumatic wound may show signs of infection within 24 hours. A surgical wound infection usually develops postoperatively within 14 days. True False

false

Hyperparathyroid

fatigue, muscle weakness, renal calculi, back and joint pain (increased calcium), low Ca, high phosphorus diet

Pneumonia

fever and chills are usually present. For the elderly confusion is often present.

Urine: Casts - definition

fibrous or protein materials, such as pus and fats, that are thrown off into the urine in kidney disease

the client just had surgery to create an ileostomy. the nurse assesses the client in the immediate postoperative period for which most frequent complication of this type of surgery? "1. folate deficiency 2. malabsorption of fat 3. intestinal obstruction 4. fluid and electrolyte imbalance

fluid and electrolyte imbalancea frequent compliction that occurs following ileostomy is fluid and electrolyte imbalance. the client requires constant monitoring of intake and output to prevent this from occurring. losses require replacement by IV infusion until the client can tolerate a diet orally. intestinal obstruction is a less common complication. fat malabsorption and folate deficiency are complications that could occur later in the postoperative period.

4 essential elements of a teaching plan for patients with frequent UTI's:

fluid intake of 3L per day, good handwashing, void every 2-3 hours, take all prescribed medications, cotton underwear

Below Knee Amputation

foot of bed elevated for first 24 hours, position prone daily to provide for hip extension.

Urine: Casts - Nursing Implications

for casts to form pH must be acidic and the urine concentrates. Two types of cases: Hyalinecasts are comglomerations of protien, and cellular casts are conglomerations of degenerated cells.

Unstageable

full thickness tissue loss in which the base of the ulcer is covered by slough and/or eschar

What is an unstageable pressure ulcer?

full thickness tissue loss with slough or eschar at the base of the wound

Bad fetal HR

give O2, often by mask

Carafate

give before meals to coat stomach

NSAIDS, Corticosteroids, drugs for Bipolar, Cephalosporins, and Sulfanomides

give meds to be given with food

Versed

given for conscious sedation...watch for resp depression and hypotension

EYES, VERBAL,MOTOR! It is similar to measuring dating skills...max 15 points -one can do it if below 8 you are in Coma.

glasgow coma scale tip

what is supine hypotension syndrome?

gravid uterus compresses inferior vena cava and aorta when mom lays on her back indications (dizziness, ringing ears, pallor, cold/ clammy extremities. intervention: put mom on her left side.

Supervision

guidance and direction, oversight and eval by the RN to see that delegated task is accomplished.

hallucinations- redirect them delusions- distract them.

hallucinations vs delusions interventions

What is the rule of nines?

head is 9%, anterior trunk is 18%, posterior trunk is 18%, each leg is 18%, each arm is 9%, groin is 1%; must be older then 10 for this to apply

what are the possible adverse effects of nitroglycerine?

headache, flushing, HOTN, tachycardia

List 6 noninvasive, non pharmacological pain relief measures:

heat/cold application, TENS, massage, distraction, relaxation, biofeedback

Potassium - defination

helps build protein; maintains fluids; helps nerves communicate; helps muscles contract

irritable poor sucking

heroin withdrawal on neonate

CSF in meningitis will have:

high protein, and low glucose.

pain modulation

hindering the transmission of pain by release of inhibitory neurotransmitters (endorphins&enkephalins) that produce an analgesic effect

Basophils release:

histamine during an allergic response.

5 symptoms of hypoglycemia:

hunger, lethargy, confusion, tremors or shakes and sweating

What does the Morse scale assess for?

hx of falls, secondary diagnosis, ambulatory aid, IV present, gait/transferring, and mental status; the higher the score the greater the risk of falling

3 symptoms of hyperthyroidism and three symptoms of hypothyroidism

hyper: weight loss, heat intolerance, diarrhea hypo: fatigue, cold intolerance, weight gain

Cushing's disease

hyperNa, hypoK, hyperglycemia, prone to infection, muscle wasting, weakness, edema, HTN, hirsutism, moonface/buffalo hump

Pheochromocytoma

hypersecretion of epi/norepi, persistent HTN, increased HR, hyperglycemia, diaphoresis, tremor, pounding HA; avoid stress, frequent bating and rest breaks, avoid cold and stimulating foods, surgery to remove tumor

HYPERthyroidism think of MICHAEL JACKSON in THRILLER! SKINNY, NERVOUS, BULDGING EYES, Up all night, heart beating fast

hyperthyroidism tip

How does the nurse prevent hypoxia during suctioning?

hyperventilate patient before and after each endotracheal suctioning

Addison's disease

hypoNa, hyperK, hypoglycemia, dark pigmentation, decreased resistance to stress, fractures, alopecia, weight loss, GI distress

An elderly patient who has stabilized after being in the intensive care unit (ICU) for a week is preparing for transfer to the step down unit when the nurse notices that the patient has new onset confusion. The nurse will plan to ________________

inform the receiving nurse and then transfer the patient. rationale: The patient's history and symptoms most likely indicate delirium associated with the sleep deprivation and sensory overload in the ICU environment, and informing the receiving nurse and transferring the patient is appropriate. Postponing the transfer is likely to prolong the delirium. Benzodiazepines and restraints contribute to delirium and agitation.

When using a bronchodilator inhaler inconjuction with a glucocorticoid inhaler, administer the bronchodilator first

inhaler rule

Why is the patient with liver disease at increased risk for operative complications?

impairs the ability to detoxify medications used during surgery, impairs the ability to produce prothrombin to reduce hemorrhage

List 4 nursing interventions that prevent postoperative thrombophlebitis:

in bed leg exercises, early ambulation, antiembolic stockings, avoiding positions and pressures that obstruct venous flow

Hypovolemia

incrased temp, rapid/weak pulse, increase respiration, hypotension, anxiety, urine specific gravity >1.030

Dumping syndrome

increase fat and protein, small frequent meals, lie down after meal to decrease peristalsis, wait 1 hr after meals to drink.

A patient with a subarachnoid hemorrhage is intubated and placed on a mechanical ventilator. When monitoring the patient, the nurse will need to notify the health care provider if the patient develops _______________

increased jugular vein distention (JVD). rationale: Increases in JVD in a patient with a subarachnoid hemorrhage may indicate an increase in intra-cranial pressure (ICP) and that the PEEP setting is too high for this patient. A respiratory rate of 18, O2 saturation of 94%, and green nasogastric tube drainage are normal.

Thyroid storm

increased temp, pulse and HTN

Hypernatremia

increased temp, weakness, disorientation/delusions, hypotension, tachycardia; hypotonic solution

Theophylline

increases the risk of digoxin toxicity and decreases the effects of lithium and Dilantin

What does the wedge pressure measure?

indirectly measures left ventricle preload; it reflects the pressure in the left ventricle at end diastole (LVEDP)

Normal Carbon Monoxide?

infant: less/= 12%

hypostatic pneumonia

inflammation of the lung from stasis or pooling of secretions from lack of movement and exercise

what are nonsteroidal anti-inflammatories?

inhibit prostaglandin synthesis adverse effects: headache, eye changes, dizziness, GI disturbances, bleeding, rash

To inflate the cuff of an endotracheal tube (ET) when the patient is on mechanical ventilation, the nurse ______________

injects air into the cuff until a slight leak is heard only at peak inflation. rationale: The minimal occluding volume technique involves injecting air into the cuff until an air leak is present only at peak inflation. The volume to inflate the cuff varies with the ET and the patient's size. Cuff pressure should be maintained at 20 to 25 mm Hg. An accurate assessment of cuff pressure cannot be obtained by palpating the pilot balloon.

Describe the preoperative nursing care for a client undergoing a laryngectomy:

involve the family and patient in manipulation of the tracheostomy equipment before surgery, plan acceptable communication methods, refer to the speech pathologist, discuss rehabilitation program

What is a superficial partial thickness (2nd degree) burn?

involves the entire epidermis and top 3rd of dermis (can be either superficial or deep partial thickness) and causes redness, blistering, moisture, and blanches with pressure; very painful d/t nerve exposure, becomes itchy and dry during healing,

What is a full thickness burn (3rd degree)?

involves the epidermal, and entire dermal layer and cannot regenerate self; causes little pain, and is surrounded by 2nd degree burns, appears black, waxy white, pale, marbled, yellow, or red; can have black lines d/t thrombosed vessels; has eschar

Hemoglobin - definition

iron-containing protein in red blood cells that carries oxygen for delivery to cells

Nitroglycerine

is administered up to 3 times (every 5 minutes). If chest pain does not stop go to hospital. Do not give when BP is < 90/60.

Describe the nurse's d/c instructions to a patient with venous peripheral vascular disease:

keep extremities elevated when sitting, rest at first sign of pain, keep extremities warm (but not with a heating pad) change position often, avoid crossing legs, wear unrestricted clothing

methadone

is an opioid analgesic used to detoxify/treat pain in narcotic addicts.

Amiodarone

is effective in both ventricular and atrial complications.

TPN

is given in subclavian line.

Protonix

is given prophylactically to prevent stress ulcers

Glaucoma intraocular pressure

is greater than the normal (22 mm Hg), give miotics to constrict (pilocarpine) NO ATROPINE.

Mucomyst

is the antedote to tylenol and is administered orally

weight

is the best indicator of dehydration

CVA

is with dead brain tissue.

hemophilia

is x-linked. Mother passes disease to son

What is the purpose of a spacer?

it helps the medication reach the lungs used in children and elderly helps avoid mouth fungus,nervousness and other side effects

Neutropenic patients

no live vaccines, no fresh fruits, no flowers should be used

Prolapsed Cord

knee-chest position or Trendelenburg

Which of the followig is an appropriate post op nursing intervention for the pt who has had a partial or total nephrectomy?

label and secur all caths, tubes, and drains

Positioning with pneumonia

lay on the affected side to splint and reduce pain. But if you are trying to reduce congestion the sick lung goes up. (Ever had a stuffy nose, and you lay with the stuff side up and it clears?)

Urine: RBC - Lab Values

less than or equal to 2

With flecainide (Tambocor), an antiarrythmic:

limit fluids and sodium intake, because sodium increases water retention which could lead to heart failure.

Tylenol poisoning

liver failure possible for about 4 days. Close observation required during this time-frame, as well as tx with Mucomyst.

Common sites for metastasis include the:

liver, brain, lung, bone, and lymph

after Thyroidectomy

low or semi-Fowler's, support head, neck and shoulders.

3 systems that maintain acid-base balance:

lungs, kidneys, chemical buffers

5 nursing interventions after chest tube insertion:

maintain a dry occlusive dressing on the chest tube, keep all tubing connections tight and taped, monitor the patients clinical status, encourage the patient to breathe deeply periodically, monitor the fluid drainage and mark the time of measurement and the fluid level

What d/c instructions should be given to a patient who has had urinary calculi?

maintain a high gluid intake, 3-4 L per day, follow prescribed diet and avoid a supine position

Important nursing interventions to prevent acute renal railure in the critically ill pt include which of the follwing?

maintain fluid voluem and cardiac output

what is the nursing care for a total hip arthroplasty?

maintain hip in abduction flexed no more than 45-60 degrees, HOB no more than 45 degrees, accommodate bed pan for hip

Jews

no meat and milk together

Partial Thromboplastin Time - Definition

measures the presence of plasma factors that act in a portion of the coagulation pathway

pitocin

med used for uterine stimulation

The first sign of pyloric stenosis in a baby

mild vomiting that progresses to projectile vomiting. Later you may be able to palpate a mass, the baby will seem hungry often, and may spit up after feedings.

Transient Ischemic Attack (TIA)

mini stroke with no dead brain tissue, precursor to MI

When assisting with insertion of a pulmonary artery (PA) catheter, the nurse identifies that the catheter is correctly placed when the _______________

monitor shows a typical PAWP tracing. rationale: The purpose of a PA line is to measure PAWP, so the catheter is floated through the pulmonary artery until the dilated balloon wedges in a distal branch of the pulmonary artery, and the PAWP readings are available. After insertion, the balloon is deflated and the PA waveform will be observed. Systemic arterial pressures are obtained using an arterial line. The length of catheter needed for insertion will vary with patient size.

During mechanical ventilation, what are the three major nursing interventions?

monitor the patient's respiratory status and secure connections, establish a communication mechanism with the client, keep the airway clear by coughing and suctioning

While waiting for cardiac transplantation, a patient with severe heart failure has a ventricular assist device (VAD) implanted. When developing the plan of care, the nursing actions should include ___________

monitoring the surgical incision for signs of infection. rationale: The insertion site for the VAD provides a source for transmission of infection to the circulatory system and requires frequent monitoring. Patient's with VADs are able to have some mobility and may not be on bed rest. The VAD is a bridge to transplantation, not a permanent device. Immunosuppression is not necessary for nonbiologic devices like the VAD.

Describe the physical appearance of patients who have Cushing's disease

moon face, obesity in trunk, buffalo hump in back, muscle atrophy and thin skin

Albumin - Defination

most abundant plasma protein, 60% of the total protein, made by the liver, plays an important role in osmotic balance, contributes to the viscosity of blood, transportation of lipids/hormones/calcium..., and helps to maintain pH

How does a Stage 1 pressure ulcer appear?

nonblanchable, erythema, intact skin, skin discoloration, warmth and hardness

transmission of pain

movement of pain impulses from the periphery to the spinal cord & then to the brain

Bence Jones protein in the urine confirms:

multiple myeloma

Hypokalemia

muscle ewakness, dysrhythmias, increase K (raisins, bananas, apricots, oranges, beans, potatoes, carrots, celery)

Hypercalcemia

muscle weakness, lack of coordination, abdominal pain, confusion, absent tendon reflexes, sedative effect on CNS

Diaphragm

must stay in place 6 hours after intercourse. They are also fitted so must be refitted if you lose or gain a significant amount of weight.

Multiple Sclerosis

myelin sheat destruction, disruption in nerve impulse conduction.

Acetaminophen antedote:

n-Acetylcysteine

Addisonian crisis

n/v, confusion, abdominal pain, extreme weakness, hypoglycemia, dehydration, decreased BP

Hyponatremia

nausea, muscle cramps, increased ICP, muscular twitching, convulsion; osmotic diuretics, fluids

Urea Nitrogen - defination

nitrogenous end product of protein metabolism

Kids with RSV:

no contact lenses or pregnant nurses in rooms where ribavirin is being administered by hoot, tent, etc.

Gastric Ulcer pain

occurs 30 minutes to 90 minutes after eating, not at night, and doesn't go away with food

Spinal shock

occurs immediately after spinal injury

During Internal Radiation

on bedrest while implant in place

What is a superficial burn (1st degree)?

only epidermal layer causes erythema, edema, pain, and blanching

Glucagon increases the effects of:

oral anticoagulants.

That does a +4 on the RASS scale mean?

overtly combative, violent, immediate damage to staff; one of the worst scores

The nurse is reviewing a clients ABG report. What ABG value reflects the acid concentration in the client's blood?

pH

Urine pH?

pH of 4.6 - 8 with 6 as avg.

Demerol NOT morphine sulfate

pain med for pancreatitis

pathological jaundice- 24 hrs (lasts 7 days) physiological- after 24 hrs

pathological vs physiological jaundice

Glaucoma

patients lose peripheral vision. Treated with meds

amniocentesis

perform it before 20 weeks gestation to check for cardiac and pulmonary abnormalities.

droplet

pertussis, mumps (for 9 days), seasonal influenza, mono for 24 hours, rubella,

. 1 t (teaspoon)= 5 ml 1 T(tablespoon)= 3 t = 15 ml 1 oz= 30 ml 1 cup= 8 oz 1 quart= 2 pints 1 pint= 2 cups 1 gr (grain)= 60 mg 1 g (gram)= 1000 mg 1 kg= 2.2 lbs 1 lb= 16 oz * To convert Centigrade to F. F= C+40, multiply 9/5 and substract 40 * To convert Fahrenheit to C. C= F+40, multiply 5/9 and substract 40

pharma conversions

Atropine used to decrease secretions Phenergan an antiemetic used to reduce nausea Diazepam is a commonly used tranquilizer given to reduce anxiety before OR Demerol is for pain control Do not give demerol to pts. with sickle cell crisis. Iron injections should be given Z-track so they don't leak into SQ tissues.

pharma tips

what is PKU?

phenylketonuria, genetic disorder caused by a deficiency in liver enzyme phenylalanine hydroxylase if not recognized, phenyl ketone buildup causes MR

Factors affecting sleep

physical illness drugs and substances emotional stress environment lifestyle exercise and fatigue food and caloric intake sound

What immediate action should the nurse take when a chest tube becomes disconnected from a bottle or suction apparatus?

place the end of the tube in sterile water at a 2cm level, apply an occlusive dressing and notify health care provider stat

Cord compression

place the mother in the TRENDELENBERG position because this removes pressure of the presenting part off the cord. (If her head is down, the baby is no longer being pulled out of hte body by gravity)

What immediate actions should the nurse implement when a client is having a myocardial infarction?

place the patient on strict bedrest to lower oxygen demands on the heart, administer O2 by nasal cannula at 2-5 L/min, take measures to alleviate pain and anxiety, administer prn pain medications, and antianxiety medications

what is placenta previa?

placenta abnormally implanted near or over cervical os indications: painless, bright red blood and soft uterus bw 29-30 weeks interventions: bedrest, ultrasound to locate placenta, not vaginal/ rectal exams before fetal viability, amniocentesis for lung maturity, daily hgb/ hct checks, cross-matched blood available

placenta previa = there is no pain, there is bleeding. abruptio Placentae = pain, but no bleeding.

placenta previa vs. abruptio placentae

The intensive care unit (ICU) charge nurse will determine that teaching about hemodynamic monitoring for a new staff nurse has been effective when the new nurse ______________

positions the zero-reference stopcock line level with the phlebostatic axis. rationale: For accurate measurement of pressures, the zero-reference level should be at the phlebostatic axis. There is no need to rebalance and recalibrate monitoring equipment hourly. Accurate hemodynamic readings are possible with the patient's head raised to 45 degrees or in the prone position. The anatomic position of the phlebostatic axis does not change when patients are repositioned.

After catheter removal in a TURP patient, what day should bleeding subside?

postoperative day 4

angiotensin II

potent vasodialator in the lungs. Aldosterone attracts sodium.

Autonomic dysreflexia

potentially life threatening emergency - elevate head of bed to 90 degree - loosen constrictive clothing - assess for bladder distention and bowel impaction (triger) - Administer antihypertensive meds (may cause stroke, MI, seisure )

Potassium

potentiates dig toxicity.

PTU and Tapazole

prevention of thyroid storm

A patient's age, gender,anxiety, culture, and __________ influence the pain experience

previous experience&meaning of pain

what do airborne precautions include?

private room with monitored negative air pressure with 6-12 air changes/h, keep door closed and patient in room, mask on patient if being transported, can cohort with other patients if same organism

transduction of pain

process that begins in the periphery when pain-producing stimulus send an impulse across a peripheral nerve fiber

What is apneustic breathing?

prolonged inspiration followed by a pause followed by expiration and a possible pause

A pt tells the nurse that she has had to change her primary care physician four times in the past three years because of changes in her health insurance at work and asks what is happening to health care. The best response by the nurse includes information that :

prospective payment systems and preferred provider organizations are used to provide more cost-effective health care.

NSAIDs inhibit the synthesis of

prostaglandins

Describe universal precautions:

protection from blood and body fluids is the goal of universal precautions

perception of pain

protects the body from damage, and is stimulated by extremes of pressure and temperature, as well as chemicals released from damaged tissues(physical component)

In providing community based patient care, nurses

provide coordinated and continues care for pts and their familes where they are in the community or in the home.

After Lumbar puncture

pt lies in flat supine (to prevent headache and leaking of CSF)

after cataract surgery

pt will sleep on unaffected side with a night shield for 1-4 weeks.

What is PAD?

pulmonary artery diastolic pressure, the lowest pressure during diastole of the right ventricle

When monitoring for the effectiveness of treatment for a patient with left ventricular failure, the most important information for the nurse to obtain is ______________

pulmonary artery wedge pressure (PAWP). rationale: PAWP reflects left ventricular end diastolic pressure (or left ventricular preload). Because the patient in left ventricular failure will have a high PAWP, a decrease in this value will be the best indicator of patient improvement. The other values would also provide useful information, but the most definitive measurement of improvement is a drop in PAWP.

Brachial pulse

pulse area cpr on an infant.

A pericardial thump is only indicated in

pulseless VT of VF or when ventricular asytole on the motnitor responds to a thump with a QRS complex

what are indications of cocaine abuse?

pupillary dilation, tachy/bradycardia, altered BP, sweating, chills, n/ v, weight loss, euphoria, talkativeness, hypervigilance May have impaired social fxning, episodes of violence, fighting, grandiosity, visual/ tactile hallucinations

nonblanchable hyperemia

redness of the skin due to dilation of the superficial capillaries. The redness persists when pressure is applied to the area, indicating tissue damage

blanchable hyperemia

redness of the skin due to dilation of the superficial capillaries. When pressure is applied to the skin, the area blanches, or turns a lighter color

What is ataxic respirations?

respirations that are completely irregular in pattern and depth with irregular periods of apnea

After kidney surgery what are the primary assessment the nurse should make?

respiratory status, breathing is guarded because of pain, circulatory status, the kidney is very vascular, pain assessment, urinary assessment and most importantly the urinary output

RN to RN assignments transfer

responsibility and accountability

hypoxia

restless, anxious, cyanotic tachycardia, increased resps. (also monitor ABG's)

what is the treatment for PKU?

restrition of foods containing phenylalanine (meat, eggs, beans, bread) and blood levels to monitor efficacy of dietary restriction

Paracentesis

semi fowlers or upright on edge of bed, empty bladder. Post- v.s., report elevated temp, observe for signs of hypovolemia.

post-thyroidectomy

semi-Fowler's, prevent ncek flexion/hyperextension, trach at bedside

What does the Braden scale assess for?

sensory preception, mobility, moisture, activity, nutrition, friction and shear; the higher score the lesser chance of developing pressure sore

Anectine is used for:

short-term neuromuscular blocking agent for procedures like intubation and ECT. Norcuron is for intermediate or long-term.

During epidural puncture

side-lying

Fat embolism

sign is petechiae. Treated with heparin.

Blood transfusion

sign of allergies in order: 1)Flank pain 2)Frequent swallowing 3)Rashes 4)Fever 5)Chills

Trousseau and Chovstek's signs

signs of hypocalcemia

Myxedema/hypothyroidism:

slowed physical and mental function, sensitivity to cold, dry skin and hair

What is the most common risk factor associated with lung cancer?

smoking

Urine: Ketones - Nursing Implications: general - diet

special diets (carb-free, high protien, high fat) may cause ketonuria

How do patients experiencing angina describe their pain?

squeezing, heavy, burning, radiates to left arm or shoulder, transient or prolonged

what tasks can NAP be assigned

stable patients (could be complex also) and tasks that are routine, simple, repetitive, everyday activities that don't require nursing judgment such as feeding, hygiene, ambulation

An appropriate goal of nursing care for a pt with end-stage renal disease is the pt will be able to do which of the following?

state the advantages and disadvantages of types of renal replacement therapies

What are the most important nursing interventions for a client with possible renal calculi?

straining all urine is the most important intervention, then accurate I&O, and administering pain meds

What does the CD4 T-cell count describe?

the number of infection-fighting lymphocytes the patient has

What is the most important assessment data for the nurse to obtain in a client with an arrhythmia?

the patients ability to tolerate the arrhythmia

with delegation, you can transfer

the responsibility but not the accountability.

what is the Guthrie test?

the screening for PKU

Why does the CD4 T-cell count drop in HIV infections?

the virus destroys CD4 T-cells as it invades them and replicates them

assignment

the work you must get accomplished during your shift

milieu therapy= taking care of patient/environment cognitive therapy= counseling crisis intervention=short term.

therapies

Calcium - Name a drug type that may cause increased Ca Blood values?

thiazide diuretics can cause increased calcium values

Describe ways a pediatric patient may acquire HIV infection:

through infected blood products, through sexual abuse, and through breast milk

During the oliguric phase of renal failure, proteins should be severely restricted, what is the rationale

toxic metabolites (urea and creatinine) that accumulate in the blood are derived mainly from protein catabolism

what are indications of delirium tremens?

tremors, anxiety, panic, disorientation, hallucination (alcohol hallucinosis 48h after last drink including auditory, visual, or tactile), vomitting, grand mal seizures (first 48h after withdrawal)

Hypomagnesemia

tremors, tetany, seizures, dyrshythmias, depression, confusion, dysphagia; dig toxicity

Healing by primary intention is expected when the edges of a clean surgical incision are sutured or stapled together, tissue loss is minimal or absent, and the wound is uncontaminated by microorganisms. True False

true

Peritoneal Dialysis when Outflow is Inadequate

turn pt from side to side BEFORE checking for kinks in tubing (according to Kaplan)

Late deceleration

turn the mother to her left side, to allow more blood flow to the placenta

Ritalin

tx of ADHD..assess for heart related side effects report immediately...child may need a drug holiday b/c it stunts growth.

Amphojel

tx of GERD and kidney stones....watch out for contipation

Apresoline(hydralazine)-

tx of HTN or CHF, Report flu-like symptoms, rise slowly from sitting/lying position; take with meals.

Librium

tx of alcohol w/d...dont take alchol with this...very bad nausea and vomiting can occur

Oncovin (vincristine)

tx of alcohol w/d...dont take alchol with this...very bad nausea and vomiting can occur

Vistaril

tx of anxiety and also itching...watch for dry mouth. given preop commonly

Theophylline

tx of asthma or COPD..therap drug level: 10-20

Carafate

tx of duodenal ulcers..coats the ulcer...so take before meals.

Diamox

tx of glaucoma, high altitude sickness...dont take if allergic to sulfa drugs

Timolol (Timoptic)

tx of gluacoma

dopamine (intropine)

tx of hypotension, shock, low cardiac output, poor perfusion to vital organs...monitor EKG for arrhythmias, monitor BP

Bentyl

tx of irritable bowel....assess for anticholinergic side effects.

Cogentin

tx of parkinson and extrapyramidal effects of other drugs

Tigan

tx of parkinson and extrapyramidal effects of other drugs

Sinemet

tx of parkinson...sweat, saliva, urine may turn reddish brown occassionally...causes drowsiness

Artane

tx of parkinson..sedative effect also

Kwell

tx of scabies and lice...(scabies)apply lotion once and leave on for 8-12 hours...(lice) use the shampoo and leave on for 4 minutes with hair uncovered then rinse with warm water and comb with a fine tooth comb

Dilantin

tx of seizures. thera drug level: 10-20

Name the necessary elements to include in teaching a new diabetic:

underlying cause of the disease, it's management, meal planning, exercise, insulin administration, sick-day management, symptoms of hyperglycemia, symptoms of hypoglycemia

What is oliguria?

urine output of 300-500ml/day

To verify the correct placement of an endotracheal tube (ET) after insertion, the best initial action by the nurse is to _______________

use an end-tidal CO2 monitor to check for placement in the trachea. rationale: End-tidal CO2 monitors are currently recommended for rapid verification of ET placement. Auscultation for bilateral breath sounds and checking chest expansion also are used, but they are not as accurate as end-tidal CO2 monitoring. A chest x-ray confirms the placement but is done after the tube is secured.

Hemovac

used after mastectomy, empty when full or q8hr, remove plug, empty contents, place on flat surface, cleanse opening and plug with alcohol sponge, compress evacuator completely to remove air, release plug, check system for operation.

continuous passive motion machine

used in knee replacement

Dopamine

used to increase BP since cardiac output decreases with dysrythmias.

How is the patient positioned in the immediate postoperative period and why?

usually on the side or with the head to the side in order to prevent aspiration of any emesis

airborne and contact

varicella and disseminated shingles

what are the indications for rheumatoid arthritis?

vascular granulation tissue called pannus destroys cartilage and bone causing painful, swollen, red, warm, stiff joints (esp in the AM), fatigue, anorexia, weight loss, decreased mobility

The parathyroid gland relies on the presence of:

vitamin D to work.

Creatinine - defination

waste product of muscle metabolism, filtered out of the blood by the kidneys and excreted in urine

phenylalanine

when it increases, brain problems occur.

Myasthenia Gravis

worsens with exercise and improves with rest

evisceration

wound separation with protrusion of organs

closed wound

wound that involves underlying tissue without break in the skin

primary intention

wounds that heal under conditions of minimal tissue loss(partial thickness)

How often is HIV transmitted from mother to fetus (vertical transmission) if the mother is NOT treated during pregnancy?

30-50% of the time

What condition results from all treatments for hyperthyroidism?

hypothyroidism resulting in thyroid hormone replacement

Isotonic fluids:

LR and NS

4 nursing interventions for assisting the client to cough productively:

encourage deep breathing, increase fluid intake to 3 L per day, use humidity to loosen secretions, suction airway to stimulate coughing

What condition increases the likelihood that digitalis toxicity will occur?

hypokalemia, which is more common when diuretics and digitalis preparations are given together

List 3 nursing actions that prevent postoperative wound dehiscence and evisceration

teach the patient to splint the incision when coughing, encouraging coughing, and deep breathing early in the postoperative period when the sutures are strong, monitor for signs of infection, malnutrition and dehydration, and encourage a high protein diet.

How does past experience with pain influence current pain experience?

the more pain experience in childhood the greater the perception of pain in adulthood or with current pain experience

What lab values should be monitored daily in a patient with thrombophlebitis who is undergoing anticoagulant therapy?

PTT, PT, Hgb, Hct and platelets

Identify 3 nursing/medical interventions to prevent postoperative paralytic ileus:

early ambulation, limiting narcotic analgesics, NG tube decompression


Set pelajaran terkait

Listening Guide Quiz 2: Hildegard: Alleluia, O virga mediatrix

View Set

MOLECULAR GENETICS UTOLEDO QIAN CHEN

View Set

Ch 3: Job Analysis in HR Selection

View Set

Quiz 5- Ocean Climate Change, Climate Change So Far

View Set

Information Processing Chapter 7

View Set

PSY MODULE 40 BASIC CONCEPTS OF PSYCHOLOGY DISORDERS

View Set